Sunteți pe pagina 1din 76

Principii şi metode aplicate la rezolvarea problemelor

de matematică

1. Principiul parităţii
În matematica elementară întâlnim multe probleme care folosesc noţiunea de paritate.
Principiul parităţii constă în separarea cazurilor pare şi impare dintr-o situaţie dată.

Regulile parităţii:
- suma a două numere pare este un număr par
- suma a două numere impare este un număr par
- suma dintre un număr par şi altul impar este un număr impar
- produsul a două numere pare este un număr par
- produsul a două numere impare este un număr impar
- produsul dintre un număr par şi un număr impar este un număr par.

Exerciţii rezolvate :
1.1. Demonstraţi că dacă suma a două numere întregi este un număr impar, atunci
produsul lor este un număr par.

Soluţie.
Fie a şi b numerele date. Din ipoteză a + b = 2n + 1, n Є N. Deci unul din numerele a sau b este par.
Fie a = 2k .
Atunci b = 2n + 1 – a = 2n + 1 - 2k = 2(n - k) + 1, adică b este impar.
Atunci a x b este produsul dintre un număr par şi altul impar, deci va fi impar.

1.2. Demonstraţi că 2n (n ≥ 2,n Є N ) se poate scrie ca o sumă de două numere


naturale impare consecutive, iar 3n se poate scrie ca o sumă de trei numere
naturale consecutive şi ca sumă a trei numere impare consecutive.

Soluţie.
Pentru orice n ≥ 2,n Є N, 2n este număr par.
Avem: 2n = 2 x 2n-1= 2n-1 + 2n-1 = (2n-1 - 1) + (2n-1 + 1), unde evident pentru n ≥ 2,n Є N numerele
(2n-1 - 1) şi (2n-1 + 1) sunt numere impare consecutive .

Pentru orice n ≥ 2,n Є N, 3n Є N şi 3n = 3 x 3n-1 = 3n-1 + 3n-1 + 3n-1 = (3n-1 -1 ) + 3n-1 + (3n-1 + 1 ) , unde
pentru n ≥ 2,n Є N , numerele (3n-1 -1 ) , 3n-1 şi (3n-1 + 1 ) sunt numere consecutive.

Pe de altă parte pentru n ≥ 2,n Є N mai avem


3n = 3 x 3n-1 = 3n-1 + 3n-1 + 3n-1 = (3n-1 - 2 ) + 3n-1 + (3n-1 + 2 ) , unde pentru n ≥ 2,n Є N ,
numerele (3n-1 - 2 ) , 3n-1 şi (3n-1 + 2 ) sunt numere impare consecutive.

1.3. Se consider şirul numerelor naturale de la 1 la 1979 adică :1,2,3,4,...,1977,1978,1979.


Luaţi la întâmplare oricare două numere din acest şir şi înlocuiţi-le cu diferenţa
lor ( din cel mai mare scădem pe cel mai mic ). La fiecare operaţie de acest fel
numărul numerelor din şir scade cu unu (fiindcă am înlocuit două numere cu
unul singur). Continuăm operaţiunea până la final când obţinem un singur număr.
Să se arate că acest număr este par.
1
Soluţie.
La fiecare etapă a operaţiei descrise, numărul numerelor impare din şir rămâne neschimbat sau descreşte cu doi, deoarece dacă,
în primul caz, luăm un număr par şi unul impar, diferenţa lor este impară, deci am înlocuit numărul impar cu un alt număr
impar .
Dacă, în al doilea caz luăm două numere impare, atunci diferenţa lor este un număr par, deci numărul numerelor impare scade cu
doi deoarece am înlocuit două numere impare cu unul par. În şirul 1,2,3,4, ... ,1979 avem (1+1979)/2 numere impare, adică
990 numere impare.
La fiecare pas rămâne un număr par de numere impare şi atunci ultimul număr va fi cu siguranţă par.

1.4. Se consideră k numerele naturale impare, respectiv : n1 ,n2 , n3 ...,nk .


n1  n2 n2  n3 n3  n4 n n n n
Să se demonstreze că printre numerele: ; ; ;...; k 1 k ; k 1
2 2 2 2 2
există un număr impar de numere impare.

Soluţie.
Suma a două numere impare este un număr par, deci numerele
n1  n2 n2  n3 n3  n4 n n n n
; ; ;...; k 1 k ; k 1 sunt numere naturale.
2 2 2 2 2
Să presupunem că printre acestea se află un număr par de numere impare.
Atunci suma lor respectiv

n1  n2 n2  n3 n3  n4 n n n n
   ...  k 1 k  k 1  n1  n2  n3  ...  nk 1  nk
2 2 2 2 2
este un număr par.
Dar aceeaşi sumă este suma unui număr impar de numere impare, deci ea este un număr impar.
Contradicţie.
Deci presupunerea făcută a fost falsă şi atunci , printre numerele considerate în ipoteză există un
număr impar de numere impare.

1.5. Se consideră un număr natural impar n şi 2003 numere, fiecare dintre ele egale
cu n sau – n. Arătaţi că nu putem împărţi cele 2003 numere în două grupe, astfel
încât suma numerelor dintr-o grupă să fie egală cu suma numerelor din
cealaltă grupă.
Soluţie.

Deoarece 2003 este număr impar, rezultă că numărul numerelor dintr-o grupă şi numărul numerelor din cealaltă
grupă sunt de parităţi diferite. Să presupunem că prima grupă conţine un număr impar de numere şi atunci a doua grupă va
conţine un număr par de numere. Deoarece în prima grupă sunt un număr impar de numere, rezultă că în această grupă numărul
numerelor egale cu n şi cel al numerelor egale cu –n sunt de parităţi diferite, deci suma numerelor din această grupă este impară.
Pentru a doua grupă avem situaţiile :
1) grupa conţine un număr par de numere egale cu n şi un număr par de numere egale cu –n, deci suma numerelor din această
grupă este pară ;
2) grupa conţine un număr impar de numere egale cu n şi un număr impar de numere egale cu –n, deci suma numerelor din
această grupă este de asemenea pară ;
În concluzie suma numerelor dintr-o grupă este pară iar suma numerelor din cealaltă grupă este impară şi deci egalitatea nu poate
avea loc.

1.6. Suma unui număr par de numere impare consecutive este 7984. Să se găsească
aceste numere.
Soluţie.
Considerăm un şir par de numere consecutive impare, adică :

2n + 1, 2n + 3, 2n + 5, …, 2n + 2k + 1 , unde 2n apare de k + 1 ori.


Rezultă :

2
2n + 1 + 2n + 3 + 2n + 5 + … + 2n + 2k + 1 = 7984 (1)
Deoarece 2n apare de k + 1 ori şi respectiv :

1 + 3 + 5 + … + ( 2k + 1 ) = ( k + 1 )2 rezultă că ( 1 ) devine

2n( k + 1 ) + ( k + 1 )2 = 7984  ( k + 1 )(2n + k + 1 ) = 7984 = 16  499

Din ipoteza k + 1 este par atunci 2n + k + 1 este tot număr par şi din ultima egalitate rezultă :

 k 1  2  k 1  4  k 1  8
a)  b)  c)  din care rezultă soluţiile :
 2n  k  1  8  499 2n  k  1  4  499 2n  k  1  2  499
a) { 3991 , 3993 }
b) { 1993, 1995, 1997, 1999 }
c) { 991, 993, 995, 997, 999, 1001, 1003, 1005 }

1.7. Numerele 1, 2, 3, 4, … , 100 sunt scrise pe 100 de bileţele ( câte unul pe fiecare
bileţel ). Se aleg la întâmplare două bileţele, se calculează diferenţa dintre numărul
mai mare şi numărul mai mic de pe cele două bileţele şi se scrie aceasta pe un
nou bileţel.
Se aruncă cele două bileţele şi în locul lor se pune bileţelul cu diferenţa calculată.
Procedăm astfel până când rămâne un singur bileţel cu valoarea ultimei diferenţe.
Ce paritate are numărul scris pe acesta bileţel ?
Soluţie.
După o operaţie de tipul x – y cu x > y rezultă că se înlocuiesc numerele x şi y cu x – y.
Deoarece x + y şi x – y au aceeaşi paritate rezultă că orice operaţie de tipul din enunţ lasă neschimbată paritatea elementelor
sumei numerelor de pe bilete.
100 101
Suma 1  2  3  ...  100   5050 , deci atunci rezultă că numărul de pe ultimul bileţel va fi un număr par.
2

1.8. Se consideră şirul 91, 92, 93, … , 92010 . Se aleg din acest şir doi termeni, se împarte cel mai mare la
cel mai mic, iar în şir, jn locul lor se pune câtul împărţirii.Se continuă operaţiunea până când
rămâne un bilet cu valoarea ultimul cât calculat. Ce paritate are acest cât şi care este ultima sa
cifră ?
Soluţie.
După o operaţie de tipul celei din enunţ se înlocuiesc numerele 9x şi 9y cu 9x – y , unde x > y .
Ultimul termen care se va obţine va avea forma 9z . Ultima sa cifră va fi 1 sau 9 după cum z este par sau impar.
Dar x + y şi x – y au aceiaşi paritate şi anume este paritatea sumei 1 + 2 + 3 + … + 2010 = 1005 x 2011 deci suma este impară ,
rezultă atunci că ultima cifră va fi 9.

11m  57
1.9. Dacă m Є N, cercetaţi dacă fracţia este ireductibilă .
7 m  13
Soluţie.
Deoarece 11m şi 7m sunt numere impare atunci numărătorul şi numitorul, ca sume de două numere impare, vor fi pare. Deci
fracţia se simplifică cu 2 şi deci nu este ireductibilă .

1.10. Un elev a rupt dintr-o carte, la întâmplare, un număr de 11 foişi a adunat numerele
tuturor paginilor rupte, obţinând suma S. Este posibil ca S să fie divizibilă cu 4 ?
Soluţie.
Paginile unei foi sunt două numere consecutive, deci suma lor este un număr impar. Numărul foilor rupte este 11,
deci suma tuturor paginilor va fi egală cu suma a 11 numere impare, adică S va fi tot un număr impar.
Deci S nu este divizibilă cu 4.
3
5  (1) m  2  (1) n 1
1.11. Se consideră numărul F  , unde m, nЄ Z.
4  (1) 2 n  a  (1) m
Să se determine numerele naturale a pentru care expresia F este
număr întreg .
Soluţie.
Avem evident cazurile :
1) n par şi m impar
Avem ( -1 )m = -1; ( - 1 )n+1 = -1 , ( -1 )2 - n = 1 şi atunci
5  (1) m  2  (1) n 1 5  2 7 7
F 2n
  
4  (1)  a  (1) m
4 a 4 a a4
2) n impar şi m par
Avem ( -1 )m = 1; ( - 1 )n+1 = 1 , ( -1 )2 - n = -1 şi atunci

5  (1) m  2  (1) n 1 5 2 7
F 2n
 
4  (1)  a  (1) m
4  a a  4

3) n şi m impare
Avem ( -1 )m = -1; ( - 1 )n+1 = 1 , ( -1 )2 - n = -1 şi atunci

5  (1) m  2  (1) n 1 5  2 3 3
F 2n
  
4  (1)  a  (1) m
4  a  a  4 a  4

4) m şi n pare
Avem ( -1 )m = 1; ( - 1 )n+1 = -1 , ( -1 )2 - n = 1 şi atunci

5  (1) m  2  ( 1) n 1 5  2 3
F 2n
 
4  (1)  a  (1) m
4a a4
Din cele patru cazuri rezultă
7
a) F Є Z , deci a Є { -3, 3, 5, 11 } şi respectiv
a4
3
b) F  Є Z , deci a Є { -7, -5, -3, -1 }
a4
de unde rezultă că soluţiile naturale sunt doar a Є { 3, 5, 11 }

1.12. Determinaţi numerele naturale n, n ≥ 2, pentru care există o alegere a


semnelor + şi – astfel încât 1 ± 2 ± 3 ± 4 ± … ± n = 0 .
Soluţie.
Fie { x1, x2, x3, …. , xk, y1, y2, y3, …. , ym } = { 1, 2, 3, … , n } , unde k + m = n
Pentru a avea loc relaţia din enunţ trebuie ca x1 + x2 + x3 + …. + xk = y1+ y2 + y3 + …. + ym
Deoarece x1 + x2 + x3 + …. + xk + y1+ y2 + y3 + …. + ym =2(x1 + x2 + x3 + …. + xk ) şi
n(n  1)
x1 + x 2 + x 3 + .... + x k + y1 + y 2 + y3 + .... + ym =1+ 2 + 3 + ... + n=-
2
n(n  1)
Rezultă că trebuie să fie un număr par adică trebuie ca n = 4p cu pЄN* sau n = 4p + 3 cu pЄN .
2
Condiţia este şi suficientă deoarece
1) Dacă n = 4p cu pЄN*, atunci alegând semnele ca mai jos, avem :
1 ± 2 ± 3 ± 4 ± … ± n = 1 – 2 + 3 – 4 + … + ( 2p – 1) – 2p - ( 2p + 1 ) + ( 2p + 2 ) - … - ( 4p – 1 ) + 4p =

= [ ( 1 – 2 ) + ( 3 – 4 ) + … + ( 2p – 1 – 2p ) ] + {[ -( 2p+1 )+ ( 2p + 2 )] + [ -( 2p+3 ) + ( 2p + 4 )] + … +

+ [ - ( 4p – 1 ) + 4p ] } = ( - 1 – 1 – 1 - … - 1 ) + ( 1 + 1 + 1 . .. + 1 ) = 0
p termeni p termeni

4
2) Dacă n = 4p + 3 cu pЄN , alegând corespunzător semnele rezultă :

1 ± 2 ± 3 ± 4 ± … ± n = 1 – 2 + 3 – 4 + … + ( 2p – 1) – 2p - ( 2p + 1 ) + ( 2p + 2 ) - … - ( 4p + 2 ) + ( 4p + 3) =

= ( 1 + 2 - 3 ) + {( 4 – 5 ) + ( 6 – 7 ) +… + [( 2p + 2 ) – ( 2p + 3 )} + [ -( 2p+4 )+ ( 2p + 5 )] +
+ [ -( 4p+2 ) + ( 4p + 3 )] } = 0 + ( - 1 – 1 – 1 - … - 1 ) + ( 1 + 1 + 1 . .. + 1 ) = 0
p termeni p termeni

1.13. Determinaţi numerele naturale a şi b ştiind că a2 = 2069 + 2b + a.


Soluţie.
Avem succesiv : a2 – a = 2069 + 2b  a( a – 1 ) = 2069 + 2b .
Cum membrul stâng este par rezultă 2b număr impar adică b = 0 şi 2b = 20 = 1.
Atunci avem : a( a – 1 ) = 2070  a( a – 1 ) = 45 x 46 , deci a = 45 .

1.14. Determinaţi numerele naturale de forma abba , cu a şi b prime , astfel încât :

abba + ab + ba + 1 să fie număr par .


Soluţie.
Avem a şi b cifre şi în plus numere prime, deci a, b Є { 2, 3, 5, 7 }.
1) Dacă a = 2 , atunci rezultă
2bb2 + 2b + b2 + 1 = 2k ( număr par )
Rezultă 2bb2 + 2 + b trebuie să fie număr impar  2b + b2 număr impar  adică b2 număr impar de
b 2

unde obţinem :
1.1. b = 3 şi atunci abba = 2332 iar abba + ab + ba + 1 = 2332 + 23 + 32 + 1 =
= 2332 + 8 + 9 + 1 = 2350 sau

1.2. b = 5 şi atunci abba = 2552 iar abba + ab + ba + 1 = 2552 + 25 + 52 + 1 =


= 2552 + 32 + 25 + 1 = 2610 sau

1.3. b = 7 şi atunci abba = 2772 iar abba + ab + ba + 1 = 2772 + 27 + 72 + 1 =


= 2772 + 128 + 49 + 1 = 2950

2) Dacă a = 3 , atunci rezultă


3bb3 + 3b + b3 + 1 = 2k ( număr par )
Rezultă 3bb3 + 3 + b trebuie să fie număr impar  b3 număr impar
b 3

2.1. b = 3 şi atunci abba = 3333 iar abba + ab + ba + 1 = 3333 + 33 + 33 + 1 = 3388 sau

2.2. b = 5 şi atunci abba = 3553 iar abba + ab + ba + 1 = 3553 + 35 + 53 + 1 =


= 3553 + 243 + 125 + 1 = 3922 sau

2.3. b = 7 şi atunci abba = 3773 iar abba + ab + ba + 1 = 3773 + 37 + 73 + 1 =


= 3773 + 2187 + 343 + 1 = 6304

3) Dacă a = 5 , atunci rezultă


5bb5 + 5b + b5 + 1 = 2k ( număr par )
Rezultă 5bb5 + 5 + b trebuie să fie număr impar  b5 număr impar
b 5

3.1. b = 3 şi atunci abba = 5335 iar abba + ab + ba + 1 = 5335 + 53 + 35 + 1 = 5704 sau

3.2. b = 5 şi atunci abba = 5555 iar abba + ab + ba + 1 = 5555 + 55 + 55 + 1 = 11806 sau

3.3. b = 7 şi atunci abba = 5775 iar abba + ab + ba + 1 = 5775 + 57 + 75 + 1 = 100708

4) Dacă a = 7 , atunci rezultă


7bb7 + 7b + b7 + 1 = 2k ( număr par )
Rezultă 7bb7 + 7 + b trebuie să fie număr impar  b7 număr impar
b 7

4.1. b = 3 şi atunci abba = 7337 iar abba + ab + ba + 1 = 7337 + 73 + 37 + 1 = 9868 sau

4.2. b = 5 şi atunci abba = 7557 iar abba + ab + ba + 1 = 7557 + 75 + 57 + 1 = 112358 sau

5
4.3. b = 7 şi atunci abba = 7777 iar abba + ab + ba + 1 = 7777 + 77 + 77 + 1 = 1654864

1.15. Suma a 10 numere naturale nenule, distincte, este 108. Arătaţi că printre acestea
cel puţin două sunt impare.

Soluţie.
Dacă toate cele 10 numere ar fi pare atunci suma primelor 10 numere naturale pare este
2 + 4 + 6 + … + 20 = 110 > 108 . Deci nu toate numerele pot fi pare .
Dacă un număr ar fi impar atunci suma este impară, deci nu convine şi atunci pot fi 2 numere impare.
Exemplu : 2 + 4 + 6 + … + 14 + 16 + 17 + 19 = 108.

1.16. a) Care este paritatea numărului natural obţinut ca diferenţă dintre suma a 2011
numere naturale impare şi suma a 2011 numere naturale pare ?
b) Stabiliţi paritatea numărului A = ( n + 3 )( 3n + 5 )( n + 8 ) + 3n , unde n Є N.

Soluţie.
a) Suma a 2011 numere impare este număr impar, iar suma a 2011 numere naturale pare este număr par.
Deci diferenţa dintre cele două sume este pară .
b) Avem ( n + 8 ) – ( n + 3 ) = 5 , deci n + 8 şi n + 3 sunt de parităţi diferite, adică unul din aceste numere este număr par.
Atunci produsul ( n + 3 )( 3n + 5 )( n + 8 ) este un număr par.
Cum 3n este număr impar, rezultă că A este număr impar .

2. Divizibilitatea numerelor naturale


Teoremă : Dacă un număr natural N are descompunerea de forma
N  p1k1  p2k2  ...  pnkn
atunci numărul divizorilor lui N este dat de formula

δ(N) = (k1 + 1)(k2 + 1 ) … (kn + 1 )

iar suma divizorilor lui N este dată de relaţia

p1k1 1  1 p2k2 1  1 pnkn 1  1


S ( (N ))    ... 
p1  1 p2  1 pn  1
Demonstraţie :
Din descompunerea N  p1k1  p2k2  ...  pnkn rezultă că lista divizorilor lui N este de
forma următorului tabel :
 Divizori daţi de puterile lui p1, adică : 1, p11 , p12 , p13 ,..., p1k1 , deci rezultă k1 + 1 divizori ;
 Divizori daţi de puterile lui p2, adică : 1, p12 , p22 , p23 ,..., p2k2 , deci rezultă k2 + 1 divizori ;
 ……..
1 2 3 k
 Divizori daţi de puterile lui pn, adică : 1, pn , pn , pn ,..., pn n , deci rezultă kn + 1 divizori .
Observăm următoarele ::
1) Orice număr din tabel este divizor al lui N
2) Linia unu conţine k1 + 1 divizori, linia doi conţine k2 + 1 divizori, ….., linia n conţine
kn + 1 divizori.
3) Dacă înmulţim, pe rând, fiecare divizor din linia întâi cu fiecare divizor din linia a doua , obţinem, în
total (k1 + 1)(k2 + 1 ) divizori ;
4) Dacă înmulţim, pe fiecare dintre aceşti divizori (k1 + 1)(k2 + 1 ) cu fiecare divizor din linia treia,
obţinem, în total (k1 + 1)(k2 + 1 )( (k3 + 1) divizori şi aşa mai departe , până când, pe baza acestui
raţionament obţinem în total (k1 + 1)(k2 + 1 ) … (kn + 1 ) divizori ai lui N.
5) În acest număr total de divizori sunt incluşi şi 1 , respectiv N ca divizori ai lui N., deci notând
cu δ( N ) numărul total al divizorilor lui N , avem :
δ(N) = (k1 + 1)(k2 + 1 ) … (kn + 1 )

6
6) Să calculăm mai întâi suma S1  1  p11  p12  p13  ...  p1k1
Avem p1  S1  p1  p12  p13  p14  ...  p1k1 1 şi prin scăderea. membru cu membru, a celor
două egalităţi obţinem :

p1  S1  S1   p1  p12  p13  p14  ...  p1k1 1    1  p1  p12  p13  ...  p1k1   p1k1 1  1
p1k1 1  1
adică S1 ( p1  1)  p k1 1
 1 , de unde obţinem S1 
1
p1  1

7) Dacă scriem produsul pentru cele n sume, obţinem ;

p1k1 1  1 p2k2 1  1 p kn 1  1
S ( (N ))    ...  n
p1  1 p2  1 pn  1

Exerciţii rezolvate :
2.1. Fie S suma divizorilor naturali ai numărului 2001.
Să se arate că 5 x S este număr natural pătrat perfect.

Soluţie.
Avem : 2001=3 x 23 x 29 şi atunci suma divizorilor numărului 2001 este:
p1k1 1  1 p2k2 1  1 p3k3 1  1 311  1 2311  1 2911  1
S ( (2007))        4  24  30
p1  1 p2  1 p3  1 3  1 23  1 29  1
Atunci obţinem :
5  S ( (2007))  5  4  24  30   23  3  5   120 2
2

2.2. Să se arate că pătratul produsului tuturor divizorilor naturali


ai numărului 2001 este 20018.
Soluţie.
Avem următoarea
Lema . Dacă d1 ,d2 ,...,dk sunt toţi divizorii naturali ai numărului n. atunci
avem relaţia:

 d1  d 2  ...  d k 
2
 nk
unde d1 = 1 şi dk = n ( am considerat pe 1 şi n printre divizorii lui n ).
Fie d1 < d2 < d3 < … < dk şi atunci avem :
n n n n
d1  ; d2  ; d3  ;....; d k  (1)
dk d k 1 d k 2 d1
Din relaţia ( 1 ), prin înmulţire membru cu membru, avem :

 d1  d 2  ...  d k 
2
 nk
În cazul nostru 2001 = 3 x 23 x 29 , numărul divizorilor lui 2001 este:
(1+1)(1+1)(1+1)=8. Pentru cei opt divizori naturali ai numărului 2001
avem relaţia
 d1  d2  ...  d8 
2
 20018

2.3. Aflaţi toate numerele naturale care se divid cu 42 şi au 42 divizori .

Soluţie.
Deoarece 42 = 2 x 3 x 7 rezultă că numerele sunt de forma 2 x  3 y  7 z  k , unde x ≥ 1, y ≥ 1, z ≥ 1,
iar factorul k nu are divizori pe 2, 3, sau 7 .
Fie atunci k  p1m1  p2m2  p3m3  ...  prmr .
Atunci numărul divizorilor lui k va fi ( m1 + 1)( m2 + 1 )( m3 + 1 ) … ( mr + 1 ) iar numărul divizorilor lui
7
2 x  3 y  7 z  k va fi ( x + 1 )( y + 1 )( z + 1 )( m1 + 1)( m2 + 1 )( m3 + 1 ) … ( mr + 1 ).

Rezultă că trebuie să găsim x ≥ 1, y ≥ 1, z ≥ 1 astfel încât să avem

( x + 1 )( y + 1 )( z + 1 )( m1 + 1)( m2 + 1 )( m3 + 1 ) … ( mr + 1 ). = 210 = 2 x 3 x 7 (1)

Din ( 1 ) rezultă )( m1 + 1)( m2 + 1 )( m3 + 1 ) … ( mr + 1 ), deci vom avea k = 1 şi atunci avem :

{ x + 1, y + 1 , z + 1 } = { 2, 3, 7 } de unde rezultă

( x, y, z, ) Є { ( 1,2,6 ), ( 1,6,2), ( 2,1,6), (2,6,1),(6,1,2),(6,2,1) }


Obţinem astfel numerele :

21 x 32 x 76 ; 21 x 36 x 72 ; 22 x 31 x 76 ; 22 x 36 x 71 ; 26 x 31 x 72 ; 26 x 32 x 21

2.4. Determinaţi toate numerele de patru cifre, descompuse în factori primi,


care au suma bazelor egală cu suma exponenţilor.

Soluţie.
Fie N  p1k1  p2k2  p3k3  ...  pnkn . Atunci avem relaţia : p1 + p2 + p3 + … + pn = k1 + k2 + k3 + … + kn .
a) Deoarece 2 + 3+ 5+ 7 17, atunci dacă N ar avea cel puţin patru factori în descompunerea sa atunci ar fi cel puţin de
forma 214 x 3 x 5 x 7 ( vezi 14 + 1 + 1 + 1 = 17 ). Dar 214 x 3 x 5 x 7 > 9999 , deci situaţia aceasta nu este posibilă .
b) Dacă N ar avea trei factori atunci ar putea fi cel puţin de forma 210 x 3 x 7 ( vezi 2 + 3 + 7 = 10 + 1 + 1 = 12 ).
Dar 210 x 3 x 7 > 9999 şi atunci vom căuta numere cu factorii primi 2, 3 şi 5 şi suma exponenţilor 10.
Obţinem astfel :

28 x 3 x 5 = 3840 ; 27 x 32 x 5 = 5760 ; 26 x 33 x 5 = 8640 ; 27 x 3 x 52 = 9600


c) Dacă avem doi factori primi, atunci mai mari ca 5 atunci numărul este cel puţin de forma
511 x 7 ( vezi 5 + 7 = 11 + 1 = 12 ). Dar 511 x 7 > 9999 şi atunci rezultă că un factor este 2 sau 3.
Astfel obţinem :

24 x 53 = 2000 ; 23 x 54 = 5000 ; 28 x 7 = 1792 ; 27 x 72 = 6722


d) Dacă avem un singur factor obţinem
55 = 3125 deoarece 77 > 9999 .

2.5. Să se găsească un număr de cinci cifre ştiind că se divide cu 3 şi cu 11 şi


că numărul divizorilor săi este 63.

Soluţie.
Numărul divizorilor fiind impar, rezultă că exponenţii factorilor numărului căutat trebuie să fie pari, deci numărul se divide cel
puţin prin 32 x 112 = 1089.
Împărţind numărul de cinci cifre la 1089 rezultă un cât ce are două cifre.
Cum 63 ( numărul divizorilor ) se divide cu 7 , rezultă că numărul de cinci cifre conţine puterea a şasea a unui factor, care nu
poate fi decât 2.
Rezultă numărul 26 x 32 x 112 = 69696

2.6. Să se determine numerele naturale care au exact patru divizori, iar


produsul acestor divizori să fie 3025.

Soluţie.
Numerele care au exact patru divizori n = p3 sau n = p x q cu p şi q prime diferite .
1) dacă n = p3, atunci n are divizori pe 1, p, p2 şi p3 iar produsul lor va fi 1 x p x p2 x p3 = p6 = n2
2) dacă n = p x q , atunci divizorii lui n sunt 1, p, q, p x q , deci produsul lor va fi 1 x p x q x p x q = ( pq )2.
Rezultă că vom căuta numerele n pentru care n2 = 3025 = 552.
Rezultă n = 55 = 5 x 11

8
2.7. Determinaţi cel mai mic număr natural care are exact 12 divizori.

Soluţie.
Numărul căutat poate fi de formele :
1) n = p11 deoarece, în acest caz, divizorii săi sunt 1, p, p2, p4, … , p11, adică, în total 12 divizori cu p număr prim ;
2) n = p2 x q3 deoarece, în acest caz, divizorii săi sunt 1, p, q, p2, q2, pq, p2q, q3, pq2, p2q2, pq3, p2q3 adică un număr total de
12 divizori cu p şi q numere prime diferite .
3) n = p x q x t2 deoarece, în acest caz, divizorii săi sunt 1, p, q, t, pq, pt, qt, t2, pt2, qt2, pqt, pqt2, adică, în total,
un număr de 12 divizori.
Atunci rezultă :
1) cel mai mic număr natural de forma n = p11 este n = 211 = 2048
2) cel mai mic număr natural de forma n = p2 x q3 este n = 22 x 33 = 106
3) cel mai mic număr natural de forma n = p x q x t2 este n = 5 x 3 x 22 = 60

2.8. În câte moduri pot fi alipite patru triunghiuri echilaterale astfel încât să aibă laturi
comune ?

Soluţie.
Notăm cu T1, T2, T3 şi T4 cele patru triunghiuri echilaterale. Două dintre ele nu pot avea două laturi comune căci atunci ele ar
coincide; deci rezultă că vor avea cel mul o latură comună
Fixăm un triunghi, de exemplu pe T1. Atunci rezultă următorul mod de alipire

T1 T1
T2 T2 T3
T3 T3
T1 T4 T4
T3 T4

2.9. Câte cifre are numărul 168 x 2513 ? Dar numărul 2100 ?

Soluţie.
Numărul 168 x 2513 se scrie 168 x 2513 = ( 42 )8 x 2513 = 416 x 2513 = ( 413 x 2513 ) x 43 =

= 43 x ( 4 x 25 )13 = 64 x 10013 = 64 x ( 102 )13 = 64 x 1026 , deci numărul dat are 26 de zerouri după cifrele 64 ,
adică acesta are 2 + 26 = 28 de cifre
Numărul 2100 va fi comparat, pentru a i se afla numărul de cifre cu un număr care este apropiat ca valoare de
el dar se termină tot cu un număr de zerouri.
Astfel avem :
210 = 1024 < 1025 = 1000 + 25 = 103 + 25 , deci rezultă :

103 < 210 < 103 + 25 (1)


Înmulţim inegalitatea ( 1 ) cu 4 = 22 şi rezultă :

4 x 103 < 212 < 4 x 103 + 100  22 x 103 < 212 < 102 ( 22 x 10 + 1) 

22 x 103 < 212 < 102 x 41 (2)

Ridicăm relaţia ( 2 ) la pătrat şi obţinem : 24 x 106 < 224 < 104 x 412 

24 x 106 < 224 < 104 x 1681 < 104 x 1681 (3)
Din relaţia ( 3 ) obţinem :

24 x 106 < 224 < 104 x 17x 102 (4)

Înmulţim cu 2 ultima inegalitate şi rezultă :

25 x 106 < 225 < 106 x 2 x 17 

25 x 106 < 225 < 34 x 106 (5)


9
Inegalitatea ( 5 ) o ridicăm la puterea a 4 –a şi obţinem :

220 x 1024 < 2100 < 344 x 1024

Dar mai avem că 220 x 1024 > 106 x 1024  220 x 1024 > 1030
iar 344 < 107 şi atunci obţinem :

1030 < 2100 < 1031 de unde rezultă că 2100 are 31 de cifre.

2.10. Se consideră numerele 1, 2, 3, 4, … , 48, 49,50. Înmulţim aceste numere între ele
două câte două ( considerăm un singur produs din a x b şi b x a).
Câte dintre produsele obţinute sunt multiplii de 3 ?

Soluţie.
În şirul 1, 2, 3, 4, … , 48, 49, 50 sunt în total 16 multiplii de 3, respectiv 3, 6, 9, 12, … , 45, 48
Avem evident produsele :
1 x 2, 1 x 3, 1 x 4, …. , 1 x 50, 2 x 3, 2 x 4, 2 x 5, …. , 2 x 50, …., 48 x49, 48 x 50, 49 x 50,
Produsul dintre un număr natural şi multiplu de 3 este tot un multiplu de 3 şi atunci rezultă :
 în şirul 1, 2, 3, 4, , 50 produsele multiplii de 3 se obţin din înmulţirea cu 3 a numerelor
1, 2, 4, 5, 6, 7, … , 48, 49, 50, deci în total 49 produse
 Prin înmulţirea lui 6 cu celelalte numere, scoate pe 6 x 3 care a intrat la 3 x 6 şi atunci produsele sunt date
din înmulţirea cu 6 a numerelor 1, 2, 4, 5, 7, 8, … , 48, 49, 50, deci în total
sunt ( 49 – 1 ) multiplii de trei ;
 Continuând raţionamentul până la ultimul număr, rezultă că succesiv sunt
( 49 – 2 ), ( 49 – 3 ), … , ( 49 – 16 ) , deci în total sunt

49 + ( 49 – 1 ) + ( 49 – 2 ) + ( 49 – 3 ) + … + ( 49 – 15 ) =

= 16 x 49 – ( 1 + 2 + 3 + … + 15 ) =

15 16
= 16 x 49 -  49 16  8 15  664
2

p
2.11. Să se determine al câtelea termen al şirului de mai jos este q .
1 1 1 1 2 2 2 2 n n n n
, , ,..., , , , ,..., ,...., , , ,...,
1 2 3 n 1 2 3 n 1 2 3 n
Soluţie.
Grupăm fracţiile astfel :
1 1 1 1
Gr. 1: , , ,...,
1 2 3 n
2 2 2 2
Gr. 2 : , , ,...,
1 2 3 n
3 3 3 3
Gr. 3 , , ,...,
1 2 3 n
…………………………………………………..
p p p p p p p p
Gr. p : , , ,..., , , , ,...,
1 2 3 q 1 q q 1 q  2 n
……………………………………..
n n n 1
Gr. n : , , ,..., :
1 2 3 n
Adică :
* în fiecare grupă sunt n termeni ;

10
p
* grupa care îl conţine pe are la numărător pe p ;
q
p
* în faţa grupei care îl conţine pe sunt p – 1 grupe a n termeni fiecare ;
q
p
* în grupa care îl conţine pe acest termen are rangul q .
q
Rezultă atunci :
* în faţa grupe p sunt ( p – 1 ) x n termeni ;
p
* în grupa p termenul ocupă locul q ;
q
p
* atunci rangul termenului în şirul iniţial este ( p – 1 ) x n + q
q

2.12. Să se determine c.m.m.d.c. al numerelor naturale a şi b care au suma egală cu 86 şi


câtul împărţirii lui a la b este 4 şi restul r.

Soluţie.
Avem a + b = 86 şi a = b x 4 + r cu r < b . Înlocuind avem succesiv :
4b + r + b = 86  5b + r = 86 cu r < b .
Pentru b Є N şi r < b rezultă : b = 17 şi r = 1 , deci a = 4 x 17 + 1 = 69 şi ( a , b ) = ( 17, 69 ) = 1
b = 16 şi r = 6 , deci a = 4 x 16 + 6 = 70 şi ( a , b ) = ( 16, 70 ) = 2
b = 15 şi r = 11 , deci a = 4 x 15 + 11 = 71 şi ( a , b ) = ( 15, 71 ) = 1

2.13. Să se determine numerele naturale a, b şi c astfel încât


a( b + c ) = 2009 , b( c + a ) = 2010 şi c ( a + b ) = 2011.

Soluţie.
Avem : ab + ac = 2009 ; bc + ab = 2010 şi ac + bc = 2011.
Din prima şi a doua egalitate avem : bc + ab – ( ab + ac ) = 2010 – 2009  bc – ac = 1 
 c( b – a ) = 1, deci c = 1 şi b – a = 1 .
Din a doua şi a treia egalitate avem : ac + bc – ( bc + ab ) = 2011 – 2010  ac – ab = 1 
 a( c – b ) = 1 , deci a = 1 şi c – b = 1
Dacă c = 1 şi a = 1, atunci b – a = 1  b – 1 = 1 şi b = 2 iar din c – b = 1 avem 1 – b = 1 , deci b = 0 imposibil
deci nu avem soluţii .
sau
Prin însumarea celor trei rezultate avem :
2( ab + bc + ac ) = 2009 + 2010 + 2011 = 3 x 2010 
ab + bc + ca = 3015 ( 1 )
Din ( 1 ) şi ab + ac = 2009 avem : bc = 3015 – 2009 = 1006 = 2 x 503 , deci obţinem :
b = 2 şi c = 503 sau
b = 503 şi b = 2 .
Din ( 1 ) şi bc + ab = 2010 avem : ac = 3015 – 2010 = 1005 = 3 x 5 x 67 , deci obţinem :
a = 3 şi c = 335 sau
a = 5 şi c = 201 sau
a = 15 şi c = 67 sau
a = 67 şi c = 15 sau
a = 201 şi c = 5 sau
a = 335 şi c = 3
Din ( 1 ) şi ac + bc = 2011 avem : ab = 3015 – 2011 = 1004 = 22 x 251 , deci obţinem :
a = 2 şi b = 502 sau
a = 4 şi b = 251 sau
a = 251 şi b = 4 sau
a = 502 şi c = 2 sau
Analizând cele trei seturi de rezultate observăm că nu avem nici o soluţie comună şi atunci rezultă că problema
nu are soluţii.

2.14. Să se afle un număr de patru cifre care împărţit la răsturnatul său dă


11
câtul 7 şi restul 618 .

Soluţie.
Avem abcd = 7 x dcba + 618 . Din relaţia dată avem a, d Є { 1, 2, 3, … , 9 } şi b, c Є { 0, 1, 2, 3, … , 9 } .

Dacă d ≥ 2 atunci 7 x dcba are mai mult de patru cifre.


Atunci rezultă d = 1 de unde obţinem :
abc1 = 7 x 1cba + 618

În aceste condiţii, din abc1 = 7 x 1cba + 618 rezultă a = 7 sau a = 8 sau a = 9 .

1) dacă a = 7 avem 7bc1 = 7 x 1cb7 + 618  7000 + 100b + 10c + 1 = 7000 + 700c + 70b + 49 + 618 
 30b = 690c + 667 care nu dă soluţie pentru b, c Є { 0, 1, 2, 3, … , 9 }

2) dacă a = 8 avem 8bc1 = 7 x 1cb8 + 618  8000 + 100b + 10c + 1 = 7000 + 700c + 70b + 56 + 618 
 1000 + 30b = 690c + 674  326 + 30b = 690c  15b + 163 = 345c 
345c  163 13 345c  163 13
 b  23c  10  b  23c  10  care nu dă soluţie
15 15 15 15
pentru b, c Є { 0, 1, 2, 3, … , 9 }

2) dacă a = 9 avem 9bc1 = 7 x 1cb9 + 618  9000 + 100b + 10c + 1 = 7000 + 700c + 70b + 63 + 618 
 2001 + 30b = 690c + 681  1320 + 30b = 690c  b + 44 = 23c , de unde rezultă :
b = 2 şi c = 2 şi atunci obţinem
abcd = 9221

2.15. Calculaţi câte numere naturale de trei cifre au proprietatea că, prin
împărţirea la 19 şi 17 , dau acelaşi rest 13 .

Soluţie.
Avem N = abc . Rezultă N = 19a + 13 = 17b + 13  19a = 17b = N – 13 = k.
Cum 19 x 17 = 323 atunci N – 13 Є { 323, 646, 969 ) adică N Є { 336, 659, 982 }

2.16. Determinaţi numerele naturale de forma ab care împărţite la a x b


dau restul b .

Soluţie.
Avem 10a + b = a x b x k + b  10a = a x b x k  b x k = 10 , de unde avem b = 2 şi k = 5
sau b = 5 şi k = 2, iar a Є { 1, 2, 3, … , 9 } .

2.17. Se determină câtul şi restul împărţirii numărului 5n + 8 la n + 1, unde n


este număr natural nenul .

Soluţie.
Avem succesiv : 1) pentru n = 1 rezultă 13 = 2 x 6 + 1 , deci c = 6 şi r = 1 ;
2) pentru n = 2 rezultă 18 = 6 x 3 + 0 , deci c = 6 şi r = 0
3) pentru n ≥ 3 rezultă 5n + 8 = 5( n + 1 ) + 3 , deci c = 5 şi r = 3

2.18. Fie N = 1 + 3 + 32 + … + 32009 .


a) Să se afle restul împărţirii lui N la 11.
b) Să se rezolve ecuaţia în a :
(1 + 3 + 32 + … + 32009 ) : (1 + 3 + 32 + … + 32005 ) + 70 x a3 = 2011

Soluţie.
a) Avem 1 + 3 + 32 + … + 32009 = ( 1 + 3 + 32 + 33 + 34 ) + 35( 1 + 3 + 32 + 33 + 34 ) + …. +
+ 32005( 1 + 3 + 32 + 33 + 34 ) = 121( 1 + 35 + … + 32005 ) = M 11

12
b) Avem 121( 1 + 35 + … + 32005 ) : ( 1 + 35 + … + 32005 ) + 70 x a3 = 2011
121 + 70 x a3 = 2011 ; 70a3 = 1890 ; a3 = 27 deci a = 2

2.19. Fie n Є N* şi S suma resturilor obţinute prin împărţirea numerelor


1, 2, 3, … , 100 la n .
a) Să se calculeze S în cazul n = 5 .
b) Să se determine valoarea lui n pentru care S = 100.

Soluţie.
a) Resturile împărţirii la 5 sunt { 1, 2, 3, 4, 0 }.. Atunci grupăm numerele astfel :
( 1 , 2, 3, 4, 5 ), ( 6, 7, 8, 9, 10 ), …. , ( 96, 87, 98, 99, 100 )
În total 20 de grupe
Sumele resturilor în fiecare grupe sunt egale cu 1 + 2 + 3 + 4 + 0 = 10 şi atunci S = 20 x 10 = 200

c) Pentru n = 2 suma resturilor la un grup de două numere consecutive este 0 + 1 = 1.În total sunt 100 : 2 = 50
grupe, deci S = 50 x 1 = 50 < 100 ;
Pentru n = 3 suma resturilor la un grup de 3 numere consecutive este 0 + 1 + 2 = 3 . În total sunt 33 de grupe ,
plus numărul 100 care da restul 1. Atunci S = 33 x 3 + 1 = 100.
Pentru n ≥ 4 rezultă S > 100, deci soluţia este n = 3.

3. Principiul lui Dirichlet ( principiul cutiei )


Matematicianul german Peter Gustav Dirichlet (1805-1859) a elaborat
un principiu extrem de simplu cu aplicaţii neaşteptate în variate domenii,
principiu care-i poartă numele şi care este o metodă de demonstraţie
de tipul următor.
"Dacă repartizăm n + 1 obiecte în n cutii, atunci cel puţin două
obiecte vor fi în aceeaşi cutie."

Justificare:
Considerăm cazul cel mai nefavorabil aşezând în fiecare cutie câte un obiect.
Deci am folosit n cutii şi n obiecte. Obiectul cu numărul n + 1 trebuie pus şi el într-o cutie oarecare.
Dar în acea cutie există deja un obiect. Aşadar în acea cutie există deja un obiect pus anterior.
În acea cutie vor fi două obiecte.
Noţiunea de CUTIE precizează nume de obiecte iar obiectele desemnează numere, lucruri,
figuri geometrice etc.

Forma generală a principiului lui Dirichlet este următoarea:

"Dacă aşezăm kn + 1 obiecte în n cutii, atunci cel puţin k + 1 obiecte,


k Є N, vor fi în aceeaşi cutie."

Exerciţii rezolvate :
3.1. La un turneu de şah au participat n ≥ 2 şahişti. Să se demonstreze că
în orice moment al turneului dinaintea ultimei runde cel puţin doi şahişti
au acelaşi număr de victorii.
Soluţie.
În orice moment al turneului dinaintea ultimei runde, fiecare şahist a jucat maximum n - 2 partide şi a putut
obţine 0,1,2,...,n - 2 victorii, deci în total n - 1 posibilităţi (cutii).
Deoarece la turneu au participat n şahişti rezultă că cel puţin doi şahişti au acelaşi număr de victorii înaintea
13
ultimei runde.

3.2. Arătaţi că în orice mulţime formată din 5 numere naturale există două
a căror diferenţă este divizibilă cu 4.

Soluţie.
La împărţirea unui număr cu 4 obţinem unul din resturile 0,1,2,3 (deci patru cutii).
Deoarece avem 5 numere (5 obiecte şi 4 cutii) rezultă că cel puţin două numere vor da acelaşi rest la
împărţirea cu 4.
Ele sunt de forma x = 4k + r şi y = 4p + r .
Atunci diferenţa lor este x - y = 4(k - pl) , adică un număr divizibil cu 4.

3.3. Într-o şcoală sunt 367 elevi. Să se demonstreze că există cel puţin doi
elevi care îşi serbează ziua în aceeaşi zi a anului.

Soluţie.
Un an are 365 sau 366 zile. Considerând cazul cel mai nefavorabil când în fiecare zi a anului ar fi născut
câte un elev, înseamnă că în total ar fi născuţi 365 sau 366 elevi, dar în total sunt 367 elevi.
Deci al 367-lea elev a fost şi el născut într-o zi a anului în care a mai fost născut un elev.
Deci într-o zi s-au născut 2 elevi, deci cei doi îşi vor serba ziua de naştere în aceeaşi zi.

3.4. Fiind date n + 1 numere naturale (n ≠ 0) atunci cel puţin două dintre
ele dau acelaşi rest la împărţirea cu n.

Soluţie.
Folosim teorema împărţirii cu rest. Fiind date numerele naturale a şi b (b ≠ 0) există în mod unic numerele
naturale q şi r astfel ca
a = bq + r cu r < b .
În cazul problemei noastre numerele fiind împărţite la n există pentru rest n valori posibile: 0,1,2,...,n-1.
Deoarece împărţim n+1 numere vor exista n+1 resturi, dintre care cel mult n sunt diferite.
Rezultă că cel puţin două dintre cele n+1 numere împărţite la n dau acelaşi rest.

3.5. Să se arate că oricum am alege 7 numere pătrate perfecte (distincte)


exist cel puţin două a căror diferenţă se divide cu 10.

Soluţie.
Dacă a este numărul a cărui pătrat este a2 atunci la împărţirea cu 10 a lui a obţinem unul din resturile: 0,1,2,3,4,5,6,7,8,9.
Atunci a2 , la împărţirea cu 10 va da unul din resturile: 0,1,4,5,6,9.
Deoarece avem 7 pătrate perfecte şi numai resturile 0,1,4,5,6,9, atunci există cel puţin două pătrate perfecte
care dau acelaşi rest la împărţirea cu 10, deci diferenţă a lor se divide cu 10.

3.6. Din cei 24 de elevi ai unei clase 19 au participat la olimpiada de limba română, 16 la
olimpiada de matematică, a5 la olimpiada de geografie şi 14 la olimpiada de fizică . Să
se arate că cel puţin 8 elevi au participat la toate cele patru olimpiade.

Soluţie.
Presupunem că nici un elev nu a participat la patru olimpiade, deci cei 24 de elevi au participat fiecare la cel
puţin trei olimpiade. Dacă au participat la câte trei olimpiade atunci în total la câte trei olimpiade au
participat 24 x 3 = 72 de elevi.
Dar numărul total de elevi participanţi la cele patru olimpiade este 19 + 16 + 15 + 14 = 64 , deci 72 – 64 = 8
elevi sunt în plus, adică cel puţin 8 elevi au participat la toate cele patru olimpiade

14
3.7. Să se arate că printre oricare 52 de numere naturale, există cel puţin două care au
suma sau diferenţa divizibilă cu 100 .

Soluţie.
Fie o mulţimea formată din primele 52 de numere naturale . Rezultă că există cel puţin două numere care au
suma divizibilă cu 100, respectiv 52 + 48 = 100 şi 51 + 49 = 100.
Fie o altă mulţime A formată din 52 de numere naturale, diferită de prima. Notăm cu u ( x ) ultimele două cifre
cu care se termină elementul x din A. Împărţim această mulţime în următoarele submulţimi :

A1 = { x Є A | u( x ) = 00 } ; A2 = { x Є A | u( x ) = 01 sau u ( x ) = 99 } ; A3 = { x Є A | u( x ) = 02 sau u ( x ) = 98 } ;
……………… ; A50 = { x Є A | u( x ) = 49 sau u ( x ) = 51 } ; A51 = { x Є A | u( x ) = 50 } .
Deoarece mulţimea A are 52 de elemente, conform principiului lui Dirichlet cel puţin două se vor găsi în aceeaşi
submulţime Ak.
Dar în fiecare submulţime Ak suma sau diferenţa a două elemente se divide cu 100 , deci cel puţin două elemente
au suma sau diferenţa divizibilă cu 100.

3.8. Să se arate că oricare ar fi 7 numere pătrate perfecte, există două a căror diferenţă
se divide cu 10.

Soluţie.
Pătratele perfecte se termină cu una din cifrele : 0, 1, 4, 5, 6, 9 , deci sunt în total 6 terminaţii.
Deoarece sunt 7 pătrate rezultă că două dintre ele vor avea aceiaşi terminaţie.
Aşadar diferenţa celor două pătrate cu aceiaşi terminaţie se termina în 0 şi atunci ea se divide cu 10.

3.9. Să se arate că oricare trei numere naturale prime a, b, c cu a > b > c ≥ 5, atunci N =
( a2 – b2 )( p2 – c2 )( b2 – c2 ) se divide cu 30

Soluţie.
Deoarece a, b, c sunt numere prime diferite de 2 rezultă că ( a – b )( a + b ) se divide cu 4 , deci
a2 – b2 se divide cu 2.
Deoarece a, b, c sunt numere prime diferite de 3 rezultă că acestea sunt de forma 3k + 1 sau 3k + 2.
Cum cel puţin două din ele au aceiaşi formă fie a = 3p + 1 şi b = 3q + 1. Atunci obţinem :
a2 – b2 = ( a – b ) ( a + b ) = 3( p – q )( 3p + 3q + 2 ) , deci a2 – b2 se divide cu 3.
Dacă două dintre cele trei numere se divid cu 5, atunci diferenţa pătratelor lor se divide cu 5.
Dacă numerele a, b, c nu se divid cu 5, atunci conform teoremei împărţirii cu rest acestea dau
resturile respectiv 1, 2, 3 sau 1, 2, 4 sau 2, 3, 4. În fiecare caz există două numere a căror sumă se divide
cu 5 ( vezi suma resturilor 2 + 3 = 5, respectiv 1 + 4 = 5 ). Deci N se divide cu 5 .
Rezultă aşadar că N se divide cu 2 x 3 x 5 = 30

3.10. Să se arate că pentru oricare număr natural impar n există un număr


k Є N* astfel încât ( 2k – 1 ) să fie divizibil cu n.

Soluţie.
Considerăm numerele 21 – 1 ; 22 – 1 ; 23 – 1; ….; 2n – 1 ; 2n + 1 – 1 şi fie resturile împărţirii acestor numere la n respectiv r1; r2; r3;
… , rn; rn + 1.
Dar prin împărţirea la n se obţin n – 1 resturi , deci printre cele n + 1 resturi există cel puţin două resturi egale .
Rezultă atunci că diferenţa lor se divide cu n .
Fie a = 2p – 1 şi b = 2q – 1 cele două numere care dau acelaşi rest la împărţirea cu n.
Avem :
( 2p – 1 ) – ( 2q – 1 ) = 2p - 2q = 2q( 2p – q – 1 ) = n x t
Dar 2 şi n sunt relativ prime şi atunci rezultă că 2p – q – 1 = 2k – 1 se divide cu n, unde k = p – q.
q

3.11. Într-o cutie se află 36 de bile numerotate de la 1 la 36. Ion încearcă să elimine
bilele din cutie , în etape. Fiecare etapă constă în următoarea succesiune de
operaţii :
 Ion extrage la întâmplare patru bile din urna ;
 Ion elimină două bile din cele patru dacă diferenţa numerelor înscrise pe acestea este divizibilă cu 3 ;
15
 Ion reintroduce în urnă bilele care nu au fost eliminate.
a) Arătaţi, că la fiecare etapă, Ion poate elimina cel puţin două bile ;
b) Arătaţi că, dacă în cutie rămân numai patru bile , atunci Ion le poate
elimina pe toate .
Soluţie
a) Avem d = 3 x c + r unde r < 3 , adică orice bilă este numerotată cu un număr egal cu unul din variante :
3k ; 3k + 1 ; 3k + 2 .
. Dacă Ion scoate patru bile atunci rezultă că printre acestrea vor avea notatiile de fforma 3k şi 3p
sau 3k + 1 şi 3p + 1 sau 3k + 2 şi 3p + 2. Atunci rezultă că la fiecare extragere diferenta a două
bile va fi multiplu de 3, adică ea va fi de forma :
3k – 3p = 3( k – p ) sau
3k + 1 – ( 3p + 1 ) = 3( k – p ) sau
3k + 2 – ( 3p + 2 ) = 3( k – p )
Deci la fiecare extragere Ion va elimina cel puţin două bile .

b) Eliminând continuu câte două bile rerzultă că în urna vor rămâne permanent un număr par de
bile ( 36 – 2 = 34; 34 – 2 = 32 etc ).
Dacă bilele sunt numerotate de la 1 la 36 rezultă că în total, în urnă, avem :
:
 12 bile de form 3k ; 12 bile de form 3k + 1 şi 12 bile de forma 3k + 2
După 16 operaţiuni Ion elimină 32 de bile, când în urnă au rămas doar 4 bile, rezultă ;
 în urna au ramas 4 bile de forma 3k
 în urna au ramas 4 bile de forma 3k + 1
 în urna au ramas 4 bile de forma 3k + 2
 în urna au ramas 2 bile de forma 3k şi 2 bile de forma 3k + 1
 în urna au ramas 2 bile de forma 3k şi 2 bile de forma 3k + 2
 în urna au ramas 2 bile de forma 3k + 1 şi 2 bile de forma 3k + 2
Deci în final, cele patru bile pot fi eliminate deoarece sunt două situaţii în care diferenţa a două
numere de pe două bile este multiplu de 3 .

4. Principiul invariantului
Invariantul este o mărime, o relaţie, sau o proprietate care rămâne
neschimbată în urma aplicării sau intervenţiei unei transformări.
Deci o situaţie iniţială este supusă în mod repetat unor transformări.
De obicei se cere s se demonstreze că în urma acestor transformări
nu se poate ajunge la o anumită formă . Aceasta se poate face alegând
caracteristica obiectului care a fost supus transformării, adică "invariantul"
transformării. Dacă, în final, obiectul nu posedă "invariantul" atunci el nu
poate fi obţinut în urma transformărilor descrise.

Exerciţii rezolvate :
4.1. Consider m un număr natural căruia îi schimbăm în mod arbitrar
ordinea cifrelor. Este posibil ca diferenţa dintre numărul iniţial şi cel final
să se împartă fără rest la 9 ?

Soluţie.
Restul împărţirii numărului la 9 este acelaşi cu restul împărţirii sumei cifrelor sale la 9.
Suma cifrelor este aceeaşi, rezultă că restul împărţirii numărului la 9 este un invariant.

16
4.2. Pe o tablă sunt scrise semne de "+" şi " - ". Ştergem două semne şi le
înlocuim cu un semn, după următoarea regulă : dacă cele două semne
şterse sunt identice le înlocuim cu "+", iar dacă ştergem două semne
diferite le înlocuim cu " - ". Arătaţi că ultimul semn care rămâne după
un număr de paşi nu depinde de ordinea alegerii perechilor.

Soluţie.
În acest caz paritatea numărului de minusuri va fi invariantul. Dacă la început numărul de minusuri este impar,
ultimul semn care va rămâne este minus, iar dacă la început numărul de minusuri este par, la sfârşit
va rămâne plus.

4.3. Trei greieri se găsesc pe o dreaptă în ordinea: A, B, C.


Ei încep să sară capra, adică să sară unul peste altul (dar nu peste
doi odată ). Pot fi în aceeaşi ordine după 2003 sărituri?

Soluţie.
În urma unei sărituri de acest fel numărul perechilor de greieri inversaţi creşte sau se micşorează
cu 1 (proprietatea invariant ). Dup un număr impar de sărituri (2003) va exista un număr impar de
perechi de greieri inversaţi.
Deci nu se poate obţine ordinea iniţială (ce nu conţine o astfel de pereche).

4.4. O cameră are dimensiunile podelei de 7m şi 10m. În cele patru colţuri


ale camerei se aşează câte un dulap având baza pătrat cu latura de 1m.
Să se arate ce rămâne din suprafaţa podelei nu poate fi acoperită cu
plăci dreptunghiulare de dimensiuni 3m /1m.

Soluţie.
Se împarte camera într-o reţea de pătrate cu latura 1m pe care le vopsim în trei culori: roşu, alb, negru ca mai jos:
RANRANRANR
ANRANRANRA
NRANRANRAN
RANRANRANR
ANRANRANRA
NRANRANRAN
RANRANRANR
Obţinem 24 de R, 23 de A, 23 de N. Eliminând colţurile rămân 20 de pătrăţele roşii, 23 de pătrăţele albe, 23 de pătrăţele negre.
Dar oricum am aşeza o placă de 3 / 1 m ea acoperă un pătrăţel roşu, unul alb şi unul negru.
Dacă s-ar putea acoperi suprafaţa cu un număr întreg de plăci ar trebui să existe acelaşi număr de pătrăţele
pentru fiecare culoare.

5. Probleme de logică
Iată câteva probleme de logică a căror rezolvare se realizează printr-o serie de
17
judecăţi logice ce solicit inventivitate, perspicacitate, etc. şi foarte puţin calcul.

Exerciţii rezolvate :

5.1. Mama a observat că din dulap au dispărut cinci tablete de ciocolată .


Ele puteau fi luate de cei trei copii: A, B, C. Fiind traşi la răspundere,
ei au dat mai întâi următoarele răspunsuri:
A: N-am luat nici o ciocolată !
B: N-am luat nici o ciocolată !
C: N-am luat nici o ciocolată !
După un nou "interogatoriu" copiii au făcut următoarele declaraţii:
A: B a luat mai multe tablete decât C!
B: (către A): Minţi!
C: Toate au fost luate de A şi B!
A (către C): Minţi!
Aflaţi câte tablete de ciocolată au fost luate de către fiecare copil, ştiind
că fiecare a făcut atâtea declaraţii false câte tablete de ciocolată a luat.

Soluţie.
Fiindcă au "dispărut" 5 ciocolate i s-au făcut 7 declaraţii, rezult că cinci declara ii erau false iar 2 (7-5) adevărate.
La al doilea "interogatoriu" prima afirmaţie a lui A este fie adevărat şi atunci afirma ia lui B este falsă , fie este
falsă şi atunci afirmaţia lui B este adevărat .
Tot din "interogatoriul" al doilea afirmaţia lui C este fie adevărată şi atunci cea de-a doua afirmaţie a lui A este falsă , fie este
falsă şi atunci cea de-a doua afirmaţie a lui A este adevărat .
Deci rezult că cele două afirmaţii adevărate au fost făcute la cel de-al doilea "interogatoriu", deci la primul "interogatoriu" toţi
copiii au făcut declaraţii false, de unde rezultă că fiecare din cei trei copii a luat cel puţin o
ciocolată .
Deci a doua afirmaţie a lui C este falsă , deci C a luat două tablete de ciocolată . B face numai două afirmaţii, deci el nu poate lua
mai multe ciocolate decât C, rezultă că la al doilea "interogatoriu" prima afirmaţie a lui A este falsă , deci afirmaţia lui B este
adevărat .
Deci A a luat două ciocolate, B a luat o ciocolată , iar C a luat două ciocolate.

5.2. Într-un bloc locuiesc familiile A, B, C, D, E, F, G, H, I, K, L, M, N, O,


P, R. La parter şi la fiecare etaj locuiesc câte două familii. Se mai ştie că :
Familia A locuieşte cu două etaje mai jos ca familia B, iar aceasta cu şase
etaje mai sus ca familia C. Familiile F şi G locuiesc la acelaşi etaj.
Familia M locuieşte cu patru etaje mai sus ca familia N şi cu două etaje
mai jos ca familia F.
Un etaj deasupra familiei N locuieşte familia O. Familia A locuieşte cu
trei etaje mai sus ca familia R, iar familia P locuieşte cu cinci etaje mai
jos decât familia G.
a) Câte etaje are blocul?
b) La ce etaj locuieşte familia A?

Soluţie.
a) Deoarece la parter şi la fiecare etaj locuiesc două familii, iar în tot blocul locuiesc 16 familii, rezult că blocul are
opt nivele (parter şi apte etaje).
b) Diagramele alăturate (stabilite conform enunţului) pun în evidenţă modul cum sunt distribuite în bloc familiile
N, O, M, F, G, P precum i C, R, A, B. Blocul având opt nivele, rezultă că familia N poate locui numai la parter
sau la etajul întâi.
Aceeaşi remarcă şi pentru familia C. Familiile N şi C nu pot locui la acelaşi nivel, pentru că ar trebui ca familiile
18
O, P, R să locuiască la acelaşi etaj, situaţie imposibil , pentru că la un nivel pot locui numai două familii.
Dacă familia N locuieşte la parter atunci familia C ar trebui să locuiască la etajul întâi, situaţie imposibil
deoarece ar rezulta că familiile O, P, C locuiesc la acelaşi etaj. Dacă familia N locuieşte la etajul întâi,
atunci familia C locuieşte la parter. Urmărind comparativ diagramele observăm că este o situaţie posibil pentru
că la un nivel pot locui numai două familii.
Acestea fiind precizate putem stabili distribuţiile familiilor în bloc: Familiile F şi G la etajul şapte, familia B la
etajul şase, familia M la etajul cinci, familia A la etajul patru, familiile P i O la etajul doi, familiile N i R la etajul întâi,
iar familia C la parter. În cele şase locuri neocupate se vor distribui familiile D, E, H, I, K, L după voie.

5.3. La un turneu de fotbal particip 15 echipe, fiecare dintre acestea


jucând cu toate celelalte. Pentru victorie se acordă 3 puncte, pentru meci
egal 2 puncte, iar pentru înfrângere un punct. În clasamentul întocmit la
sfârşitul turneului nu există echipe cu acelaşi număr de puncte. ştiind că
ultima echipă are 21 de puncte, să se arate că prima a făcut cel puţin un
meci nul.

Soluţie.
14 15
Fiecare echipăa disputat 14 meciuri. Numărul meciurilor disputate a fost  105 deoarece fiecare
2
meci a fost numărat de două ori ( şi când a jucat A cu B şi când a jucat B cu A). Deoarece echipa clasată pe
ultimul loc are 21 de puncte, iar în clasament nu sunt echipe cu acelaşi număr de puncte rezultă că numărul
de puncte este mai mare sau egal cu
14 15
21 + (21 + 1) + (21 + 2) + ... +(21 + 14) = 2115   420
2
Deoarece la fiecare meci s-au acordat 4 puncte, rezultă că numărul de puncte
acordat a fost 105  4  420 .
Deci echipele au obţinut punctajele: 21, 22,23,...,34, 35.
Să arătăm că echipa de pe locul întâi cu 35 puncte a făcut cu siguranţă cel puţin un meci nul.
Presupunem că nu a făcut nici un meci nul. Atunci dacă x este numărul victoriilor şi y numărul înfrângerilor avem:

x + y = 14 şi 3x + y = 35,
21 7
de unde x şi y  .
2 2
Dar x, y trebuie să fie naturale.
Deci presupunerea făcută este falsă , atunci echipa de pe locul întâi a făcut cel puţin un meci nul.

5.4. La un concurs de atletism particip trei echipe: A1 , A2 , A3 , fiecare cu


trei concurenţi. Concurentul care soseşte primul primeşte 18 puncte,
cel care soseşte al doilea 16 puncte, cel care soseşte al treilea 14 puncte
,..., cel care soseşte ultimul primeşte două puncte. Punctajul unei echipe
este suma punctelor obţinute de cei trei reprezentanţi ai săi.
Aflaţi ce loc a ocupat fiecare echipă ştiind că :
i) Primele trei locuri au fost ocupate de concurenţi de la echipe diferite.
ii) Fiecare concurent de la echipa A2 avea în faţa sa un concurent de
la echipa A1 .
iii) Concurenţii echipei A3 au sosit unul după altul.

Soluţie.
Din prima şi a treia condiţie rezultă că cei trei reprezentanţi ai echipei A3 au sosit al treilea, al patrulea şi
al cincilea. Din primele două condiţii rezultă că primul a sosit un concurent de la echipa A1 , iar al doilea
un concurent de la echipa A2 .
Din cele de mai sus şi din a doua condiţie rezultă că al şaselea şi al optulea au sosit concurenţii de la
19
echipa A1 , iar al şaptelea şi al nouălea au fost concurenţii de la echipa A2 . Deci echipa A1 a
acumulat 18+8+4=30 (puncte), echipa A2 a acumulat 16+6+2=24 (puncte), iar echipa A3 a
acumulat 14+12+10=36 (puncte).
Deci pe locul întâi se află echipa A3 , pe locul doi echipa A1 , iar pe locul trei echipa A2 .

5.5. Opt şahişti participă la un turneu, jucând fiecare cu fiecare.


Pentru fiecare victorie un jucător primeşte un punct, pentru remiză un
jumătate de punct, iar pentru înfrângere nu primeşte nici un punct.
La sfârşitul turneului primii doi clasaţi au obţinut punctaje diferite, iar
cel de-al doilea a obţinut atâtea puncte câte au obţinut ultimii patru
şahişti împreună . Să se afle cum s-a încheiat partida dintre şahiştii
clasaţi pe locurile trei şi cinci.

Soluţie.
Deoarece au fost opt jucători şi fiecare a jucat cu fiecare, un şahist a jucat şapte partide şi ar fi putut câştiga
cel mult şapte puncte (când învingea în toate cele apte partide). Ultimii patru şahişti au jucat între ei şase
partide. (Dacă A,B,C,D sunt ultimii şahişti, au jucat: A cu B , A cu D, B cu C , B cu D i C cu D).
Deci ultimii patru şahişti au realizat împreună cel puţin şase puncte. Deci şahistul de pe locul doi a obţinut
cel puţin şase puncte (deoarece el a obţinut un număr egal de puncte cu suma ultimilor patru).
Deoarece primii doi jucători au punctaje diferite înseamnă că al doilea a obţinut exact şase puncte, căci
dacă obţinea 6,5 puncte primii doi aveau acelaşi punctaj, iar dacă ar fi obţinut şapte puncte era pe primul loc.
Deci şahiştii de pe ultimele patru locuri au obţinut exact şase puncte, aceasta înseamnă că ei au pierdut toate partidele jucate
împotriva primilor patru clasaţi. Deci şahistul de pe locul cinci a pierdut partida susţinut cu
cel de pe locul trei.

5.6. În trei coşuri sunt mere. Câte mere sunt în fiecare coş ştiind că în
primele 2 împreună este un măr, în ultimele 2 împreună este cel
puţin un măr, iar în ultimul şi al treilea împreună , numai unul.

Soluţie.
Dacă mărul din primele 2 coşuri s-ar afla în primul coş , atunci din a treia condiţie ar rezultat că în al treilea
coş nu se află nici un măr, deci al doilea şi al treilea coş ar fi goale şi astfel nu ar avea loc a doua condiţie.
În concluzie primul coş este gol, al doilea coş conţine un măr, iar al treilea coş conţine tot un măr.

5.7. Cineva are o damingeană cu 12 l de vin şi mai are la dispoziţie două


vase : unul de 8 l şi unul de 5 l. Cum procedează să separe 6 l în
vasul de 8 l. Generalizare .

Soluţie.
Avem succesiv rezultatele din tabelul următor :
Vase / Paşi Initial Pas 1 Pas 2 Pas 3 Pas 4 Pas 5 Pas 6
12 l 12 4 4 9 9 1 1
8l 0 8 3 3 0 8 6
5l 0 0 5 0 3 3 5
Total 12 l 12 l 12 l 12 l 12 l 12 l 12 l
Generalizând problema avem :
„ Dacă damingeana are a litri, primul vas b litri şi al treilea vas c l cu a > b > c şi a + c > 2b si se cere să
separăm k litri unde b > k > c , adică k = b – m şi b + m = 2c „

Rezultă tabelul :
Vase / Paşi Initial Pas 1 Pas 2 Pas 3 Pas 4 Pas 5 Pas 6
al a a–b a-b a–b+c a–b+c a–b+c-b a–b+c-b
bl 0 b b-c b-c 0 b b–m=k

20
cl 0 0 c 0 b-c b-c b–c+m=
= 2c – c = c
Total a a a a a a a

6. Probleme de ordonare
Pentru a stabili care dintre două numere a şi b este mai mare, putem folosi mai
multe procedee, dintre care cele mai des întrebuinţate sunt:

1) Stabilim semnul diferenţei a - b .


Dacă a – b > 0 , atunci a > b .
Dacă a – b = 0 , atunci a = b .
Dacă a - b < 0 , atunci a < b .
a
2) Dacă numerele a şi b sunt pozitive şi b ≠ 0, comparăm raportul cu 1 .
b
a
Dacă < 1, atunci a < b
b
a
Dacă = 1, atunci a = b
b
a
Dacă > 1, atunci a > b
b

3) În unele situaţii este suficient să demonstrăm existenţa unui număr ” c ”


situat între cele două numere (exemplu: din a < c < b , rezult a < b ).
În unele situaţii avem nevoie de metode ingenioase pentru a rezolva
problemele. Există cazuri când operaţia de ordonare ajută la dovedirea egalităţii a două
numere x şi y prin stabilirea simultană a inegalităţilor x ≤ y şi y ≤ x .

Exerciţii rezolvate :
6.1. Comparaţi numerele 31 cu 1714 .
11

-
Soluţie.
Avem succesiv

3111 < 3211 = (25 )11 = 255 < 256 = (24 )14 = 1614 < 1714 .

6.2. Scrieţi în ordine crescătoare numerele: 422 ,334 ,244 ,637 .

Soluţie.
Avem succesiv

637 < 647=(26 )7 = 242 < 244 = (22 )22 = 422 < 251 = (23 )17 = 817 < 917 =

= ( 32 )17 = 334

Deci 637 < 244 < 422 < 334 .


21
6.3. Să se arate că pentru orice numere naturale a şi b avem
a  2360 b  51800

a  3240 b  71440
Soluţie.
Avem succesiv
2360 = ( 23 )120 = 8120 ;

3240 = ( 32 )120 = 9120

Atunci obţinem că 2360 < 3240 de unde rezultă că a+ 2360 < a + 3240 şi deci avem

a  2360
1
a  3240
Analog avem succesiv 51800 = ( 55 )360 = 3125360 şi 71440 = ( 74 )360 = 2401360
b  51800
Rezultă astfel : b +5 1800
>b+7 1440
de unde obţinem : 1
b  71440
Deci prima fracţie din ipoteză este subunitară iar a doua este supraunitară şi atunci relaţia cerută
este adevărată .

6.4. Comparaţi fracţiile A şi B unde

2  22  23  ...  21997 3  32  33  ...  31331


A B
1 1 1 1 şi 1 1 1 1
 2  3  ...  1997  2  3  ...  1331
2 2 2 2 3 3 3 3

Soluţie.

Amplificăm prima fracţie cu 21998 şi a doua cu 31332 şi obţinem:

21998 (2  22  23  ...  21997 )


 21998   23   8666
666
A  1997 1996 1995
2  2  2  ...  2  2 2

şi respectiv

31332 (3  32  33  ...  31331 )


B
3  3  3  ...  3  3)
1331 1330 1329 2
 31332
  3 
2 666
 9666

Deci rezultă A>B.

6.5. Comparaţi numerele :


a) 21987 şi 31241 ; b) 31987 şi 41490 c) 41987 şi 51703

22
Soluţie.
a) Avem succesiv : 21987 = 28 x 248 + 3 = ( 28 )248 x 23 = 256248 x 8 > 243248 x 3 = ( 35 )248 x 3 = 31240 x 3 = 31241
b) Avem succesiv : 31987 = 32 x 993 + 1 = ( 32 )993 x 3 = 9993 x 3
Pe de altă parte avem : 41490 = 22980 = ( 23 )993 x 2 = 8993 x 2
Atunci rezultă 9993 x 3 > 8993 x 2, deci 31987 > 41490 .
c) Avem succesiv : 41987 = 4 x 41987 = 41988 = ( 47 )284 = ( 27 )568 = 128568 > 125568 = ( 52 )568 = 51704 =
= 51703 x 5 > 51703 x 4
Atunci avem 41987 > 51703

6.6. Care număr este mai mare : 9991000 + 1000999 sau 999999 + 10001000

Soluţie.
Avem succesiv : 9991000 + 1000999 = 999 x 999999 + 1000999
Dar 999999 + 10001000 = 999999 + 1000 x 1000999 = 999999 + ( 999 + 1 ) x 1000999 =
= 999999 + 999 x 1000999 + 1000999
Avem 999 x 999 + 1000999 < 999999 + 999 x 1000999 + 1000, deci
999

9991000 + 1000999 < 999999 + 10001000

6.7. Fie a, b, c, d numere naturale diferite de zero astfel încât a < c şi d < b. Să se scrie în
a c ac ac ac
ordine crescătoare fracţiile , , , ,
b d bd bd bd

Soluţie.
a ac c a  a a  c c  c 2c c
Avem evident   deoarece    
b bd d b  d b  d d  d 2d d
ac
Deoarece c > a şi b > d rezultă c – a < 0 şi b – d > 0, deci fracţia  0 , adică fiind singura fracţie negativă,
bd
rezultă că este şi cea mai mică .
ac ac

Pe de altă parte b + d > b – d şi atunci rezultă
bd bd
ac a ac ac c ac a ac c ac
Rezultă astfel ordinea :     sau    
bd b bd bd d bd b bd d bd

7 8 9 1983 1984
6.8. Scrieţi în ordine crescătoare următoarele fracţii : , , ,..., ,
10 15 20 9890 9895
Soluţie.
Fiecare fracţie se observă că permite următoarea descompunere :
7 52 1 1
  
10 10 2 5
8 53 1 1
  
15 15 3 5
9 54 1 1
  
20 20 4 5
………………………
1983 5  1978 1 1
  
9890 9890 1978 5
1984 5  1979 1 1
  
9895 9895 1979 5
1
Deoarece este o valoare comună râne să verificăm ceilalţi termeni ai fiecărei sume.
5
23
1 1 1 1 1 7 8 9 1983 1984
Astfel rezultă :    ...   , deci    ...  
2 3 4 1978 1979 10 15 20 9890 9895

19992000  20001999
6.8. Arătaţi că fracţia este subunitară .
19991999  20002000
Soluţie.
Presupunem 19992000 + 20001999 < 19991999 + 20002000  19992000 - 19991999 < 20002000 - 20001999
1999
 
Astfel rezultă : 19991999 1998 < 20001999 1999 
 1999 
  
1999
 2000  1998

7. Metoda reducerii la absurd


Metoda reducerii la absurd este o metodă specifică de demonstraţie în
matematică .
La baza acestei metode stă una din legile fundamentale ale logicii clasice:
„ legea terţului exclus „ , ce are următorul enunţ :

„ Din două propoziţii contradictorii una este adevărată , cealaltă falsă ,


iar a treia posibilitate nu există „.
Legea terţului exclus nu ne precizează care din cele două propoziţii este
adevărat şi care este falsă .
Când la două propoziţii contradictorii aplicăm legea terţului exclus este
suficient să stabilim că una dintre ele este falsă pentru a deduce că cealaltă este
adevărată .
Metoda reducerii la absurd constă în a admite în mod provizoriu, ca adevărată
propoziţia contradictorie propoziţiei de demonstrat, apoi pe baza acestei presupuneri se
deduc o serie de consecinţe care duc la un rezultat absurd, deoarece ele contrazic sau ipoteza
problemei date sau un adevăr stabilit mai înainte.
Mai departe raţionăm astfel:
 dacă presupunerea ar fi fost adevărată , atunci în urma raţionamentelor logic
corecte ar fi trebuit să ajungem la o concluzie adevărată ;
 deoarece am ajuns la o concluzie falsă , înseamnă că presupunerea noastră a
fost falsă .
 Aceasta duce la concluzia că presupunerea făcută nu este posibilă şi rămâne
ca adevărată concluzia propoziţiei date.
Metoda reducerii la absurd nu se reduce la propoziţia că "a demonstra o
propoziţie este acelaşi lucru cu a demonstra contrara reciprocei ei", deoarece
pot apărea şi situaţii în care nu se contrazice ipoteza ci o altă propoziţie
(un rezultat cunoscut, o axiomă, o teoremă).
Metoda reducerii la absurd se foloseşte atât în rezolvarea problemelor de calcul
(de aflat) cât şi la rezolvarea problemelor de "demonstrat".
Metoda este des utilizat în demonstrarea teoremelor reciproce, precum şi în
demonstrarea teoremelor de unicitate.

24
Exerciţii rezolvate :
39n  4
7.1. Arătaţi că pentru orice n Є N fracţia este ireductibilă .
26n  3
Soluţie.
Presupunem că fracţia este reductibilă şi fie c.m.m.d.c. al numărătorului şi numitorului, respectiv
d = (39n + 4,26n + 3) cu d Є N* , d ≠ 1.
Din d | (39n + 4) şi d | (26n + 3) obţinem că d | (78n + 8) şi d | (78n + 9) ,
de unde rezultă că d | [(78n + 9) - (78n + 8)], deci d |1, de unde rezultă d = 1 .
Fals, deci fracţia este ireductibilă .

7.2. Să se arate că nu există numere naturale a pentru care numerele

14a  5 17 a  5
şi
9 12
să fie simultan naturale.

Soluţie.
14a  5 17a  5
Presupunem că există numere întregi a astfel ca şi
9 12
14a  5 17 a  5
să fie simultan naturale, adică pentru b,c Є N , unde b= şi c =
9 12
de unde rezultă 14a + 5 = 9b şi 17a – 5 = 12c.
Scăzând prima relaţie din a doua obţinem:

3a – 10 = 12c - 9b , de unde 3(a - 4c + 3b) = 10 .

Atunci obţinem că 3 divide pe 10, ceea ce este absurd.

7.3. Să se arate că dacă suma a cinci numere naturale distincte şi nenule


este 27 atunci printre ele se află cel puţin un număr prim .

Soluţie.
Presupunem, prin reducere la absurd, că printre cele cinci numere nu există un număr prim.
Dacă le considerăm pe cele mai mici numere atunci suma lor este 1 + 4 + 6 + 8 + 9 = 28, ceea ce contrazice
ipoteza şi atunci rezultă că există un număr prim printre cele cinci

7.4. Demonstraţi că nu există un număr natural de două cifre care să fie de


două ori mai mare decât răsturnatul său.

Soluţie.
Presupunem, prin reducere la absurd, că există xy astfel încât xy = 2yx .. Avem succesiv :
10 x + y = 20y + 2x ; 8x = 19y.
Rezultă astfel că 19 | x, imposibil deoarece x este cifră şi x < 10.

8. Calculul ultimei / penultimei cifre a unei


25
expresii numerice cu puteri sau a unui
produs
Metoda de determinare a ultimei / penultimei cifre a unei expresii numerice
cu puteri sau a unui produs are la bază următoarele observaţii :
1) toate puterile lui 5 se termină cu cifra 5 ;
2) toate puterile lui 6 se termină cu cifra 6 ;
3) toate puterile lui 10 se termină cu cifra 0, ba mai mult aceste puteri vor avea
un număr de zerouri egal cu exponentul ;
4) notând cu u( a ) ultima cifră a unui număr a , avem :
u(24k + 1 ) = 2 ; u(24k + 2 ) = 4 ; u(24k + 3 ) = 8 ; u(24(k + 1) ) = 6

u(34k + 1 ) = 3 ; u(34k + 2 ) = 9 ; u(34k + 3 ) = 7 ; u(34(k + 1) ) = 1

u(44k + 1 ) = 4 ; u(44k + 2 ) = 6 ; u(44k + 3 ) = 4 ; u(44(k + 1) ) = 6

u(74k + 1 ) = 7 ; u(74k + 2 ) = 9 ; u(74k + 3 ) = 3 ; u(74(k + 1) ) = 1

u(84k + 1 ) = 8 ; u(84k + 2 ) = 4 ; u(84k + 3 ) = 2 ; u(84(k + 1) ) = 6

u(94k + 1 ) = 9 ; u(94k + 2 ) = 1 ; u(94k + 3 ) = 9 ; u(94(k + 1) ) = 1


5) dacă n este un număr natural, atunci n4 are ultima cifră 0, 1, 5 sau 6 ;

6) au loc relaţiile :

u(34k + 1 ) + u(34k + 2 ) + u(34k + 3 ) + u(34(k + 1) ) = u( 3 + 9 +7 +1 = u( 20 ) = 0

u(44k + 1 ) + u(44k + 2 ) = u(44k + 3 ) + u(44(k + 1) ) = u( 10 ) = 0

u(74k + 1 ) + u(74k + 2 ) + u(74k + 3 ) + u(74(k + 1) ) = u( 7 + 9 + 3 +1 = u( 20 ) = 0

u(84k + 1 ) + u(84k + 2 ) + u(84k + 3 ) + u(84(k + 1) ) = u( 8 + 4 + 2 + 6 = u( 20 ) = 0

u(65k + 1)+u(65k + 2)+u(65k + 3)+u(65k + 4)+u(65k+5) = u( 6+6+6+6+6 ) = u( 30 ) = 0

Exerciţii rezolvate :

8.1. Să se determine ultima cifră a numărului


A = 22000 + 32001 + 42002 + 52003 + 72004 + 82005 + 92006 + 122010 + 162012
Soluţie.
Avem succesiv : u( 22000 ) = u( 24 x 500 ) = 6 ; u( 32001 ) = u( 34 x 500 + 1 ) = 3 ;
u( 42002 ) = u( 24 x 500 + 2 ) = 6 ; u( 52003 ) = 5 ; u( 72004 ) = u( 24 x 500 + 4 ) = 1 ;
u( 82005 ) = u( 84 x 501 + 1 ) = 8 ; u( 92006 ) = u( 94 x 501 + 2 ) = 1 ;

26
u( 122010 ) = u( 22010 ) = u( 24 x 502 + 2 ) = 4 ;
u( 162012 ) = u( 62012 ) = u( 24 x 503 ) = 6 ;
Deci obţinem u( A ) = u( 6 + 3 + 6 + 5 + 1 + 8 + 1 + 4 + 6 ) = u( 40 ) = 0

8.2. Să se determine ultimele două cifre ale numărului :


N = 728 + 726
Soluţie.
Avem succesiv 728 + 726 = 726( 72 + 1 )= 728 x 50 = 728 x 5 x 10 deci u(N ) =0 este ultima cifră
Atunci penultima cifră a lui N va fi ultima cifră a numărului 728 x 5, adică
U(728 x 5 ) = u(728 ) x u( 5 ) = u( 74 x 7 ) x u( 5 ) = u( 1 x 5 ) u ( 5 ) = 5, adică N are forma

N = ****…*50

8.3. Să se determine ultima cifră a numărului :


N = 11 + 22 + 33 + … + 2020
Soluţie.
Se observă că ne avantajează următoarea grupare a termenilor :
u( 11 ) + u( 1111 ) = u( 11 ) + u( 111 ) = u( 1 + 1 ) = u( 2 ) = 2;
u( 22 ) + u( 1212 ) = u( 22 ) + u( 212 ) = u( 4 + 6 ) = u( 10 ) = 0;
u( 33 ) + u( 1313 ) = u( 33 ) + u( 313 ) = u( 7 + 3 ) = u( 10 ) = 0;
u( 44 ) + u( 1414 ) = u( 44 ) + u( 414 ) = u( 6 + 6 ) = u( 12 ) = 2;
u( 55 ) + u( 1515 ) = u( 55 ) + u( 515 ) = u( 5 + 5 ) = u( 10 ) = 0;
u( 66 ) + u( 1616 ) = u( 66 ) + u( 616 ) = u( 6 + 6 ) = u( 12 ) = 2;
u( 77 ) + u( 1717 ) = u( 77 ) + u( 717 ) = u( 3 + 7 ) = u( 10 ) = 0;
u( 88 ) + u( 1818 ) = u( 88 ) + u( 818 ) = u( 6 + 4 ) = u( 10 ) = 0;
u( 99 ) + u( 1919 ) = u( 99 ) + u( 919 ) = u( 9 + 9 ) = u( 18 ) = 8;
u( 1010 ) + u( 2020 ) = 0
Atunci rezultă u( N ) = u( 2 + 2 + 2 + 8 ) = u( 14 ) = 4

8.4. Să se determine ultimele trei cifre ale numărului :


N = 22012 - 22010 + 22007
Soluţie.
Avem N = 22012 - 22010 + 22007 = 22007 ( 25 – 23 + 1 ) = 22005 x 25 = 22003 x 22 x 25 = 22003 x 100 .
Rezultă că ultimele două cifre ale lui N sunt 00.
Atunci antipenultima cifră a lui N va fi u( 22003 ) = u( 24 x 500 + 3 )= 8

Deci N = *** …*800

8.4. Să se determine ultimele cifre ale numerelor :


N = 1 + 3 + 32 + … + 357 şi respectiv M = 4 + 42 + … + 436
Soluţie.
Avem
u(34k + 1 ) + u(34k + 2 ) + u(34k + 3 ) + u(34(k + 1 ) = u(34k + 1 + 34k + 2 +34k + 3 +34(k + 1) = u( 3 + 9 + 7 + 1 ) = u( 20 ) = 0
Deci rezultă
u( 3 + 32 + 33 + 34 ) = 0 ;
u( 35 + 36 + 37 + 38 ) = 0 ;
……………………………
u( 353 + 354 + 355 + 356 ) 0
Mai rămân doi termeni, respectiv 1 şi 357 şi atunci u( 1 + 357 ) = u( 1 + 3 ) = u( 4 ) = 4
Deci rezultă : u ( N ) = 4 + 0 + 0 + … + 0 = 4

Apoi avem : u(44k + 1 ) + u(44k + 2 ) = u(44k + 3 ) + u(44(k + 1) ) = u( 10 ) = 0 şi atunci grupăm termenii,


doi câte doi, obţinem :
M = ( 4 + 42 ) + ( 43 + 44 ) + … + ( 435 + 436 )
Cum u(4 + 42) = u(43 + 44) = …. = u(435 + 436) = 0 , rezultă u(M ) = 0

8.5. Să se determine ultima cifră a numărului aflat pe locul 2012 al şirului


1, 3, 7, 15, 31, 63, ….

Soluţie.
27
Se observă că şirul este dat de următoarea regulă de formare a acestuia

a1 = 21 – 1; a2 = 22 – 1; a3 = 23 – 1 ; a4 = 24 – 1 ; …. ; an = 2n – 1 ; …..
Termenul aflat pe locul 2003 este a2003 = 22003 – 1 şi atunci u( a2003 ) = u ( 22003 – 1 ) = u( 22003 ) – U( 1 ) =

= u( 24 x 500 + 3 ) – 1 = 8 – 1 = 7

9. Calculul puterilor consecutive ale unui


număr natural
Problema care se pune este de a se calcula suma :

S = 1 + a + a2 + a3 + a4 + … + an , n Є N şi a Є N*

Pentru aceasta procedăm astfel :


 înmulţim egalitatea cu a şi obţinem
aS = a + a2 + a3 + a4 + a5 + … + an + an + 1
 facem diferenţa între cele două egalităţi şi obţinem

aS – S = (a + a2 + a3 + a4 + a5 + … + an + an + 1) – (1 + a + a2 + a3 + a4 + … + an ) =

= an + 1 – 1
 din ultima egalitate rezultă : S( a – 1 ) = an + 1 – 1 , deci

S = (an + 1 – 1) : ( a - 1 )

Exerciţii rezolvate :
9.1. Comparaţi a cu b , dacă a = 2 x252x253x … x2100 şi b = 1 + 2 + 22 + … + 23774.
51

Soluţie.
100101 5051

Avem
a  2(1 2 3...100) (1 2 3...50)  2 2 2
 2501012551 şi respectiv

b  1  2  22  ...  23774  23775  1


Deci rezultă a > b

9.2. Scrieţi numărul 20012002 ca sumă de 2001 numere consecutive .

Soluţie.
Avem succesiv : 20012002 = 2001 x 20012001 = ( 1 + 1 + 1 + … + 1) x 20012001 =
2001 termeni
= 20012001 + 20012001 + … + 20012001 = ( 20012001 – 1000 ) + ( 20012001 – 999 ) + ( 20012001 – 998 ) + …. +
2001 termeni

+ ( 20012001 – 2 ) + ( 20012001 – 1 ) + 20012001 + ( 20012001 + 1 ) + ( 20012001 + 2 ) + ( 20012001 + 3 ) + … +

+ ( 20012001 + 999 ) +( 20012001 + 1000 )

9.3. Aflaţi numărul natural n ştiind că :


4 + 50 + 4 x 5 + 4 x 52 + 4 x 53 + … + 4 x 55n – 1 + 2 x 54n = 4375

28
Soluţie.
Avem succesiv : 4 + 50 + 4 x 5 + 4 x 52 + 4 x 53 + … + 4 x 55n – 1 + 2 x 54n =

= 4 + 1 + 4 x 5 x ( 1 + 5 + 52 + … + 55n – 2 ) + 2 x 54n = 5 + 4 x 5[( 55n-1 -1 ) :4 ] + 2 x 54n =

= 5 + 5( 55n-1 -1 ) + 2 x 54n
Deci avem :

5 + 5( 55n-1 -1 ) + 2 x 54n = 4375  55n + 2 x 54n = 55 + 2 x 54 de unde rezultă n = 1

9.4. Aflaţi numărul natural n ştiind că :


( 1 + 2 + 22 + … + 21980 ) x n = 2 + 22 + 23 + … + 21982
Soluţie.
Avem S = 1 + 2 + 22 + … + 21980 pe care înmulţind-o cu 22 obţinem : 22 x S = 22 + 23 + 24 + … + 21982.
Apoi avem : 22 x S – S = 21982 – 1 sau 3S = 21982 – 1; S = ( 21982 – 1 ) : 3
De asemenea avem 2 + 22 + 23 + … + 21982 = 2( 1 + 2 + 23 + … + 21981 ) = 21982 - 1
Înlocuind în egalitatea din enunţ avem succesiv :

[( 21982 – 1 ) : 3] x n = 2( 21982 – 1 )  n = 6

9.5. Calculaţi suma :


S = 24 x 52002 + 8 x 32001 + 24 x 52000 + 8 x 31999 + … + 24 x 54 +
+ 8 x 33 + 24 x 52 + 8 x 3
Soluţie.
Punem 24 = 25 – 1 şi 8 = 9 – 1 şi avem succesiv S = S 1 + S2 unde :

S1 = 24 x 52002 + 24 x 52000 + … + 24 x 54 + 24 x 52 = ( 25 – 1 ) x 52002 + ( 25 – 1 ) x 52000 + … +


+ ( 25 – 1 ) x 54 + ( 25 – 1 ) x 52 = 52004 – 52002 + 52002 – 52000 + … + 56 – 54 + 54 – 52 = 52004 - 52
şi
S2 = 8 x 32001 + 8 x 31999 + … + 8 x 33 + 8 x 3 = ( 9 – 1 ) x 32001 + ( 9 – 1 ) x 31999 + … +
+ ( 9 – 1 ) x 33 + ( 9 – 1 ) x 3 = 32003 – 32001 + 32001 – 31999 + … + 35 – 35 + 33 – 3 = 32003 - 3
Atunci obţinem S = S1 + S2 = 52004 - 52 + 32003 – 3 = 52004 + 32003 – 28.

10. Pătrate perfecte. Cuburi perfecte.


Numărul natural n este pătrat perfect dacă există un număr natural a astfel
încât n = a2.
Ultima cifră a unui pătrat perfect poate fi 0, 1, 4, 5, 6, 9. De aici deducem că
nu pot fi pătrate perfecte numerele care se termină în 2, 3, 7 sau 8 .
De reţinut :
1) dacă numărul n este un pătrat perfect şi p un număr prim care îl divide
pe n, atunci p2 divide pe n . Ex. n = 16; p = 2 rezultă 2 | 16 şi 2 2 | 16
2) între două pătrate perfecte consecutive nu se află nici un pătrat perfect .dacă
numărul n este un pătrat perfect şi p un număr prim care îl divide
Numărul natural n este cub perfect dacă există un număr natural a astfel
încât n = a3.

Exerciţii rezolvate :

10.1. Să se arate că numărul A = 2( 1 + 2 + 3 + … + 2001 ) + 2002 este


pătrat perfect .

29
Soluţie.
Avem 1 + 2 + 3 + … + 2001 = ( 2001 x 2002 ) : 2 şi atunci avem
A = 2001 x 2002 + 2002 = 2002( 2001 + 1 ) = 2002 2

10.2. Să se arate că numerele a = 6738 + 9243 , b = 534 + 517 şi


c = 2n x 3n + 1 + 2n + 1 x 3n nu sunt pătrate perfecte.
Soluţie.
Avem u( 6738 ) = u( 738 ) = u( 74 x 9 + 2 ) = 9 ; u( 9243 ) = u( 243 ) = u( 24 x 10 + 3 ) = 8 şi atunci
u( a ) u( 9 + 8 ) = u( 17 ) = 7 deci a nu poate fi pătrat perfect .
Analog avem :
b = 534 + 517 = 517( 517 + 1 ) şi avem u( b ) = u( 517 )x u(517 + 1 ) = u[5 x ( 5 + 1 )] = u( 5 x 6) = u(30 ) = 0
deci nu se poate trage concluzii despre b prin metoda ultimei cifre.
Observăm însă că

517 x 517 < 517 x (517 + 1 ) < (517 + 1)( 517 + 1) , deci ( 517 )2 < b < ( 517 + 1 )2 şi atunci b nu poate fi pătrat perfect
deoarece este situat între două pătrate perfecte consecutive , respectiv (5 17 )2 şi (517 + 1)2 .
Avem c = 2n x 3n + 1 + 2n + 1 x 3n = 2n x 3n( 3 + 2 ) = 2n x 3n x 5.
Se observă că 5 | c dar 52 nu divide c , deci c nu poate fi pătrat perfect .

10.3. Să se arate că suma primelor numere naturale impare este pătrat perfect,
iar suma primelor n numere naturale pare nu este pătrat perfect .

Soluţie.
Avem succesiv 1 + 3 + 5 + 7 + … + ( 2n - 1 ) = ( 1 + 2n - 1) + ( 3 + 2n - 3 ) + … = 2n + 2n + 2n + … + 2n =
n : 2 parnteze n : 2 ori
= 2n x n : 2 = n2
Apoi 2 + 4 + 6 + … + 2n = 2( 1 +2 + 3 + … + n ) = 2n( n+1 ) : 2 = n( n + 1 )
Dar n2 < n( n+1 ) < ( n + 1 )2 deci suma este cuprinsă între două numere naturale pătrate perfecte succesive şi nu
poate fi pătrat perfect.

10.4. Determinaţi toate numerele naturale de trei cifre, scrise în baza 10, care
adunate cu răsturnatele lor să dea pătrate perfecte .

Soluţie.
Fie abc numărul căutat. Atunci avem abc + cba = n 2  100a +10b + c + 100c + 10b + a = 101( a + c ) + 20b = n 2.
Se ştie că ultima cifră a unui pătrat poate fi 0, 1, 4, 5, 6 sau 9 şi că ultima cifră a lui 20b este 0 , rezultă :
1) a + c = 1 , b = 1 de unde avem a = 1 , c = 0 şi b = 1 , deci abc = 110 şi n 2 = 110 + 11 = 121 = 112
2) a + c = 1 , b = 1 de unde avem a = 0 , c = 1 şi b = 1 , deci abc = 110 şi n 2 = 11 + 110 = 121 = 112 - nu este
soluţie deoarece trebuie ca a  0 .
3) a + c = 4 , b = 4 de unde avem a = 1, c = 3 şi b = 4 , deci abc = 143 şi n 2 = 143 + 341 = 484 = 222
4) a + c = 4 , b = 4 de unde avem a = 2, c = 2 şi b = 4 , deci abc = 242 şi n 2 = 242 + 242 = 484 = 222
5) a + c = 4 , b = 4 de unde avem a = 3, c = 1 şi b = 4 , deci abc = 341 şi n 2 = 341 + 143 = 484 = 222
6) a + c = 5 , b = 6 de unde avem a = 1, c = 4 şi b = 6 , deci abc = 164 şi n 2 = 164 + 461 = 625 = 252
7) a + c = 5 , b = 6 de unde avem a = 2, c = 3 şi b = 6 , deci abc = 263 şi n 2 = 263 + 362 = 625 = 252
8) a + c = 5 , b = 6 de unde avem a = 3, c = 2 şi b = 6 , deci abc = 362 şi n 2 = 362 + 263 = 625 = 252
9) a + c = 5 , b = 6 de unde avem a = 4, c = 1 şi b = 6 , deci abc = 461 şi n 2 = 461 + 164 = 625 = 252
10) a + c = 6 nu pot da soluţii deoarece 626, 646, 666, 686, etc . nu sunt pătrate perfecte .
11) a + c = 9 , b = 9 de unde avem a = 1, c = 8 şi b = 9 , deci abc = 198 şi n 2 = 198 + 891 = 1089 = 332

12) a + c = 9 , b = 9 de unde avem a = 2, c = 7 şi b = 9 , deci abc = 297 şi n 2 = 297 + 792 = 1089 = 332

13) a + c = 9 , b = 9 de unde avem a = 3, c = 6 şi b = 9 , deci abc = 196 şi n 2 = 396 + 693 = 1089 = 332

14) a + c = 9 , b = 9 de unde avem a = 4, c = 5 şi b = 9 , deci abc = 195 şi n 2 = 495 + 594 = 1089 = 332
15) a + c = 9 , b = 9 de unde avem a = 5, c = 4 şi b = 9 , deci abc = 194 şi n 2 = 594 + 495 = 1089 = 332
16) a + c = 9 , b = 9 de unde avem a = 6, c = 3 şi b = 9 , deci abc = 110 şi n 2 = 693 + 396 = 1089 = 332
17) a + c = 9 , b = 9 de unde avem a = 7, c = 2 şi b = 9 , deci abc = 110 şi n 2 = 792 + 297 = 1089 = 332
18) a + c = 9 , b = 9 de unde avem a = 8, c = 1 şi b = 9 , deci abc = 110 şi n 2 = 891 + 198 = 1089 = 332

30
10.5. Determinaţi numerele naturale ab , scrise în baza 10, care adunate cu răsturnatul lor
dau un pătrat perfect iar dacă din ele se scad răsturnatele lor se obţine un cub perfect.

Soluţie.
Avem succesiv :ab + ba = k2 şi ab – ba = p3 .
Obţinem astfel : 10a + b + 10b + a = 11( a + b ) = k 2 şi respectiv 10a + b – 10b – a = 9( a – b ) = p 3 .
Din 11( a + b ) = k2 rezultă a + b = 11x2 iar din 9( a – b ) = p3 rezultă a – b = 9y3 cu a, b ≤ 9 .
Astfel obţinem :a = 7 şi b = 4 deoarece x = 1 şi y = 1pentru a avea a, b ≤ 9 .

10.5. Să se arate că A = x4 + 10y + 2 nu poate fi pătrat perfect, oricare ar fi x şi y numere


naturale.

Soluţie.
Avem u( x4 ) Є { 0, 1, 5, 6 }, u( 10y ) = 0 şi atunci u( A ) Є { 0 + 0 + 2, 1 + 0 + 2, 5 + 0 + 2, 6 + 0 + 2 } = { 2, 3, 7,8 }
deci A nu poate fi pătrat perfect .

10.6. Se dau numerele naturale :


A = ( 220 : 45 -1024 + 102 – 62 – 82 )2011+ 20112 – 2010 x 2011
şi B = 2 x 103 + 1 – 24 x 53
a) calculaţi : ( A – 2010 x B )2011
b) arătaţi că 2012 x ( A + B ) este pătrat perfect .

Soluţie.
a) Avem : a = ( 220 : 210 – 210 + 102 – 62 – 82 )2011 = ( 210 – 210 + 100 – 36 - 64 )2011 = 0
Atunci A = a + 20112 – 2010 x 2011 = 2011( 2011 – 2010 ) = 2011
Apoi B = 2 x 103 – 2 x 103 + 1 = 1
Astfel obţinem : ( A – 2010 x B )2011 = ( 2011 – 2010 x 1 )2011 = 1
b) Avem 2012 x ( A + B ) = 2012 x 2012 = 20122

11. Criptaritm
Problemele care se sunt de acest gen constau în determinarea cifrelor unui
număr natural, scris în baza 10, folosind operaţiile definite în mulţimea
numerelor naturale . În asemenea exerciţii numerele sunt reprezentate de
litere. Literele de acelaşi fel reprezintă cifre egale , dar literele diferite nu
reprezintă neapărat cifre diferite.

Exerciţii rezolvate :
11.1. Să se determine numărul natural de 2 cifre, scris în baza 10, care are
proprietatea că ab + bc = 154 .

Soluţie.
Avem succesiv : ab + ba = 10a + b + 10b + a = 11( a + b ) = 154 , de unde avem a + b = 14.
Din a + b = 14 deoarece a şi b sunt cifre rezultă soluţiile : a = 5 şi b = 9; a = 6 şi b = 8; a = 7 şi b = 7;
a = 8 şi b = 6 ; a = 9 şi b = 5 .

11.2. Să se reconstituie adunarea :


BALADA + LADA + ADA + DA + A = 257605

31
Soluţie.
Avem : BALADA +
LADA
ADA
DA
A
---------------
257605
Analizând suma rezultă A + A + A + A + A = 5A ≤ 5 X 9 = 45 deci avem succesiv :
A + A + A + A = 5A Є { 5, 15, 25, 35 , 45 } şi atunci obţinem :
1) dacă 5A = 5, atunci A = 1 şi adunând cifrele zecilor rezultă D + D + D + D = 4D ≤ 4 X 9 = 36 ; cum din
însumarea cifrelor zecilor la rezultat se obţine 0, atunci D + D + D + D = 4D Є { 0, 10, 20, 30 } , de unde
rezultă D = 0
Dacă D = 0 , atunci adunând cifrele sutelor avem A + A + A = 3A = 6, deci A = 2 ≠ 5 deci nu avem soluţii.
2) dacă 5A = 15, atunci A = 3 şi adunând cifrele zecilor rezultă D + D + D + D + 1 = 4D + 1 ≤ 4 X 9 + 1 = 37 ;
cum din însumarea cifrelor zecilor la rezultat se obţine 0, atunci
D + D + D + D + 1 = 4D + 1 Є { 0, 10, 20, 30 } , care nu dă soluţii .
3) dacă 5A = 25, atunci A = 5 şi adunând cifrele zecilor rezultă D + D + D + D + 2 = 4D + 2 ≤ 4 X 9 +2 = 38 ;
cum din însumarea cifrelor zecilor la rezultat se obţine 0, atunci 4D + 2 Є { 0, 10, 20, 30 } , de unde rezultă
soluţiile D = 2 şi D = 7
3. 1. Pentru D = 2 avem 4D + 2 = 10 şi atunci adunând cifrele sutelor la rezultat se obţine cifra 6 deci avem
A + A + A + 1 = 3A + 1 ≤ 3 x 9 + 1 = 28 şi respectiv obţinem 3A + 1 = 6 sau 3A + 1 = 16 sau
3A + 1 = 26. Din 3A + 1 = 6 şi 3A + 1 = 26 nu avem soluţii iar din 3A + 1 = 16 obţinem
A = 5 ceea ce verifică 5A = 25.
Pentru A = 5 şi D = 2 adunând cifrele miilor avem L + L + 1 = 2L + 1 ≤ 2 X 9 + 1 = 19, deci rezultă
2L + 1 = 7 sau 2L + 1 = 17
3.1.1. Din 2L + 1 = 7 rezultă L = 3 şi atunci din adunarea zecilor de mii avem A = 5 ceea ce
verifică presupunerea iniţială ;
3.1.2. Din 2L + 1 = 17 rezultă L = 8 şi atunci din adunarea zecilor de mii avem A + 1 = 5 ceea ce
nu verifică presupunerea iniţială ;

3. 2. Pentru D = 7 avem 4D + 2 = 30 şi atunci adunând cifrele sutelor la rezultat se obţine cifra 6 deci avem
A + A + A + 3 = 3A + 3 ≤ 3 x 9 + 3 = 30 şi respectiv obţinem 3A + 3 = 6 sau 3A + 3 = 16 sau
3A + 3 = 26. Din 3A + 3 = 6 obţinem a = 1 ≠ 5 iar din 3A + 3 = 16 şi 3A + 3 = 26 nu obţinem
soluţii
Pentru A = 5 , D = 2 şi L = 3 din adunarea cifrelor zecilor şi sutelor de mii rezultă B = 2.
Aşadar se obţine soluţia : A = 2, B = 2, D = 2 şi L = 3 care verifică suma, respectiv :

253525 + 3525 + 525 + 25 + 5 = 257605

4) dacă 5A = 35, atunci A = 7 şi adunând cifrele zecilor rezultă D + D + D + D + 3 = 4D + 3 ≤ 4 X 9+3 = 39 ;


cum din însumarea cifrelor zecilor la rezultat se obţine 0, atunci 4D + 3 Є { 0, 10, 20, 30 } ,
de unde rezultă că nu avem soluţii.

5) dacă 5A = 45, atunci A = 9 şi adunând cifrele zecilor rezultă D + D + D + D + 4 = 4D + 3 ≤ 4 X 9+4 = 40 ;


cum din însumarea cifrelor zecilor la rezultat se obţine 0, atunci 4D + 4 Є { 0, 10, 20, 30, 40 } ,
de unde rezultă 4D + 4 = 20 Şi 4D + 4 = 40, de unde obţinem D = 4 şi respectiv D = 9.
5. 1. Pentru D = 4 avem 4D + 4 = 20 şi atunci adunând cifrele sutelor la rezultat se obţine cifra 6 deci avem
A + A + A + 2 = 3A + 2 ≤ 3 x 9 + 2 = 29 şi respectiv obţinem 3A + 2 = 6 sau 3A + 2 = 16 din care nu
obţinem soluţii .
5. 2. Pentru D = 9 avem 4D + 4 = 40 şi atunci adunând cifrele sutelor la rezultat se obţine cifra 6 deci avem
A + A + A + 4 = 3A + 4 ≤ 3 x 9 + 4 = 31 şi respectiv obţinem 3A + 4 = 6 sau 3A + 4 = 16 sau
3A + 4 = 26 .
Din 3A + 4 = 6 şi 3A + 4 = 26 nu avem soluţii iar din 3A + 4 = 16 obţinem 3A = 12 , A = 4 ≠ 5 deci
nu este soluţie.

11.3. Aflaţi numerele de patru cifre abcd , care verifică relaţia :

abcd + bcd + cd + d = 3102

Soluţie.
Aducem numărul la forma de scriere sistematică în baza 10 şi obţinem :
1000a + 100b + 10c + d + 100b + 10c + d + 10c + d + d = 3102 sau
1000a + 200b + 30c + 4d = 3102 sau
32
500a + 100b + 15c + 2d = 1551 (1)
Din ( 1 ) rezultă că suma din membrul stâng având valoarea 1551 conduce la concluzia că a poate lua
valoarea 1, 2 sau 3.
1) pentru a = 1 se obţine 100b + 15c + 2d = 1051 (2)
Din ( 2 ) rezultă : 15 x 9 + 2 x 8 = 153 , deci 15c + 2d ≤ 153 cu c, d ≤ 9 .
Dacă 15c + 2d ≤ 153, atunci 100b ≥ 1501 – 153 = 898 , deci avem b ≥ 898 : 100, adică b = 9.
Dacă b = 9 atunci 15c + 2d = 1051 – 900 = 151 (3)
Dacă 15c + 2d = 151, deoarece 2 x 9 = 18 rezultă 2d ≤ 18 şi atunci 15c ≥ 151 – 18 = 123 ,
deci c ≥ 123 : 15 adică c = 9 .
Pentru c = 9 rezultă 2d = 151 – 135 = 16, deci obţinem d = 8

Am obţinut prima soluţie abcd = 1998

2) pentru a = 2 se obţine 100b + 15c + 2d = 551 (4)


Din ( 4 ) rezultă : 15 x 9 + 2 x 8 = 153 , deci 15c + 2d ≤ 153 cu c ≤ 9 şi d ≤ 8.
Dacă 15c + 2d ≤ 153, atunci 100b ≥ 501 – 153 = 398 , deci avem b ≥ 398 : 100, adică b ≥ 4 .
2.1. Dacă b = 4 atunci 15c + 2d = 551 – 400 = 151 (5)
Dacă 15c + 2d = 151, deoarece 2 x 9 = 18 rezultă 2d ≤ 18 şi atunci 15c ≥ 151 – 18 = 133 ,
deci c ≥ 133 : 15 adică c = 9 .
Pentru c = 9 rezultă 2d = 151 – 135 = 16, deci obţinem d = 8

Am obţinut a doua soluţie abcd = 2498

2.2. Dacă b = 5 atunci 15c + 2d = 551 – 500 = 51 (6)


Dacă 15c + 2d = 51, deoarece 2 x 9 = 18 rezultă 2d ≤ 18 şi atunci 15c ≥ 51 – 18 = 33 ,
deci c ≥ 33 : 15 adică c ≥ 3 .
2.2.1. Pentru c = 3 rezultă 2d = 51 – 45 = 6, deci obţinem d = 3

Am obţinut a treia soluţie abcd = 2533

2.2.2. Pentru c ≥ 4 rezultă 2d = 51 – 60 < 0 deci nu avem solufii

2.3. Dacă b ≥ 6 atunci 15c + 2d = 551 – 600 < 0 deci nu avem soluţii .

3) pentru a = 3 se obţine 100b + 15c + 2d = 51 (7)


Din ( 7 ) rezultă b = 0 şi atunci 15c + 2d = 51
Dacă 15c + 2d = 51, atunci 15c ≥ 51 – 18 = 33 , deci avem c ≥ 33 : 15, adică c ≥ 3 .
3.1. Dacă c = 3 atunci 2d = 51 – 45 = 6 , adică d = 3

Am obţinut a patra soluţie abcd = 3033

3.2. Dacă c ≥ 4 atunci 2d = 51 – 60 < 0 deci nu avem soluţii .

11.4. Determinaţi cifra a ştiind că numărul a9 x a1 + 16 , scris în baza 10,


este pătrat perfect .

Soluţie.
Avem succesiv : ( 10a + 9 )( 10a + 1 ) + 16 = 100a2 + 100a + 25 = 25( 4a2 + 4a + 1) =
= 25( 4a2 + 2a + 2a + 1 ) = 25[( 2a( 2a + 1 ) + ( 2a + 1 )] = 25( 2a + 1)(2a + 1 ) = 25( 2a + 1 ) 2 =

= [5(2a + 1 )]2 = ( 10a + 5 )2 = a52 , deci a Є { 1,2,3,…, 8, 9 }

11.5. Determinaţi numărul ab ştiind că 3 x ab + ba = 114.

Soluţie.
Avem succesiv : 3( 10a + b )+ 10b + a = 114  31a + 13b = 114
Deoarece a are coeficientul cel mai mare în raport cu a avem : 31a < 114, adică a < 114 : 31  a ≤ 3.
Pentru a = 3 avem 31 x 3 + 13b = 114  13b = 21 – nu avem soluţii ;
Pentru a = 2 avem 31 x 2 + 13b = 114  13b = 52 – rezultă b = 4 şi o primă soluţie este ab = 24
Pentru a = 1 avem 31 x 1 + 13b = 114  13b = 21 – nu avem soluţii .

33
11.6. Determinaţi numerele abc şi suma acestora ştiind că suma dintre acest
număr şi răsturnatul său este 645 iar cifrele a, b, c sunt nenule.

Soluţie.
Avem egalitatea abc + cba = 645  101a + 20b + 101c = 645  101(a + c ) + 20b = 645 

 101( a + c ) = 645 – 20b  101(a + b ) = 5( 129 – 4b ).Am procedat astfel pentru a elimina 2 necunoscute
şi a rămâne cu una singură iar relaţia ne-a permis acest lucru .
Din ultima egalitate rezultă 5 | ( a + c ) , deci ( a + c ) Є { 5, 10, 15 } deoarece a + b este suma a două cifre.
Dacă a + c = 5, rezultă 645 – 20b = 505 ; 20b = 645 – 505 = 140, de unde b = 7
Dacă a + c = 10, rezultă 645 – 20b = 1010 ; 20b = 645 – 1010 < 0 deci nu avem soluţii ;
Dacă a + c = 15, rezultă 645 – 20b = 1515 ; 20b = 645 – 1515 < 0 deci nu avem soluţii .
Pentru b = 7 şi a + c = 5 avem : a = 1 şi c = 4 sau ; a = 2 şi c = 3 sau a = 3 şi c = 2 sau a = 4 şi c = 1 .
Rezultă astfel soluţiile : abc Є { 174, 273, 372, 471 } şi atunci S = ( 174 + 471 ) + ( 273 + 372 ) = 2 x 645 = 1290 .

11.7. Determinaţi numerele abc şi xy ştiind că abc + bc + c = 2xy + 57 .

Soluţie.
Se ştie că 210 = 1024 şi 211 = 2048, dacă a, b, c ar lua valoarea maximă atunci abc + bc + c = 999 + 99 + 9 = 1107,

deci xy ≤ 10, adică xy = 10 .

Pentru xy = 10 rezultă abc + bc + c = 1024 + 57 = 1081  100a + 20b + 3c = 1081.


Deoarece coeficientul cel mai mare îl are a rezultă că pentru a = 9 avem 20b + 3c = 1081 – 900 = 181 .
Cum u( 20b ) = 0 rezultă u( 3c ) = 1, adică c = 7 şi atunci 20b + 21 = 181 ; 20b = 160 ; b = 8, deci o primă
soluţie este a = 9 , b = 8 şi c = 7 .respectiv x = 1 şi y = 0.
Pentru a ≤ 8 rezultă 20b + 3c ≥ 1081 – 800 adică 20b + 3c > 281 .Analog singura valoare pentru c este c = 7
şi atunci rezultă 20b > 281 - 21 = 260 , deci b > 13 care nu convine .

11.8. Determinaţi cifrele nenule x, y, z ştiind că xx  yy = xzzx

Soluţie.
Avem succesiv : 11x  11y = 1001x + 110z  112xy = 11( 91x + 10z )  11xy = 91x + 10y 
x( 11y – 91 ) = 10z
Dar trebuie ca 11y – 91 > , deci y ≥ 9, adică y = 9.
Pentru y = 9 obţinem 8x = 10z  4x = 5z , de unde avem x = 5 şi z = 4.
Verificare 55 x 99 = 5445

11.9. Determinaţi numerele de două cifre ab , cu proprietatea

ab2 = cde şi respectiv ba2 = edc

Soluţie.
Deoarece membrul stâng este un pătrat perfect rezultă că ultima cifră a membrului drept , care este nenulă ,
adică e ≠ 0 şi c ≠ 0 , trebuie să aparţină mulţimii { 1, 4, 5, 6, 9 }.
1) pentru c = 1 , avem
ab2 = 1de şi respectiv ba2 = ed1  a =1 sau a = 9 pentru a obţine ultima
cifră a membrului drept egală cu 1.
1.1. Pentru a = 1 rezultă
1b2 = 1de şi respectiv b12 = ed1 (1)
Din e Є { 1, 4, 5, 6, 9 } rezultă :
1.1.1. pentru e =1 , atunci b =1 sau b = 9
1.1.1.1. Pentru b = 1 avem 112 = 121 , deci d = 2 care verifică relaţia ( 1 ) şi deci o
soluţie este a = 1 şi b = 1 .

1.1.1.2. Pentru b = 9 avem 192 = 361 şÎ 912 = 8281 care nu verifică relaţia ( 1 ) .

1.1.2. pentru e = 4, arunci b = 2 sau b = 8 şi avem :


1.1.2.1. dacă b = 2 rezultă 122 = 144 şi 212 = 441 care verifică relaţia ( 1 ) pentru e = 4 şi d = 4,
şi deci o altă soluţie este a = 1 şi b = 2
1.1.2.2. dacă b = 8 rezultă 182 = 324 şi 812 = 6561 care nu verifică relaţia ( 1 )
34
1.1.3. pentru e = 5, arunci b = 5 şi avem 15 2 = 225 şi 512 = 2601 care nu verifică relaţia ( 1 )

1.1.4. pentru e = 6, arunci b = 4 sau b = 6 şi avem

1.1.4.1. dacă b = 4 rezultă 142 = 196 şi 412 = 1681 care nu verifică relaţia ( 1 )

1.1.4.2. dacă b = 6 rezultă 162 = 256 şi 612 = 3721 care nu verifică relaţia ( 1 )

1.1.5. pentru e = 9, arunci b = 3 sau b = 7 şi avem

1.1.5.1. dacă b = 3 rezultă 132 = 169 şi 312 = 961 care verifică relaţia ( 1 ) pentru e = 9 şi d = 6,
şi deci o altă soluţie este a = 1 şi b = 3

1.1.5.2. dacă b = 7 rezultă 172 = 289 şi 712 = 5041 care nu verifică relaţia ( 1 )

1.2. Pentru a = 9 rezultă


9b2 va fi un număr cu patru cifre şi deci nu sunt verificare relaţiile din enunţ .
Ca atare nu avem soluţii.

2) pentru c = 4 , avem
ab2 = 4de şi respectiv ba2 = ed4  a =2 sau a = 8 pentru a obţine ultima
cifră a membrului drept egală cu 4.
2.1. Pentru a = 2 rezultă
2b2 = 4de şi respectiv b22 = ed4 (2)
Din e Є { 1, 4, 5, 6, 9 } rezultă :
2.1.1.pentru e =1 , atunci b =1 sau b = 9
2.1.1.1. Pentru b = 1 avem 212 = 441 şi 122 = 144, deci d = 4 care verifică relaţia ( 2 ) şi atunci
o soluţie este a = 2 şi b = 1 .

2.1.1.2. Pentru b = 9 avem 292 = 841 şÎ 922 = 8464 care nu verifică relaţia ( 2 ) .

2.1.2. pentru e = 4, arunci b = 2 sau b = 8 şi avem :


2.1.2.1. dacă b = 2 rezultă 222 = 484 deci d = 8 care verifică relaţia ( 2 ) şi atunci o
o altă soluţie este a = 2 şi b = 2
2.1.2.2. dacă b = 8 rezultă 282 = 784 şi 812 = 6724 care nu verifică relaţia ( 2 )

2.1.3. pentru e = 5, arunci b = 5 şi avem 25 2 = 625 şi 522 = 2704 care nu verifică relaţia ( 2 )

2.1.4. pentru e = 6, arunci b = 4 sau b = 6 şi avem

2.1.4.1. dacă b = 4 rezultă 242 = 576 şi 422 = 1681 care nu verifică relaţia ( 2 )

2.1.4.1. dacă b = 6 rezultă 162 = 256 şi 612 = 1764 care nu verifică relaţia ( 2 )

2.1.5. pentru e = 9, arunci b = 3 sau b = 7 şi avem

2.1.5.1. dacă b = 3 rezultă 232 = 529 şi 322 = 1024 care nu verifică relaţia ( 2 )

2.1.5.2. dacă b = 7 rezultă 272 = 729 şi 722 = 5184 care nu verifică relaţia ( 2 )

2.2. Pentru a = 8 rezultă


8b2 va fi un număr cu patru cifre şi deci nu sunt verificare relaţiile din enunţ .
Ca atare nu avem soluţii.

3) pentru c = 5 , avem
ab2 = 5de şi respectiv ba2 = ed5  a =5 pentru a obţine ultima
cifră a membrului drept egală cu 5.

3.1. Pentru a = 4 rezultă


5b2 este un număr mai mare de trei cifre şi deci nu avem soluţii.

4) pentru c = 6 , avem
ab2 = 6de şi respectiv ba2 = ed6  a = 4 sau a = 6 pentru a obţine ultima
cifră a membrului drept egală cu 6.

35
4.1. Pentru a = 4 rezultă
4b2 este un număr mai mare de trei cifre şi deci nu avem soluţii.

4.2. Pentru a = 6 rezultă


6b2 este un număr mai mare de trei cifre şi deci nu avem soluţii.

5) pentru c = 9 , avem
ab2 = 9de şi respectiv ba2 = ed9  a = 3 sau a = 7 pentru a obţine ultima
cifră a membrului drept egală cu 9.
3b2 = 9de şi respectiv b32 = ed1 (4)
Din e Є { 1, 4, 5, 6, 9 } rezultă :
1.1.2. pentru e =1 , atunci b =1 sau b = 9
1.1.1.1. Pentru b = 1 avem 312 = 961 şi 132 = 169 , deci d = 6 care verifică relaţia ( 4 ) şi deci o
soluţie este a = 3 şi b = 1 .

1.1.1.2. Pentru b = 9 avem 392 = 1521 şÎ 932 = 8649 care nu verifică relaţia ( 4 ) .

1.1.2. pentru e = 4, arunci b = 2 sau b = 8 şi avem :


1.1.2.1. dacă b = 2 rezultă 322 are mai mult de două cifre şi deci nu este verificată relaţia ( 4 )
1.1.2.2. dacă b = 8 rezultă 382 are mai mult de două cifre şi deci nu este verificată relaţia ( 4 )

1.1.3. pentru e = 5, arunci b = 5 şi avem 35 2 are mai mult de două cifre şi deci nu este verificată relaţia ( 4 )
1.1.4. pentru e = 6, arunci b = 4 sau b = 6 şi avem

1.1.4.1. dacă b = 4 rezultă 342 are mai mult de două cifre şi deci nu este verificată relaţia ( 4 )
1.1.4.3. dacă b = 6 rezultă 362 are mai mult de două cifre şi deci nu este verificată relaţia ( 4 )

1.1.5. pentru e = 9, arunci b = 3 sau b = 7 şi avem

1.1.5.1. dacă b = 3 rezultă 332 are mai mult de două cifre şi deci nu este verificată relaţia ( 4 )

1.1.5.2. dacă b = 7 rezultă 372 are mai mult de două cifre şi deci nu este verificată relaţia ( 4 )
1.2. Pentru a = 7 rezultă
7b2 va fi un număr cu patru cifre şi deci nu sunt verificare relaţiile din enunţ .
Ca atare nu avem soluţii.

În concluzie avem soluţiile : ab Є { 11, 12, 13, 21, 22, 31 }

12. Divizibilitatea numerelor naturale


1. Criteriu de divizibilitate
Definiţie = prin criteriu de divizibilitate a unui număr natural m ,
se înţelege o condiţie necesară şi suficientă pentru ca
numărul m să se împartă exact prin numărul d iar acest
lucru se notează m = Md ( se citeşte „ m este multiplu de d „)

2. Notaţia unui număr în sistemul zecimal


Un număr natural m , în sistemul zecimal, se notează în mod unic astfel :

m = anan-1an-2 …. a3a2a1a0 (1)


unde a0, a1, a2, … , an – 2 , an – 1, an sunt cifrele sistemului zecimal, adică
avem a0, a1, a2, … , an – 2 , an – 1, an Є { 0, 1, 2, 3, 4, 5, 6, 7, 8, 9 }
Relaţia ( 1 ) se poate scrie în mod unic astfel :
m = an x 10n + an - 1 x 10n - 1 + an - 2 x 10n - 2 + … + a3 x 103 + a2 x 102 + a1 x 10 + a0
cu an ≠ 0.

36
3. Criteriu general de divizibilitate
Condiţia necesară şi suficientă ca un număr natural
m = anan-1an-2 …. a3a2a1a0 să fie divizibil prin numărul natural d este
ca suma

S = ak – 1ak – 2 …a2a1a0 – q x a2k – 1a2k – 2 …ak + 1ak + q2 x a3k – 1a3k – 2 …a2k + 1a2k –

- q3 x a4k – 1a4k – 2 …a3k + 1a3k

să fie multiplu de d unde d = 10k + q , k ≤ n este număr natural


iar q este de asemenea un număr natural .
Metoda de demonstrare se bazează pe faptul că m se poate scrie
succesiv astfel :

m = an x 10n + an - 1 x 10n - 1 + an - 2 x 10n - 2 + … + a3 x 103 + a2 x 102 + a1 x 10 + a0 =

= ( ak-1x10k-1+ ak-2x10k-2 + … + a2x102 + a1x10 + a0 ) +

+ 10k( a2k-1x10k-1+ a2k-2x10k-2 + … + ak+2 x102 + ak+1x10 + ak ) +

+ 102k( a3k-1x10k-1+ a3k-2x10k-2 + … + a2k+2 x102 + a2k+1x10 + a2k ) + …… =

= ak – 1ak – 2 …a2a0 + 10k x a2k – 1a2k – 2 …ak + 1ak + 102k x a3k – 1a3k – 2 …ak + 1ak +

+ [(10k + q ) – q ] x a2k – 1a2k – 2 …ak + 1ak + [(102k - q2 ) + q2 ] x a3k – 1a3k – 2 …a2k + 1a2k +

+ [(103k + p3 ) - p3 ] x a4k – 1a4k – 2 …a3k + 1a3k + …… = M ( 10k + q ) + s = M ( d ) + s

4. Teoreme de divizibilitate

Teorema 1 : Dacă fiecare termen al unei sume sau diferenţe este divizibil
cu un număr atunci şi suma sau diferenţa sunt divizibile
cu acel număr .

Teorema 2 : Dacă o sumă sau diferenţă a două numere şi unul din cele
două numere sunt divizibile cu un număr „d” atunci şi cel
de al doilea număr este divizibil cu „d”.

Teorema 3 : Un produs este divizibil cu un număr dat dacă cel puţin


un factor al produsului este divizibil cu acel număr.

Teorema 4 : Într-o împărţire cu rest , dacă deîmpărţitul şi împărţitorul


sunt divizibile cu un număr dat atunci şi restul va fi divizibil
cu acel număr.
5. Proprietăţile divizibilităţii

P.1 : Relaţia de divizibilitate este reflexivă, adică


„ orice număr este divizibil cu el însuşi, respectiv a | a”
37
P. 2 : Relaţia de divizibilitate este antisimetrică, adică
„ dacă a | b şi b | a atunci a = b „.

P. 3 : Relaţia de divizibilitate este tranzitivă, adică


„ dacă a | b şi b | c atunci a | c „

6. Numărul şi suma divizorilor unui număr natural, număr perfect

Teoremă : Orice număr natural compus N se poate descompune


după relaţia :
N  p1r1  p2r2  p3r3  ...  pkrk
unde p1, p2, p3, …, pk sunt numere prime între ele.

Teoremă : Numărul divizorilor unui număr natural compus N este


dat de relaţia :
 (N ) (r1  1)(r2  1)r3  1)...(rk  1)

Teoremă : Suma divizorilor unui număr natural compus N este


dat de relaţia :
p r1 1  1 p2r2 1  1 p3r3 1  1 p rk 1  1
 (N )  1    ...  k
p1  1 p2  1 p3  1 pk  1

7. Cel mai mare divizor comun ( c.m.m.d.c. ) a două sau mai


multe numere naturale
Definiţie = cel mai mare dintre divizorii a două sau mai multe numere
se numeşte cel mai mare divizor comun .

Notaţii : c.m.m.d.c.( a, b ) = ( a, b )

c.m.m.d.c.( a1, a2, a3, … , ak ) = (a1, a2, a3, … , ak )

8. Cel mai mic multiplu comun ( c.m.m.m.c. ) a două sau mai


multe numere naturale
Definiţie = cel mai mic dintre multipliii a două sau mai multe numere
se numeşte cel mai mic multiplu comun .

Notaţii : c.m.m.m.c.( a, b ) = [ a, b ]

c.m.m.d.c.( a1, a2, a3, … , ak ) = [a1, a2, a3, … , ak ]

9. Divizibilitatea unei expresii cu un număr


Algoritm : 1. Se pune în evidenţă numărul factor comun
38
2. Se descompune numărul dat în produs de factori numere
prime între ele şi se verifică divizibilitatea expresiei prin
fiecare factor.

10. Determinarea a două numere naturale când se cunosc


c.m.m.d.c. sau c.m.m.m.c. şi respectiv suma sau produsul lor.
Algoritm : 1. Se exprimă cele două numere ca produse dintre c.m.m.d.c.
şi două numere prime între ele ;
2. Se înlocuiesc aceste produse în relaţia sumei sau
produsului şi se obţine suma sau produsul numerelor
prime între ele ;
3. Din relaţia sumei sau produsului se determină valorile
numerelor prime între ele

Exerciţii rezolvate :
12.1. Arătaţi că :
a) 9 | 1572066 b) 19 | 47063 c) 37 | 1236133 d) 8 | 453864

Soluţie.
a) Avem 1 + 5 + 7 + 2 + 0 + 6 + 6 = 27. Cum 9 | 27 atunci 9 | 1572066
b) Avem a = nr.zecilor + 2 x cifra unităţilor = 4706 + 2 x 3 = 4712
a = 471 + 2 x 2 = 475 ;
a = 47 + 2 x 5 = 57 ;
a = 5 + 2 x 7 = 19, deci 19 | 19 şi atunci 19 | 47063
c) Avem „ un număr N = anan-1an-2…a3a2a1a0 se divide cu 37 dacă anan-1an-2…a3 + a2a1a0 se divide cu 37 „.
Deci rezultă : 1236 + 133 = 1369 ; 1 + 369 = 370. Deoarece 37 | 370 rezultă că 37 | 1236133
d) Avem „ un număr N = anan-1an-2…a3a2a1a0 se divide cu 8 dacă a0 + 2a1 + 4a2 se divide cu 8 „.
Deci rezultă : a0 + 2a1 + 4a2 = 4 + 2 x 6 + 4 x 8 = 48. deoarece 8 | 48 rezultă că 8 | 453864

12.2. Câţi divizori au numerele :


a) 360 b) 442 c) 210 x 59 + 29 x 58 d) 3x x x3 ştiind că ( x, 3 ) = 1

Soluţie.
a) Avem 360 = 23 x 32 x 5 şi atunci  (360)  ( 3 + 1 )( 2 + 1 )( 1 + 1 ) = 24 de divizori
b) Avem 442 = 2 x 13 x 17 şi atunci  (442)  ( 1 + 1)( 1 + 1)( 1 + 1 ) = 8 divizori
c) Avem 210 x 59 + 29 x 58 = 29 x 58( 10 + 1 ) = 29 x 58 x 11şi atunci
 (2  5 11)  ( 9 + 1)( 8 + 1)( 1 + 1 ) = 180 de divizori
9 8

d) Avem 3x x x3 şi atunci  (3  x )  ( x + 1 )( 3 + 1 ) = 4x + 4 divizori


x 3

12.3. Care este suma divizorilor numerelor :


a) 360 b) 442 c) 23 x 52 x 11 d) 3x x x3 ştiind că ( x, 3 ) = 1
Soluţie.
231  1 321  1 511  1 15 26 24
a) Avem 360 = 23 x 32 x 5 şi atunci  (360)        15 13  6  1170
2 1 3 1 5 1 1 2 4
11 11 11
2  1 13  1 17  1 3 168 288
b) Avem 442 = 2 x 13 x 17 şi atunci  (442)        3 14 18  756
2  1 13  1 17  1 1 12 16

39
231  1 521  1 1111  1 15 124 120
c) Avem 23 x 52 x 11 şi atunci  (23  52 11)        15  3112  5580
2  1 5  1 11  1 1 4 10
3x 1  1 x 31  1 3x 1  1 x 4  1
d) Avem 3x x x3 i atunci  (3  x )     
x 3

3 1 x 1 2 x 1
(3  1)(3x  3x 1  ...  3  1) ( x 2  1)( x 2  1)
  
2 x 1
 (3x  3x 1  ...  3  1)  ( x  1)( x 2  1)

12.4. Suma a două numere naturale este 1089 iar c.m.m.d.c. al lor este 121. Să se afle
numerele.
Soluţie.
Fie x, y cele două numere.. Avem x = 121a şi y = 121b cu ( a, b ) = 1.
Rezultă :
x + y = 1089  121a + 121b = 1089  121( a + b ) = 1089  a + b = 9 de unde rezultă

a = 1 şi b = 8 sau a = 2 şi b = 7 sau a = 3 şi b = 6 sau a = 4 şi b = 5 , de unde rezultă soluţiile :

121 şi 968 sau 242 şi 847 sau 484 şi 605

12.5. Decideţi dacă numărul A = 2907100 + 2908101 + 2909102 + 2910103 este divizibil cu 10.
Soluţie.
Avem u( 2907100 ) = u( 7100 ) = u( 74 x 25 ) = 1 ; u( 2908101 ) = u( 8101 ) = u( 84 x 25 + 1 ) = 8 ;

u( 2909102 ) = u( 9102 ) = u( 94 x 25 + 2 ) = 1 ; u( 2910103 ) = u( 10103 ) = 0 .

Avem u( A ) =u( 1 + 8 + 1 + 0 ) = u( 10 ) = 0 deci 10 | A.

12.6. Să se determine a astfel încât N = 5100  2101 + a să se dividă cu 9.


Soluţie.
Avem 5100  2101 + a = 2  10100 + a = 2( 9 + 1 )100 + a = M 9 + 2 + a de unde rezultă că a = 7 .

12.7. Să se determine a şi b astfel încât N = a2345b să fie divizibil cu


a) 9 b) 8 c) 11 d) 19 e) 12

Soluţie.
a) Avem a + 2 + 3 + 4 + 5 + b = a + b + 14 şi a ≠ 0 rezultă soluţiile : a = 4 şi b = 0 sau a = 3 şi b = 1 sau
a = 2 şi b = 2 sau a = 1 şi b = 3 sau a = 9 şi b = 4 sau a = 8 şi b = 5 sau a = 7 şi b = 6 sau a = 6 şi b = 7
sau a = 5 şi b = 8 sau a = 4 şi b = 9.

b) Avem a0 + 2a1 + 4a2 =b + 10 +16 = b + 26 şi a ≠ 0 rezultă soluţiile : a Є { 1,2,3,4,5,6,7,8,9 } şi b = 6 .


c) Avem a + 3 + 5 = a + 8 şi respectiv 2 + 4 + b = b + 6 . Atunci 11 | N dacă a + 8 – ( b + 6 ) = a + 2 – b se
divide cu 11 .adică : a + 2 – b = 0  a + 2 = b de unde rezultă : a = 1 şi b = 3 sau a = 2 şi b = 4 sau
a = 3 şi b = 5 sau a = 4 şi b = 6 sau a = 5 şi b = 7 sau a = 6 şi b = 8 sau a = 7 şi b = 9 sau
a + 2 – b = 11  a – b = 9 care nu ne dă soluţii.
d) Avem „ Nr.zecilor + 2 x cifra unităţilor „ m 1 = a2345 + 2b

 fie b = 1 – rezultă m1 = a2347 ; m2 = a234 + 2 x 7 = a248 ; m3 = a24 + 2 x 8 = a40 ;


m4 = a4 care nu ne dă soluţii ;

 fie b = 2 – rezultă m1 = a2349 ; m2 = a234 + 2 x 9 = a252 ; m3 = a25 + 2 x 5 = a35 ;

m4 = a3 + 2 x 5 = a3 + 10 = ( a+ 1 )3 care nu ne dă soluţii ;
 fie b = 3 – rezultă m1 = a2351 ; m2 = a235 + 2 x 1 = a237 ; m3 = a23 + 2 x 7 = a37 ;

40
m4 = a3 + 2 x 7 = a3 + 14 = ( a+ 1 )7 care ne dă soluţia a = 4 deoarece 57 = 3 x 19 .
Deci o primă soluţie este a = 4 şi b = 3 ;
 fie b = 4 – rezultă m1 = a2353 ; m2 = a235 + 2 x 3 = a241 ; m3 = a24 + 2 x 1 = a26 ;

m4 = a2 + 2 x 6 = a2 + 12 = ( a+ 1 )4 care nu ne dă soluţii.
 fie b = 5 – rezultă m1 = a2355 ; m2 = a235 + 2 x 5 = a245 ; m3 = a24 + 2 x 5 = a34 ;

m4 = a3 + 2 x 4 = a3 + 8 = ( a+ 1 )1 care nu ne dă soluţii
 fie b = 6 – rezultă m1 = a2357 ; m2 = a235 + 2 x 7 = a239 ; m3 = a23 + 2 x 9 = a41 ;

m4 = a4 + 2 x 1 = a4 + 2 = a6 care ne dă soluţia a = 7 deoarece 76 = 4 x 19 .


Deci o a doua soluţie este a = 7 şi b = 6 ;
 fie b = 7 – rezultă m1 = a2359 ; m2 = a235 + 2 x 9 = a253 ; m3 = a25 + 2 x 3 = a31 ;

m4 = a3 + 2 x 1 = a3 + 2 = a5 care ne dă soluţia a = 9 deoarece 95 = 5 x 19 .


Deci o a doua soluţie este a = 9 şi b = 7
 fie b = 8 – rezultă m1 = a2361 ; m2 = a236 + 2 x 1 = a238 ; m3 = a23 + 2 x 8 = a39 ;

m4 = a3 + 2 x 9 = a3 + 18 = ( a+ 2 )1 care nu ne dă soluţii
 fie b = 9 – rezultă m1 = a2363 ; m2 = a236 + 2 x 3 = a242 ; m3 = a24 + 2 x 2 = a28 ;

m4 = a2 + 2 x 8 = a2 + 16 = ( a+ 1 )8 care ne dă soluţia a = 1 deoarece 38 = 2 x 19 .


Deci o a doua soluţie este a = 1 şi b = 9

d) Avem a + 2 + 3 + 4 + 5 + b = a + b + 14 şi a ≠ 0 rezultă soluţiile : a = 1 şi b = 0 sau a = 1 şi b = 3 sau


a = 2 şi b = 2 sau a = 31 şi b = 1 sau a = 1 şi b = 6 sau a = 2 şi b = 5 sau a = 3 şi b = 4 sau a = 4 şi b = 3
sau a = 5 şi b = 2 sau a = 6 şi b = 1 sau a = 7 şi b = 0 sau a = 1 şi b = 9 sau a = 2 şi b = 8 sau a =3 şi b = 7
sau a = 4 şi b = 6 sau a = 5 şi b = 5 sau a = 6 şi b = 4 sau a = 7 şi b = 3 sau a = 8 şi b = 2 sau a = 9 şi b = 1
sau a = 4 şi b = 9 sau a = 9 şi b = 4 sunt soluţii pentru care 3 | N
Din 4 | N rezultă aЄ { 1,2,3,4,5,6,7,8,9 } şi b = 6.
Din cele două condiţii rezultă soluţiile : a = 1 şi b = 6 sau a = 4 şi b = 6

12.8. Să se determine a ştiind că N = 1212a6 – 9a67 este divizibil cu 7 .


Soluţie.
Un număr N =anan-1an-2…a4a3a2a1a0 dacă anan-1an-2…a4a3 - a2a1a0 se divide cu 7.
Avem
121 – 2a6 nu dă un număr divizibil cu 7 şi analog 9 – a67 nu dă un număr divizibil cu 7.
Atunci succesiv avem : 100000 + 20000 + 1000 + 200 + 10a + 6 – 9000 – 100a – 60 – 7 = 112139 – 90a =

= 112133 + 6 – 91a + a = 7(16019 – 13a ) + a + 6 = M 7 + a + 6


Rezultă 7 | ( a + 6 ) adică a = 1 sau a = 8.

12.9. Să se determine a şi b ştiind că N = abab se divide cu 18.


Soluţie.
Din 18 | N rezultă 2 | N şi 9 | N - vezi ( 2, 9 ) = 1.
Din 2 | N rezultă b Є { 0, 2, 4, 6, 8 }. (1)
Din 9 | N avem a + b + b + a = 2( a + b ) = M 9, adică a + b = 9 sau a+ b = 18
Din a + b = 9 , soluţiile care verifică şi pe ( 1 ) sunt : a = 9 şi b = 0 ; a = 7 şi b = 2 ; a = 5 şi b = 4 ;
a = 3 şi b = 6 sau a = 1 şi b = 8.
Din a + b = 18 nu avem soluţii .

12.10.Să se determine a şi b ştiind că N = 4a5b se divide cu 12.


Soluţie.
Din 12 | N rezultă 3 | N şi 4 | N - vezi ( 3, 4 ) = 1.
Din 4 | N rezultă b Є { 2, 6 }. (1)
Din 3 | N avem 4 + a + 5 + b = a + b + 9 = M 3, adică a + b = 0 sau a+ b = 9 sau a + b = 18 .
Din a + b = 0 avem soluţia a = 0 şi b = 0 . .
Din a + b = 9 , soluţiile care verifică şi pe ( 1 ) sunt :a = 7 şi b = 2 ; a = 3 şi b = 6

41
Din a + b = 18 nu avem soluţii .

12.11.Numărul abc are cifrele distincte şi strict mai mici decât 6.

Dacă abc se divide cu ( a + b + c ) , să se arate că n

( b + c )(c + a )( a +b ) se divide cu ( a + b + c ) .
Soluţie.
Avem a, b, c < 6 .
Apoi 100a + 10b + c = k( a + b + c )
În fine : 100(b + c ) + 10( c + a ) + ( a + b ) = 100b + 100c + 10c + 10a + a + b = 101b + 110c + 11a

Apoi abc + ( b + c )(c + a )( a + b ) = 100a + 10b + c + 101b + 110c + 11a = 111a + 111b + 111c 

abc + ( b + c )( c + a )( a + b ) = 111( a + b + c ) (1)

Folosim proprietatea dacă m | p şi m | ( p + q ) rezultă m | q atunci din ( 1 ) rezultă concluzia .

12.12.Să se arate că :
a)  n Є N*, numerele de forma A = 10n + 62 se divid cu 9 ;
b)  n Є N*, numerele de forma A = 103n - 385 se divid cu 15 ;
c)  n Є N, numerele de forma A = 5n+2 + 5n+1 + 5n se divid cu 31 ;
d)  n Є N*, numerele de forma A = 6n + 2n x 3n+1 + 2nx 3n+2 se divid cu 13 ;
e)  n Є N*, numerele de forma A = 72n + 32n+1 x 23n+1 + 8n+ 1 x 9n se divid cu 15
f)  n Є N*, numerele de forma
A = 52n x 72n+1x 112n + 25n x 72nx 112n+1 - 52n+1 x 49n x 121n se divid cu 5005
g)  n Є N*, numerele de forma
A = 34n+4 x 112nx 125n + 4 x 53n+2 x 81nx 121n se divid cu 1991 .
Soluţie.
a) Avem A = 10n + 62 = 100…00 + 62 . Suma cifrelor lui este 1 + 0 + 0 + … + 0 + 6 + 2 = 9 , deci 9 | A .
de n ori de n ori
b) Avem A = 10nn - 385 = 100…00 - 385 . Suma cifrelor lui este 1 + 0 + 0 + … + 0 - 3 – 8 - 5 = - 15 , deci 3 | A .
de 3n ori de n ori
De asemenea 5 | A şi atunci cum ( 3, 5 ) = 1 rezultă 15 | A .
c) Avem succesiv : A = 5n+2 + 5n+1 + 5n = 5n( 52 + 5 + 1 ) = 31 x 5n , deci 31 | A
d) Avem succesiv : A = 6n + 3 x 6n + 9 x 6n = 6n( 1 + 3 + 9 ) = 13 x 6n , deci 13 | A
e) Avem succesiv : A = 72n + 32n+1 x 23n+1 + 8n+ 1 x 9n = 8n x 9n + 3 x 9n x 2 x 8n + 8 x 8n x 9n =
= 8n x 9n( 1 + 6 + 8 ) 15 x 8n x 9n , deci 15 | A
f) Avem 5005 = 5 x 7 x 11 x 13 apoi rezultă
A = 52n x 72n+1x 112n + 25n x 72nx 112n+1 - 52n+1 x 49n x 121n = 7 x ( 5 x 7 x 11 )2n + 11(5 x 7 x 11 )2n -
- 5 X 5 x 7 x 11 )2n = ( 7 + 11 – 5 ) 5 x 7 x 11 )2n = 13 X ( 5 x 7 x 11 )2n = 5 X 7 X 11 X 13( 5 x 7 x 11 )2n-1
deci rezultă că 5005 | A
g) Avem A = 34n+4 x 112nx 125n + 4 x 53n+2 x 81nx 121n = 81 x 92n x 112n x 53n + 100 x 53n x 92n x 112n =
= 181x 92n x 112n x 53n = 181 x 11 x 92n x 112n-1 x 53n = 1991 x 92n x 112n-1 x 53n
Rezultă 1991 | A .

12.13.Să se arate că  n Є N atunci


a) numerele de forma A = 7n x 3n +1 + 3n x 7n+1 + 7 x 21n se divid cu 17 ;
b) numerele de forma B = 5n x 7n +1 + 7n x 5n+1 + 17 x 35n se divid cu 29 ;
c) numerele de forma C = 5n+1 x 7n +1 + 5n+2 x 7n+2 + 25 x 35n se divid cu 257 ;
d) numerele de forma D = 3n+1 x 5n +1 + 3n+1 x 5n+2 + 27 x 15n se divid cu 39 ;
e) numerele de forma E = 2n+1 x 3n + 2n x 3n+1 + 6n+1 se divid cu 33.
Soluţie.
a) Avem succesiv A = 7n x 3n +1 + 3n x 7n+1 + 7 x 21n = 3 x 21n + 7 x 21n + 17 x 21n = 17 x 21n , deci 17 | A
42
b) Avem succesiv B = 5n x 7n +1 + 7n x 5n+1 + 17 x 35n = 7 x 35n + 5 x 35n + 17 x 35n = 29 x 35n , deci 29 | B
c) Avem succesiv C = 5n+1 x 7n +1 + 5n+2 x 7n+2 + 25 x 35n = 35 x 35n 25 x 49 x 35n + 25 x 35n = 1285 x 35n =
= 257 x 5 x 35n , deci 257 | C
d) Avem succesiv D = 3n+1 x 5n +1 + 3n+1 x 5n+2 + 27 x 15n = 15 x 15n + 3 x 25 x 15n + 27 x 15n =
= 117 x 15n = 39 x 3 x 15n , deci 39 | D
e) Avem succesiv E = 2n+1 x 3n + 2n x 3n+1 + 6n+1 = 2 x 6n + 3 x 6n + 6 x 6n = 11 x 6n = 11 x 3 x 2n x 3n – 1 =
= 33 x 2n x 3n – 1 , deci 33 | E.

12.14.Să se arate că numerele

A = abc2 + abc22 + abc23 + … + abc21000


Soluţie.
Evident 2 | A .
Deoarece u( 1 + abc2 + abc22 + abc23 ) = u( 1) u( 2) + u( 22 ) + u ( 23 ) = u(1 + 2 + 4 + 8 ) = u( 15 ) = 5,
Deci 5 | (1 + abc2 + abc22 + abc23 ).
Grupăm termenii câte patru şi obţinem :

A = abc2 + abc22 + abc23 + abc24 + abc25 + abc26 + abc27 + abc28 + … + abc2997 + abc2998 + abc2999 + abc21000

A = abc2(1 + abc2 + abc22 + abc23 ) + abc25(1 + abc2 + abc22 + abc23 ) + … + abc2997(1 + abc2 + abc22 + abc23 ) =

= M 5( abc2 + abc25 + … + abc2997 ) = M 5 , deci 5 | A


Din 2 | A şi 5 | A rezultă 10 | A.

12.15.Să se scrie numărul A = 2 x 5n+1 + 3 x 5n + 5 + 5n – 3x 54 ca produs de factori diferiţi de 1


şi să se arate că A este multiplu de 313.
Soluţie.
Avem A = 2 x 5 x 5n + 3 x 55 x 5n + 5 x 5n = 5n x ( 10 + 3 x 55 + 5 ) =5n x ( 15 + 3 x 55 ) = 3 x 5 x 5n x ( 1 + 54 ) =

= 3 x 5 x 626 x 5n = 2 x 3 x 313 x 5n + 1

Deci rezultă 313 | A .

12.16.Ştiind că abcde este un număr divizibil cu 41, să se arate că şi numărul bcdea este
divizibil cu 41 .
Soluţie.
Avem 41 | abcde .
Apoi rezultă
bcdea = abcde0 + a – 1000000a = 10 x abcde – 99999

Dar 99999 = 41 x 2439 , deci 41 | 99999 şi cum 41 | abcde rezultă că 41 | bcdea conform teoremei :

Dacă m | a şi m b atunci m | c = a – b

12.17.Determinaţi numerele prime a, b, c ştiind că a + b = 108 şi a – b – c = 32 .


Soluţie.
Dacă a , b, c sunt numere prime şi a + b = 108 atunci a şi b sunt numere prime impare .
Dacă a şi b sunt numere prime impare şi a – b – c = 32 rezultă a – b este număr par şi atunci c este un
număr prim par, adică singura valoare pe care o poate lua este c = 2 .
Dacă c = 2 rezultă a – b = 32 + 2 adică a – b = 34.
Din a + c = 108 şi a – b = 34 rezultă a = 71 şi respectiv b = 37 .

12.18.Determinaţi numerele prime a, b, c ştiind că 4a + 5b + 15c = 75 .


Soluţie.
Dacă a, b, c sunt numere prime rezultă 4a = 75 – 5b – 15c = 5( 15 – b – 3c ) .
Rezultă aşadar că 4 | 15 – b – 3c, adică 15 – b – 3c = 12 sau 15 –b – 3c = 8 sau 15 – b – 3c = 4
43
sau 15 – b – 3c = 0.
Din 15 – b – 3c = 12 rezultă a = 15 şi b + 3c = 3 , adică b = 3 şi c= 0 sau b = 0 şi c = 1
Din 15 – b – 3c = 8 rezultă a = 10 şi b + 3c = 7 , adică b = 7 şi c= 0 sau b = 4 şi c = 1 sau b = 7 şi c = 0
Din 15 – b – 3c = 4 rezultă a = 5 şi b + 3c = 11 , adică b = 11 şi c= 0 sau b = 8 şi c = 1 sau b = 5 şi c = 2
sau b = 2 şi c = 3

12.19.Determinaţi numerele prime a, b, c ştiind că a + 10b + 12c = 92 .


Soluţie.
Din a, b, c numere prime rezultă a este număr prim par, adică a = 2.
Pentru a = 2 rezultă 10b + 12c = 90 sau 5b + 6c = 45 , de unde obţinem :
6c = 45 – 5b , adică 6c = 5( 9 – b ) .
Rezultă 6 | ( 9 – b ) , adică b = 3 sau b = 9 .
Pentru b = 3 avem c = 5 iar pentru b = 9 avem c = 0
Rezultă soluţiile a= 2, b = 3 şi c = 5 sau a = 2, b = 9 şi c = 0.

12.20.Determinaţi numerele prime ab şi ba şi numărul natural y ştiind că

5 X ab - 7y = ba
Soluţie.
Din ab şi ba număr prim rezultă că a şi b sunt cifre numere prime .
Din relaţia dată avem : 50a + 5b – 7y = 10b + a  49a - 5b – 7y = 0 
5b = 7( y – 7a ) (1)
Din ( 1 ) rezultă b | ( y – 7 ) şi b număr prim rezultă singura soluţie b = 7 , deci 7 | ( y – 7a )adică y – 7a = 7k, de
unde obţinem y = 7(a + k ) , a cifră număr prim.
1) Pentru a = 1 şi b = 7 avem 7y = 5 x 17 – 71 = 85 – 71 = 14, deci y = 2 ;
2) Pentru a = 2 şi b = 7 avem 7y = 5 x 27 – 72 = 135 – 72 = 63, deci y = 9 ;
3) Pentru a = 3 şi b = 7 avem 7y = 5 x 37 – 73 = 185 – 73 = 112, deci y = 16 ;
4) Pentru a = 5 şi b = 7 avem 7y = 5 x 57 – 75 = 285 – 75 = 210, deci y = 30 ;
5) Pentru a = 7 şi b = 7 avem 7y = 5 x 77 – 77 = 385 – 77 = 308, deci y = 44 ;
6)
Dar ab şi ba trebuie să fie numere prime şi atunci condiţie neverificată de ab = 27, ab = 57 şi ab = 77.
Atunci singurele soluţii sunt :

ab = 17, ba = 71 şi y = 2

ab = 37, ba = 73 şi y = 16

12.21.Determinaţi numerele naturale n pentru care fiecare din numerele


n + 1, n + 3, n + 7, n + 19 şi n + 25 sunt toate numere prime .

Soluţie.
Pentru n = 1 nu avem soluţie deoarece n + 3 = 4 care nu este număr prim ;
Pentru n = 2 nu avem soluţie deoarece n + 7 = 9 care nu este număr prim ;
Pentru n = 3 nu avem soluţie deoarece n + 3 = 6 care nu este număr prim ;

Pentru n = 4 avem soluţiile n + 1 = 5, n + 3 = 7, n + 7 = 11, n + 19 = 23 şi n + 25 = 29 .


Pentru n ≥ 5 rezultă n = 5k sau n = 5k + 1 sau n = 5k + 2 sau n = 5k + 3 sau n = 5k + 4 nu mai avem soluţii
deoarece :
 pentru n = 5k rezultă : n + 25 = 5( k + 5 ) ≠ număr prim ;
 pentru n = 5k + 1 rezultă : n + 19 = 5( k + 4 ) ≠ număr prim ;
 pentru n = 5k + 2 rezultă : n + 3 = 5( k + 1 ) ≠ număr prim ;
 pentru n = 5k + 3 rezultă : n + 7 = 5( k + 2 ) ≠ număr prim ;
 pentru n = 5k + 4 rezultă : n + 1 = 5( k + 1 ) ≠ număr prim ;

12.22.Determinaţi numerele naturale n pentru care fiecare din numerele


n + 1, n2 + 3, n3 + 5, n4 + 7 şi n5 + 9 sunt simultan numere prime .

Soluţie.
44
Pentru n = 1 nu avem soluţie deoarece n2 + 3 = 4 care nu este număr prim ;
Pentru n = 2 avem soluţiile n + 1 = 3, n2 + 3 = 7, n3 + 5 = 13, n4 + 7 = 23 şi n5 + 9 = 41 .
Pentru n ≥ 3 rezultă n = 3k sau n = 3k + 1 sau n = 3k + 2 nu mai avem soluţii deoarece :

 pentru n = 3k rezultă : n2 + 3 = 3( 3k2 + 1 ) ≠ număr prim ;


 pentru n = 3k + 1 rezultă : n3 + 5 = ( 3k + 1 )3 + 5 = ( 3k )3 + 3 x ( 3k )2 + 3 x 3k + 1 + 5 =
= 3( 9k3 + 9k2 + 3k + 2) ≠ număr prim ;
 pentru n = 3k + 2 rezultă : n + 1 = 3k + 2 + 1 = 3( k + 1 ) ≠ număr prim

12.23.Fie a un număr prim şi n Є N* . Să se determine a şi n dacă este verificată


relaţia a2n – 4 = 3( 4 + 42 + 43 + … + 42011)

Soluţie.
Deoarece membrul drept este număr par rezultă că şi membrul stâng este număr par , adică a 2n – 4 este
număr par .
Dacă a2n – 4 este număr par atunci a este număr par.
Dacă a este număr par şi număr prim atunci singura soluţie este a = 2.
Din a = 2 rezultă :
22n – 4 = 3( 4 + 42 + 43 + … + 42011 ) 

22n = 3( 4 + 42 + 43 + … + 42011 ) + 4 

22n = 12 ( 1 + 4 + 42 + … + 42010 ) + 4 
4 1
2011
22 n  12   4  4(42011  1)  4  42012  4  4  42012
4 1
de unde avem
22n = 22 x 2012 , deci n = 2012..

12.24.Să se găsească toate perechile de numere naturale a şi b a căror sumă


este 87 iar diferenţa lor este un divizor al lui 87.

Soluţie.
Avem 87 = 3 x 29 .
Dacă ( a – b ) | 87 rezultă : a - b =1, a - b = 3, a - b = 29, a - b = 87
Ataşând succesiv la a – b şi a + b = 87 obţinem :

1) Din a + b = 87 şi a – b = 1, rezultă a = 44 şi b = 43 ;
2) Din a + b = 87 şi a – b = 3, rezultă a = 45 şi b = 42 ;
3) Din a + b = 87 şi a – b = 29, rezultă a = 58 şi b = 29 ;
4) Din a + b = 87 şi a – b = 87, rezultă a = 87 şi b = 0 ;

12.25.Numerele 2435, 342, 4527, împărţite la acelaşi număr dau, respectiv, resturile
35, 42, 27. Să se afle numărul la care au fost împărţite .
Soluţie.
Avem : 2435 = n x a + 35 ; 342 = n x b + 42 ; 4527 = n x c + 27 

na = 2400. ; nb = 300 ; nc = 4500


Cum c.m.m.d.c.( 2400, 300, 4500 ) = 300 rezultă n = 300
Numărul acesta nu este unic şi atunci orice număr mai mare de 35, 42 şi 27 care este şi divizor al lui 300 este
soluţie, adică : 50, 60, 75, 100, 150 , 300.

12.26.Numerele 9551, 898, 1959, împărţite la acelaşi număr dau, respectiv, resturile
31, 82, 55. Să se afle numărul cel mai mic la care au fost împărţite .
Soluţie.

45
Avem : 9551 = n x a + 31 ; 898 = n x b + 82 ; 1959 = n x c + 55 

na = 9520. ; nb = 816 ; nc = 1904


Cum c.m.m.d.c.( 9520, 816, 1904 ) = 272 rezultă n = 272
Numărul acesta nu este unic şi atunci orice număr mai mare de 31, 82 şi 55 care este şi divizor al lui 272 este
soluţie, adică : 136 care este şi cel mai mic .

12.27.Să se determine numere a şi b ştiind că :


a) ( a, b ) = 18 şi a + b = 180
b) ( a, b ) = 8 şi a x b = 1344
Soluţie.
a) Fie a = 18x şi b = 18y . Atunci avem 18( x + y ) = 180 , de unde x + y = 10.
Din x + y = 10 obţinem soluţiile :
 x = 1 şi y = 9, deci a = 18 şi y = 162
 x = 2 şi y = 8 , deci a = 36 şi b = 144
 x = 3 şi y = 7 , deci a = 54 şi b = 126
 x = 4 şi y = 6 , deci a = 72 şi b = 108
 x = 5 şi y = 5 , deci a = 90 şi b = 90
 x = 6 şi y = 4 , deci a = 108 şi b = 72
 x = 7 şi y = 3 , deci a = 126 şi b = 54
 x = 8 şi y = 2 , deci a = 144 şi b = 36
 x = 9 şi y = 1 , deci a = 162 şi b = 18
b) Fie a = 8x şi b = 8y . Atunci avem 64xy = 1344 , de unde xy = 21.
Din xy = 21 obţinem :
 x = 1 şi y = 21 , deci a = 8 şi b = 168
 x = 3 şi y = 7 , deci a = 24 şi b = 56
 x = 7 şi y = 3 , deci a = 56 şi b = 24
 x = 21 şi y = 1 , deci a = 168 şi b = 8

12.28.Să se determine numere a, b şi c ştiind că :


2 2
a) ( a, b ) = 8 şi a + b = 832
b) ( a, b ) = 28 şi [ a, b ] = 784
c) ( a, b ) = 4 , ( a, c ) = 6 , ( b, c ) = 10
d) 3a + 5b = 180 şi ( a, b ) = 10
Soluţie.
a) Fie a = 8x şi b = 8y . Atunci avem 82( x2 + y2 ) = 832 , de unde x2 + y2 = 13.
Din x2 + y2 = 13 obţinem soluţiile :
 x = 2 şi y = 3, adică a = 16 şi b = 24
 x = 3 şi y = 2, adică a = 24 şi b = 16
b) Folosim relaţia
a x b = ( a, b ) x [ a, b ], adică
ab = 28 x 784
ab = 21952
Fie a = 28x şi y = 28y. Rezultă 282xy = 21952 respectiv xy = 28
Din xy = 28 obţinem soluţiile :
 x = 1 şi y = 28, de unde a = 28 şi b = 784
 x = 2 şi y = 14, de unde a = 56 şi b = 392
 x = 4 şi y = 7, de unde a = 112 şi b = 196
 x = 7 şi y = 4, de unde a = 196 şi b = 112
 x = 28 şi y = 1, de unde a = 784 şi b = 28
c) Din ( a, b ) = 4 , ( b, c ) = 6 şi ( a, c ) = 10 rezultă

 a este multiplu de 4 şi multiplu de 10 , adică a = 20x

46
 b este multiplu de 4 şi de 6, adică b = 12y
 c este multiplu de 6 şi de 10, adică b = 30z
unde x, y, z sunt numere naturale nenule .

d) Din ( a, b ) = 10 rezultă a = 10p şi b = 1q de unde înlocuind 3a + 5b = 180 obţinem


30p + 50q = 18 , adică 3p + 5q = 18
Din 3p + 5q = 18 obţinem p = 1 şi q = 3, adică a = 10 şi b = 30 .

12.29.Fie x, y şi z numere naturale nenule şi a = 3x + 4y + 5a iar


b = 2x + 5y + 8z
b) Calculaţi a + 2b şi 3a – b
b) Dacă 7 | a atunci 7 | b ?
c) Dacă 7 | b atunci 7 | a ?
Soluţie.
a) Avem a = 3x + 4y + 5z şi b = 2x + 5y + 8z , dec rezultă a + 2b = 7x + 14y + 21z = 7( x + 2y + 3z ) şi
respectiv rezultă 3a – b = 7x + 7y + 7z = 7( x + y + z ), deci 7 | ( a + 2b ) şi 7 | ( 3a – b )
b) Din 7 | a şi 7 | ( a + 2b ) rezultă 7 | b
c) Din 7 | b şi 7 | ( 2b + a ) rezultă 7 | a

12.30.Determinaţi numerele prime a, b, c astfel încât 3a + 6b + 2c = 27 .


Soluţie.
Avem 2c = 27 – 3a – 6b  2c = 3( 9 – a – 2b ) Rezultă c | 3 şi c număr prim avem c = 3 .
Pentru c = 3 obţinem 9 – a – 2b = 2 sau a + 2b = 7 , de unde avem a = 1 şi b = 3 sau a = 3 şi b = 2
sau a = 5 şi b = 1 . Eliminând valori de 1 rezultă a = 3 şi b = 2 .

12.31.Determinaţi numărul natural A de forma A = 2a x 3b , ştiind că 2A are


cu trei divizori mai mulţi decât A , iar 3A are cu patru divizori mai mulţi
decât A
Soluţie.
Din A = 2a x 3b rezultă că A are ( a + 1 )( b + 1 ) divizori.
Atunci 2A va avea
( a + 1 )( b + 1 ) + 3 (1)
iar 3A are
( a + 1 )( b + 1 ) + 4 . (2)
Dar 2A = 2 x 2a x 3b = 2a + 1 x 3b şi atunci numărul divizorilor săi este

( a + 1 + 1 )( b + 1 ) = ( a + 2 )( b + 1 ) (3)
Iar 3A = 3 x 2a x 3b = 2a x 3b + 1 şi atunci numărul divizorilor săi este

( a + 1 )( b + 1 + 1 ) = ( a + 1 )( b + 2 ) (4)

Între ( 1 ) , ( 2 ) , ( 3 ) şi ( 4 ) avem :

( a + 2 )( b + 1 ) = ( a + 1 )( b + 1 ) + 3 de unde rezultă

( a + 2 )( b + 1 ) - ( a + 1 )( b + 1 ) = 3 (5)
şi respectiv
( a + 1 )( b + 2 ) = ( a + 1 )( b + 1 ) + 4 de unde rezultă

( a + 1 )( b + 2 ) - ( a + 1 )( b + 1 ) + 4 (6)
Din ( 5 ) şi ( 6 ) rezultă

( b + 1 ) ( a + 2 – a – 1 ) = 3  b + 1 = 3 , de unde b = 2
şi respectiv
( a + 1 )( b + 2 – b – 1 ) = 4  a + 1 = 4 , de unde a = 3

Numărul căutat este A = 23 x 32 = 72.

12.32.Determinaţi cel mai mic număr natural care are 16 divizori pozitivi care
47
are în descompunerea sa doar factorii 2, 3 şi 83.
Soluţie.
Fie N numărul . Atunci avem N = 2a x 3b x 83c iar ( a + 1 )( b + 1 )( c + 1 ) = 16 .
Dacă A este cel mai mic număr şi 2 < 3 < 83 rezultă c ≤ b ≤ a .  c + 1 ≤ b + 1 ≤ a + 1 .
Cum 16 = 2 x 2 x 2 x 2 = 2 x 2 x 4 = 1 x 2 x 8 = 1 x 1 x 16
Din a + 1 = 4 , b + 1 = 2 şi c + 1 = 2 obţinem : a = 3 şi b = c = 1 şi atunci N = 23 x 3 x 83 = 1992 .
Din a + 1 = 8 , b + 1 = 2 şi c + 1 = 1 obţinem : a = 7 şi b = 1 şi c = 0 nu convine .
Din a + 1 = 16 , b + 1 = 1 şi c + 1 = 1 obţinem : a = 15 şi b = c = 0 nu convine .

12.33.Să se determine patru numere prime a căror sumă este egală cu 34.
Soluţie.
Fie a , b, c, d cele patru numere. Avem a + b + c + d = 34 . Ştiind că toate patru numerele sunt prime,
dacă 2 ar fi număr din suma egală cu 34 atunci celelalte trei numere ar avea suma tort număr par, dar
cum alt număr par mai mare ca 2 nu mai există atunci cele trei numere sunt impare şi nu pot da o sumă pară.
Deci 2 nu este termen al sumei.
Atunci toate patru numerele sunt impare .
Din a + b + c + d = 34 avem variantele : 3, 5, 7, 11, 13, 17, 19, 23, 29, 31.
Dar d = 34 – ( a + b + c ) . valoarea minimă a sumei a + b + c = 3 + 5 + 7 = 15 şi atunci d = 34 – 15 = 19,
deci rezultă că { a, b, c, d } Є { 3, 5, 7, 11, 13, 17, 19 }
Rezultă soluţiile : { 3, 5, 7, 19 } şi { 3, 7, 11, 13 }

12.34.Să se determine numărul prim p astfel încât 3p + p3 să fie număr prim.


Soluţie.
Fie N = 3p + p3 . Pentru p = 2 avem N = 32 + 23 = 17 - număr prim .
Pentru p ≥ 3 şi p număr prim rezultă că p are forma 4k + 1 sau 4k + 3, k Є N.
Pentru n = 4k + 1 avem :
3p + p3 = 3p + 1 + p3 – 1
Dar 3 + 1 = ( 4 – 1 )p + 1 = M 4 – 1 + 1 = M 4 iar p3 – 1 = ( 4k +1 )3 – 1 = M 4 + 1 – 1 = M 4, deci
p

rezultă că suma dată de 3p + p3 = M 4 diferit de număr prim , deci singura soluţie este p = 2.

12.35.Să se determine numerele n, pЄN* astfel încât numerele n, n + 2p,


n + 2p + 1, n + 2p + 2 să fie simultan numere prime.
Soluţie.
Pentru că n este număr prim rezultă că n > 2 pentru ca şi celelalte numere să fie prime.
Atunci pentru n = 3 avem :
3, 3 + 2p , 3 + 2p + 1 , 3 + 2p + 2 vor fi numere prime pentru p = 1, adică avem soluţia
3, 5, 7, 11
sau pentru n = 3 şi p = 2 avem soluţia :
3, 7, 11, 19

3a3a3a3a
12.36.Simplificaţi fracţia : f şi determinaţi cifra a astfel încât fracţia să
7a 7a 7a 7a
fie ireductibilă după simplificare .
Soluţie.
Avem : 3a3a3a3a = 3( 107 + 105 + 103 + 10 ) + a( 106 + 104 + 102 + 1 ) =
= 30(106 + 104 + 102 + 1 ) + a( 106 + 104 + 102 + 1 ) = ( 30 + a)( 106 + 104 + 102 + 1 )

Analog 7a7a7a7a = 7( 107 + 105 + 103 + 10 ) + a( 106 + 104 + 102 + 1 ) =


= 70(106 + 104 + 102 + 1 ) + a( 106 + 104 + 102 + 1 ) = ( 70 + a)( 106 + 104 + 102 + 1 )

(30  a )(106  104  102  1) 30  a


Atunci rezultă f  
(70  a )(106  104  102  1) 70  a
Pentru a Є { 1, 3, 7, 9 } fracţia este ireductibilă după simplificare.

48
13. Probleme de perspicacitate
Problemele de perspicacitate se rezolvă folosind elementele de logică matematică
care nu se încadrează în metodele matematicii.
Rezolvarea lor presupune o provocare la un „ duel al minţii „.
Soluţia se găseşte prin logică, perspicacitate şi perseverenţă.

Exerciţii rezolvate :
13.1. La o discotecă au fost 74 de elevi, băieţi şi fete. Să se afle câţi băieţi şi câtw
fete au fost la respectiva discotecă ştiind că :
a) prima fată a dansat cu un băiat, a doua fată a dansat cu doi băieţi, a treia
cu trei băieţi ş.a.m.d. iar ultima fată a dansat cu toţi băieţii.
b) prima fată a dansat cu un băiat, a doua fată a dansat cu trei băieţi, a treia
cu 9 băieţi, a patra cu 10 băieţi, a cincea cu 11 băieţi ş.a.m.d. , fiecare
fată având cu un partener de dans mai mult decât cea precedwentă, iar
ultima fată a dansat cu toţi băieţii.
Soluţie.
a) Fie a numărul fetelor şi b numărul băieţilor
Schematic avem :
- fete : F 1 F2 F 3 F4 F5 F6 ….. Fa
- băieţi : 1 2 3 4 5 6 b
Rezultă de fapt că a = b şi atunci a = b = 74 : 2 = 37 băieţi ( fete ).

b) Fie a numărul fetelor şi b numărul băieţilor


Schematic avem :
- fete : F 1 F2 F3 F4 F5 F6 ….. Fa
- băieţi : 9 10 11 12 b
Rezultă de fapt că numărul băieţilor este cu 8 mai mare decât numărul fetelor adică
a = ( 74 – 8 ) : 2 = 64 : 2 = 32 fete iar numărul băieţilor este b = 32 + 8 = 40

14. Sume de şiruri


Problemele de genul calculului sumelor unor şiruri presupune
parcurgerea următoarelor etape :
1) determinarea numărului total de termeni ai şirului ;
2) gruparea termenilor şirului după o regulă care ni se pare avantajoasă
pentru calculul sumei
3) Aplicarea unor formule de calcul sau efectuarea succesivă a calculelor
care rezultă în urma grupării termenilor şirului.
Printre formulele de calcul sunt cele ale puterilor 1, 2 şi 3 ale numerelor
naturale, respectiv :

1) suma puterilor întâi ale primelor n numere naturale :

n
n(n  1)
S1  1  2  3  ...  n   k 
k 1 2
49
2) suma puterilor a doua ale primelor n numere naturale :

n(n  1)(2n  1)
n
S 2  12  22  32  ...  n 2   k 2 
k 1 6
3) suma puterilor a treia ale primelor n numere naturale :

2
 n(n  1) 
n
S3  1  2  3  ...  n   k  
3 3 3 3 3

k 1  2 
Pe baza acestor sume putem calcula şi alte sume cum sunt :
4) suma primelor n numere naturale impare :

n n n
n(n  1)
S 2 k 1  1  3  5  ...  (2n  1)   (2k  1) 2 k  1  2   n  n( n  1  1)  n 2
k 1 k 1 k 1 2
5) suma primelor n numere naturale pare :

n n
n(n  1)
S 2 k  2  4  6  ...  2n   2k  2 k  2   n(n  1)
k 1 k 1 2
6) suma cu produse ale primelor n numere naturale cum sunt :
6.1. suma produselor câte două

n n n n
S  1  2  2  3  3  4  ...  n( n  1)   k ( k  1)   ( k  k )   k   k 
2 2

k 1 k 1 k 1 k 1

n(n  1)(2n  1) n( n  1) n( n  1)  2n  1  n(n  1)(2n  4) n( n  1)(n  2)


     1  
6 2 2  3  6 3

6.2. suma produselor câte trei

Exerciţii rezolvate :

14.1. Calculaţi următoarele sume :


a) S = 4 + 8 + 12 + … + 2000
b) S = 10 + 11 + 12 + … + 99
c) S = 3 + 8 + 13 + 18 + 23 + … + 198
d) S = 2002 + 2001 – 2000 + 1999 - 1998 + 1997 - 1996 + 1995 -
-1994 + … + 5 - 4 + 3 - 2 + 1
e) S = 1 x 2 + 2 x 3 + 3 x 4 + …. + 100 x 101
f) S = 1 x 2 x 3 + 2 x 3 x 4 + 3 x 4 x 5 + … + 100 x 101 x 102
Soluţie.
a) Prin scoaterea în factor comun a lui 4, avem :
500  501
S = 4( 1 + 2 + 3 + ... + 500 ) =4   4  250  501  501000
2
b) Se observa ca suma face parte din suma numerelor naturale pană la 99 din care lipsesc primii 9 termeni ,
deci avem : :
99 100 9 10
S =( 1 + 2 + 3 + ... + 99 )-( 1+2+3+...+9) =   50  99  5  9  4905
2 2
50
c) Numărăm mai întâi termenii şi rezultă în total 40 de termeni deoarece şirul apare astfel :
3, 3 + 1 x 5 , 3 + 2 x 5, 3 + 3 x 5, … , 3 + 39 x 5 şi atunci avem 39 de termeni în care apr puteri ale
lui 5 mai mari ca 1 şi un termen car îl conţine doar pe 3 , deci 39 + 1 = 40
Atunci prin grupare doi câte doi termeni, obţinem :

S = ( 3 + 198 ) + ( 8 + 193 ) + ( 13 + 188 ) + …..+( 48 + 153 ) = 201 + 201 + 201 + … + 201 = 20 X 201 = 4020
de 20 de ori
sau

S = 3 + ( 3 + 5 ) + ( 3 + 2x5 ) + ( 3 + 3x5 ) + … + ( 3 + 39x5) =

= ( 3 + 3 + 3 +… + 3 ) + 5( 1 + 2 + 3 + … + 39 ) = 3 x 40 + 5 x 39 x 40 : 2 = 120 + 3900 = 4020


de 40 ori
d) Suma S = 2002 + 2001 – 2000 + 1999 - 1998 + 1997 - 1996 + 1995 - 1994 + … + 5 - 4 + 3 – 2 + 1 se observă
ca este defapt suma dintre un număr natural care este 2002 şi diferenţele a două numere succesive , deci :
S = 2002 + ( 2001 – 2000 ) + ( 1999 – 1998 ) + … + ( 5 – 4 ) + ( 3 – 2 ) + 1=

= 2002 + 1 + 1 + … + 1 = 2002 +1001 = 3003


de 1001 ori
sau

S = 2002 + ( 2001 + 1999 + … + 3 + 1 ) – ( 2000 + 1998 + … + 2 ) =

= 2002 + ( 1 + 2 + 3 + 4 + 5 + … + 2000 + 2001 ) – 2( 2 + 4 + … + 2000 ) =

= 2002 + 2001 x 2002 : 2 – 4 ( 1 + 2 + … + 1000 ) = 2002 + 2001 x 1000 – 4 x 1001 x 1000 : 2 =

= 2002 + 2001 x 1001 – 2 x 1001 x 1000 = 2002 + 1001( 2001 – 2000 ) = 2002 + 1001 = 3003

e) Din S = 1 x 2 + 2 x 3 + 3 x 4 + … + 100 x 101 se observă ca dacă multiplicăm cu 3 suma aceasta


devine succesiv :

3S = 1 x 2 x 3 + 2 x 3 x 3 + 3 x 4 x 3 + …. + 100 x 101 x 3 =

= 1 x 2 x ( 3 – 0 ) + 2 x 3 x ( 4 – 1 ) + 3 x 4 x ( 5 – 2 ) + … + 100 x 101 x ( 102 – 99 ) =

= 1 x 2 x 3 – 1 x 2 x 0 + 2 x 3 x 4 – 1 x 2 x 3 + 3 x 4 x 5 – 2 x 3 x 4 + …. + 100 x 101 x 102 – 99 x 100 x 101 =

= 100 x 101 x 102 - 1 x 2 x 0 = 100 x 101 x 102 , deci

3S = 100 x 101 x 102 de unde S = 100 x 101 x 102 : 3 = 100 x 101 x 34 = 343400

sau, folosind formulele sumelor puterilor primelor numere naturale avem :

f) Din S = 1 x 2 x 3 + 2 x 3 x 4 + 3 x 4 x 5 + … + 100 x 101 x 102 se observă ca dacă multiplicăm cu 4


suma aceasta devine succesiv :

4S = 1 x 2 x 3 x 4 + 2 x 3 x 4 x 4 + 3 x 4 x 5 x 4 + …. + 100 x 101 x 102 x 4 =

= 1 x 2 x 3 x ( 4 – 0 ) + 2 x 3 x 4 x ( 5 – 1 ) + 3 x 4 x 5 x ( 6 – 2 ) + … + 100 x 101 x 102 x ( 103 – 99 ) =

= 1 x 2 x 3 x 4 – 1 x 2 x 3 x 0 + 2 x 3 x 4 x 5 – 1 x 2 x 3 x 4 + 3 x 4 x 5 x 6 – 2 x 3 x 4 x 5 + …. +

+ 100 x 101 x 102 x 103 – 99 x 100 x 101 x 102 = 100 x 101 x 102 x 103 - 1 x 2 x 3 x 0 =

= 100 x 101 x 102 x 103, deci 4S = 100 x 101 x 102 x 103 de unde

S = 100 x 101 x 102 x 103 : 4 = 26527650

sau, folosind formulele sumelor puterilor primelor numere naturale avem :

14.2. Calculaţi următoarele sume :

51
a) S = 2003+2( 1 + 2 + 3 + … + 2002)
b) S = 1 x 2 x 3 x 4 + 2 x 3 x 4 x 5 + 3 x 4 x 5 x 6 + … +
+ 49 x 50 x 51 x 52 + 50 x 51 x 52 x 53
Soluţie.
a) Avem 1 + 2 + 3 + … + 2002 = 2002 x 2003 : 2 = 1001 x 2003. Apoi 2003 + 2 x 1001 x 2003 =
= 2003( 1+ 2 x 1001 ) = 2003 x 2003 = 20032 deci S este pătrat perfect .
b) Din S = 1 x 2 x 3 x 4 + 2 x 3 x 4 x 5 + 3 x 4 x 5 x 6 + … + 100 x 101 x 102 x 103 se observă ca dacă
multiplicăm cu 5 suma aceasta devine succesiv :

5S = 1 x 2 x 3 x 4 x 5 + 2 x 3 x 4 x 5 x 5 + 3 x 4 x 5 x 6 x 5 + …. + 100 x 101 x 102 x 103 x 5 =

=1x2x3x4x(5–0)+2x3x4x5x(6–1)+3x4x5x6x(7–2)+…+

+ 100 x 101 x 102 x 103 x ( 104 – 99 ) = 1 x 2 x 3 x 4 x 5 – 1 x 2 x 3 x 4 x 0 + 2 x 3 x 4 x 5 x 6 –

- 1 x 2 x 3 x 4 x 5 + 3 x 4 x 5 x 6 x 7 – 2 x 3 x 4 x 5 x 6 + …. + 100 x 101 x 102 x 103 x 104 –

- 99 x 100 x 101 x 102 x 103 = 100 x 101 x 102 x 103 x 104 - 1 x 2 x 3 x 4 x 0 =

= 100 x 101 x 102 x 103 x 104, deci 5S = 100 x 101 x 102 x 103 x 104 de unde

S = 100 x 101 x 102 x 103 x 104 : 5 = 2207100480

14.3. Să se rezolve ecuaţia :


( 2x + 4x + 6x + … + 1000x ) – ( x + 3x + 5x + … + 999x ) = 1000
Soluţia 1
Avem succesiv :
( 2x + 4x + 6x + … + 1000x ) – ( x + 3x + 5x + … + 999x ) = ( 2x – x ) + ( 4x – 3x ) + ( 6x – 5x ) + … +

+ ( 1000x – 999x ) = x + x + x + … + x = 500x


de 500 ori

Apoi rezultă : 500x = 1000 de unde x = 1000 : 500 = 2

Soluţia 2
Folosind formulele sumelor puterilor lui n avem :

( 2x + 4x + 6x + … + 1000x ) – ( x + 3x + 5x + … + 999x ) = x( 2 + 4 + 6 + … + 1000 ) –

- x( 1 + 3 + 5 + … + 999 ) = 500  501x  5002 x  500 x(501  500)  500 x

De unde obţinem 500x = 1000 adică x = 1000 : 500 = 2


14.4. Să se rezolve ecuaţia :
x + 3x + 5x + … + 2003x = 2x + 4x + 6x + … + 2002x + 10020
Soluţia 1
Folosim formulele S 2 k 1  1  3  5  ...  (2n  1)  n 2 şi respectiv S 2 k  2  4  6  ...  2n  n(n  1)
Avem x + 3x + 5x + … + 2003x = x(1 + 3 + 5 + … + 2003 ) = 1002 2x iar

2x + 4x + 6x + … + 2002x = x( 2 + 4 + 6 + … + 2002 ) = 1001 1002x de unde rezultă

10022x - 1001 1002x = 10020  1002( 1002 – 1001 )x = 10020  1002x = 10020 de unde obţinem

X = 10020 : 1002 = 10

Soluţia 2
Avem succesiv x + ( 3x – 2x ) + ( 5x – 4x ) + … + ( 2003x – 2002x ) = x + x + x + … + x = 1002x
1002 ori
Atunci obţinem : 1002x = 10020 de unde x = 10020 : 1002 = 10

52
14.5. Să se rezolve ecuaţia :
2x + 4x + 6x + … + 2004x = 1002  1003
Soluţie
Avem 2x( 1 + 2 + 3 + … + 1002 ) = 2x  1002  1003 : 2 = 1002  1003x de unde obţinem

1002  1003x = 1002  1003, adică x = 1002  1003 : 1002  1003 = 1

14.6. Să se rezolve ecuaţia :


x + 2x + 3x + 4x + … + nx = n + ( n + 1 ) + ( n + 2 ) + … + ( n + n )
Soluţie
n(n  1) n(n  1) x
Avem x  ( 1 + 2 + 3 + ... + n ) = x  = şi respectiv
2 2

n(n  1)
n  (n  1)  (n  2)  ...  (n  n)  (n  n  n  ...  n)  (1  2  3  ...  n)  n  n 
2
de unde obţinem :

n(n  1) x n(n  1)
 nn 
2 2
n(n  1) x  2n  n(n  1)
2

( n  1) x  2n  n  1
( n  1) x  3n  1
3n  1
x
n 1

14.7. Suma a 20 numere consecutive este 590. Aflaţi aceste numere.


Soluţie
Fie a, a + 1, a + 2, … , a + 19 cele 20 numere consecutive.
Avem : a + ( a + 1 ) + ( a + 2 ) + … + ( a + 19) = 20a + ( 1 + 2 + 3 + … + 19) = 20a + 19 x 20 : 2=

= 20a + 190
Apoi rezultă : 20a + 190 = 590 ; 20a = 590 – 190 ; 20a = 400 a = 400 : 20 = 20 , deci numerele sunt

20, 21, 22, 23, … , 39


14.8. Suma a 15 numere distincte este 120. Aflaţi aceste numere.
Soluţie
Dacă cele 15 numere ar fi consecutive, începând cu 1, avem :

1 + 2 + 3 + … + 15 = 15 x 16 : 2 = 120, deci acestea sunt o soluţie.

Căutând şi alte numere în condiţiile în care numărul total al lor să fie 15 iar suma lor sa ramana 120 nu mai
găsim soluţii, deci soluţia găsită este unică .
.
14.9. Suma a 15 numere distincte este 105. Aflaţi aceste numere.
Soluţie
Dacă cele 15 numere ar fi consecutive, începând cu 1, avem :

1 + 2 + 3 + … + 15 = 15 x 16 : 2 = 120, deci suma lor este mai mare cu 5 decât 105.

În condiţiile în care trebuie să avem 15 numere cu suma lor egală cu 105, trebuie ca să reducem 5 unităţi
53
din 5 numere pentru ca acestea să rămână distincte.
Astfel rezultă singura soluţie :
0 + 1 + 2 + 3 + 4 + … + 14 = 105

14.10. Suma a 64 numere naturale este 2100. Arătaţi că printre ele sunt cel
puţin două numere egale.
Soluţie
Avem : 1 + 2 + 3 + … + 64 = 64 x 65 : 2 = 2080 şi 2 + 3 + 4 + … + 65 = 65 x 66 : 2 – 1 = 2144
Cum 2080 < 2100 < 2144, rezultă că cele 64 de numere naturale se obţin din cele două sume reducând
unele dintre ele , de unde obţinem sigur cel puţin două numere egale . :

14.11. Fie şirul a1, a2, a3, ….., cu termenul general an = a1 + ( n – 1 )x, n Є N*.
Ştiind că a1 = 2 şi a5 = 14, se cere să afle suma primelor 100 de termeni ai
şirului.
Soluţie
Avem :
a1 = 2
a2 = a1 + ( 2 – 1 ) x = 2 + x ;
a3 = a1 + ( 3 – 1 ) x = 2 + 2x ;
a4 = a1 + ( 4 – 1 ) x = 2 + 3x ;
a5 = a1 + ( 5 – 1 ) x = 2 + 4x ;
………………………………..
an = a1 + ( n – 1 )x = 2 + ( n – 1 )x
Din a5 = 14 obţinem 2 + 4x = 14 ; 4x = 14 – 2 ; 4x = 12 ; x = 3.
Atunci şirul devine :

a1 = 2
a2 = 2 + 3 ;
a3 = 2 + 2 x 3 ;
a4 = 2 + 3 x 3 ;
a5 = 2 + 4 x 3 ;
………………………………..
an = 2 + ( n – 1 )x 3 = 3n – 3 + 2 = 3n – 1
Astfel obţinem :
Sn = 2 + ( 2 + 3 ) + ( 2 + 2 x 3 ) + 2 + ( 3 x 3 ) + … + [ 2 + 3( n- 1 ) ]

n(n  1) 3n 2  4n  3n n(3n  1)
Sn = 2n + 3[ 1 + 2 + 3 + … + ( n – 1 ) ] = 2n  3   
2 2 2
100(300  1)
S100   50  301  15050
Iar 2

15. Teorema împărţirii cu rest


Definiţie : Fie d (deîmpărţit) şi b (împărţitor) două numere naturale, cu condiţia ca b
să fie nenul. Există și sunt unice numerele întregi c (câtul) și r (restul împărțirii), astfel încât
să fie satisfăcute simultan condițiile:

* d=bxc+r

* 0 ≤ r ≤ b -1

54
d = b x c + r , unde r ≤ b -1

Deîmpărţit Cât

Împărţitor Rest

Exerciţii rezolvate :

15.1. Să se afle numerele naturale care împărţite la 13 dau restul egal cu dublul
câtului.
Soluţie
Fie N un număr natural pentru care din formula împărţirii cu rest avem : N = 13 x c + 2c , unde 2c ≤ 13 
 c≤6.
Rezultă N = 15c unde c Є { 1, 2, 3, 4, 5, 6 }, deci N Є { 15, 30, 45, 60, 75, 90 }

15.2. Fie şirul de numere naturale a1, a2, a3, ….. , cu termenul general
an = 2002n + 7 .
Să se determine numărul total al termenilor şirului, ştiind că suma acestor
termeni este produsul dintre pătratul numărului de termeni şi 1001 mărit cu
201600.
Soluţie
Dacă an = 2002n + 7 , atunci suma acestor termeni este ;

Sn = a1 + a2 + a3 + … + an = 2002( 1 + 2 + 3+ … + n ) + ( 7 + 7 + 7 + … + 7 ) =
de n ori

= 2002 x n( n + 1 ) : 2 + 7n = 1001n(n + 1 ) + 7n = 1001n 2 + 1008n

Dar Sn = 1001n2 + 201600 , de unde rezultă :

1001n2 + 1008n = 1001n2 + 201600  1008n = 201600 , deci n = 201600 : 1008 = 200 , deci , în total,
şirul are 200 de termeni .

15.3. Să se arate că printre 6 numere naturale există două a căror diferenţă se


împarte exact la 5.
Soluţie
Resturile împărţirii numerelor naturale la 5 sunt elemente ale mulţimii { 0, 1, 2, 3, 4 }, deci în total
sunt 5 resturi posibile.
Dacă se dau 6 numere rezultă că sunt două care dau acelaşi rest adică a = 5m + r şi b = 5p + r, de unde
rezultă a – b = 5m + r – 5p – r = 5( m – p ), deci 5 | ( a – b )

15.4. Împărţind un număr natural n la 68 obţinem restul 39. Ce rest obţinem dacă îl
împărţim pe n la 17 ?
Soluţie
Avem n = 68a + 39 = 4 x 17a + 34 + 5 = 17( 4a + 2 ) + 5 = 17q + 5 , deci r = 5.

15.5. Determinati suma resturilor împărţirii a 100 de numere consecutive la 19,


ştiind că primul se împarte exact la 19.
Soluţie
Avem a1 = 19p; a2 = a1 + 1 = 19p + 1 ; a3 = a1 + 2 = 19p + 2 ; … ; a18 = a1+ 17 = 19p + 17 ;

55
a19 = a1 + 18 = 19p + 18; a20 = a1 + 19 = 19p + 19 = 19( p+ 1 ) = 19q + 0
Suma resturilor primelor 19 numere succesive este :
S19 = 0 + 1 + 2 + … + 18 = 1 + 2 + 3 + … + 18 = 18 x 19 : 2= 171
În totalul celor 100 de numere , între a1 şi a100 se găsesc 5 situaţii de genul S10 şi anume :
 de la a1 la a19 ;
 de la a20 la a38 ;
 de la a39 la a57 ;
 de la a58 la a76 ;
 de la a77 la a95 ;
plus numerele a96 = a1 + 95 = = 19p + 5 x 19 = 19 ( p + 5 ) = 19r ; a 97 = a1 + 96 = 19p + 5 x 19 + 1 = 19r + 1 ;

a98 = a1 + 97 = 19p + 95 + 2 = 19p + 5 x 19 + 2 = 19( p + 5 ) + 2 = 19r + 2 ;

a99 = a1 + 98 = 19p + 95 + 3 = 19p + 5 x 19 + 3 = 19( p + 5 ) + 3 = 19r + 3 ;

a100 = a1 + 99 = 19p + 95 + 4 = 19p + 5 x 19 + 4 = 19( p + 5 ) + 4 = 19r + 4 ;

Atunci avem : S100 = 5 x 171 + 0 + 1 + 2 + 3 + 4 = 855 + 10 = 865

15.6. Într-o împărţire de numere naturale nenule , deîmpărţitul este de 33 ori mai
mare decât restul, împărţitorul este dublul câtului, iar restul este jumătate
din cât.
a) aflaţi deîmpărţitul, împarţitorul, câtul şi restul .
b) arătaţi că deîmpărţitul este un produs de două numere consecutive.
Soluţie
a) Avem : d = a x c + r, unde : d = 33r; a = 2c şi r = c : 2 .
Din cele trei relaţii rezultă succesiv : d = 33r = 33 x c : 2
Apoi rezultă : 33 x c : 2 = 2c x c + c : 2; 33c = 4c2 + c ; 4c = 32; c = 8
Deci d = 33 X 8 ; 2 = 132 ; r = c : 2 = 4 ; a = 2c = 16

b) Avem 132 = 11 x 12

15.7. Determinaţi cel mai mic număr natural care împărţit la 6 dă restul 5 şi împărţit
la 5 dă restul 4.
Soluţie
Fie N numărul căutat. Avem N = 6a + 5 = 5b + 4.
N 4
Din ultima egalitate avem : 5b = N - 4 ; b
5
N 4
b , de unde rezultă N = 5k + 4 şi atunci cel mai mic număr se obţine pentru k = 5, adică N = 29.
5

15.8. Să se afle suma numerelor naturale care împărţite la un număr mai mic ca 23
dau câtul 80, iar restul egal cu sfertul câtului.
Soluţie
Fie N numărul căutat. Rezultă : N = 80a + 20 unde a < 23 .
Dar r = 20 şi atunci r < a, deci a > 20 . Atunci obţinem 20 < a < 23 , adică a = 21 şi a = 22.
Pentru a = 21 avem N = 80 x 21 + 20 = 1680 + 20 = 1700
Pentru a = 22 avem : N = 1760 + 20 = 1780, deci 1700 + 1780 = 3480

15.9. Câte numere naturale mai mici decât 4320 împărţite la 38 dau restul 11 ?
Soluţie
Fie N < 4320 notaţia numerelor cautate.
Avem : N = 38a + 11
Rezultă : 38a + 11 < 4320  38a< 4309  a < 4309 : 38  a < 113  a ≤ 112

56
15.10. Determinaţi deîmpărţitul, Împărţitorul, câtul şi restul la o împărţire de
numere naturale, ştiind că diferenţa dintre deîmpărţit şi rest este 4.
Soluţie
Avem d = a x c + r unde r < a şi d – r = 4 .
Rezultă d = r + 4 , de unde obţinem : r + 4 = a x c + r  a x c = 4, adică a = 1 şi c = 4 sau a = 2 şi c = 2
sau a = 4 şi c = 1.
Pentru a = 1 şi c = 4 avem : r < 1 , deci r = 0 şi atunci obţinem : d = 1 x 4 + 0 = 4 ; a = 1 ; c = 4 şi r = 0 .
Pentru a = 2 şi c = 2 avem r < 2, adică r = 0 sau r = 1 .
 pentru r = 0, avem : d = 2 x 2 + 0 = 4; a = 2 ; c = 2 şi r = 0
 pentru r = 1, avem : d = 2 x 2 + 1 = 5 ; a = 2 ; c = 2 şi r = 1
Pentru a = 4 şi c = 1 avem r < 4, adică r = 0 sau r = 1 sau r = 2 sau r = 3 ..
 pentru r = 0, avem : d = 4 x 1 + 0 = 4; a = 4 ; c = 1 şi r = 0
 pentru r = 1, avem : d = 4 x 1 + 1 = 5 ; a = 4 ; c = 1 şi r = 1
 pentru r = 2, avem : d = 4 x 1 + 2 = 6, a = 4 , c = 2 şi r = 2
 pentru r = 3 , avem d = 4 x 1 + 3 = 7 , a = 4 , c = 1 şi r = 3

15.11. Determinaţi numărul care împărţit la 12 obţineţi câtul 4 iar împărţit la 19


obţineţi câtul 3.
Soluţie
Avem N = 12 x 4 + r1 cu r1 < 12 şi respectiv N = 19 x 3 + r2 cu r2 < 19.
Rezultă 48 + r1 = 57 + r2 , de unde obţinem : r1 – r2 = 9.
Cum r1 < 12 , r2 < 19 şi r1 – r2 = 9 . obţinem :
* prima soluţie : r1 = 11 şi r2 = 2 şi atunci N = 48 + 11 = 57 + 2 = 59 ;
* a doua soluţie : r1 = 10 şi r2 = 1 şi atunci N = 48 + 10 = 57 + 1 = 58 ;
* a treia soluţie : r1 = 9 şi r2 = 0 şi atunci N = 48 + 9 = 57 + 0 = 57 ;

16. Mulţimi de numere naturale


A. Definiţia şi notaţia unei mulţimi
Observaţia 1 : MULŢIMEA nu poate fi definită, dar o înţelegem ca o adunătură,
grămadă, colecţie de obiecte, fiinţe sau fenomene scrise o
singură dată. Obiectele, fiinţele sau fenomenele care aparţin
unei mulţimi se numesc ELEMENTELE MULŢIMII.

Observaţia 2 : MULŢIMEA se notează cu literele mari ale alfabetului prevăzute


eventual cu indici ( A, B, C, ……, A1, A2, … , AN, ….) iar
elementele mulţimilor se notează cu literele mici ale alfabetului,
prevăzute eventual cu indici (a, b, c, …, a1, a2, a3, …,an, … )

Observaţia 3 : MULŢIMEA care nu are nici un element se numeşte


MULŢIME VIDĂ şi se notează cu Φ; mulţimea care are un
număr finit de elemente se numeşte MULŢIME FINITĂ;
mulţimea care are o infinitate de elemente se
numeşte MULŢIME INFINITĂ

Notaţiile mulţimii :
a) sub forma scrierii tuturor elementelor mulţimii între două acolade:

57
Ex. A = { 0, 1, 2, 3, 4, 5, 6, 7, 8, 9} – mulţimea cifrelor din scrierea
numerelor în baza 10 ;
B = { a1, a2, …. , an } – mulţimea elementelor notate cu a1, a2, …, an

C = { 2, 22, 23, …. , 2n } – mulţimea puterilor naturale ale lui 2 de la


puterea 1 până la puterea n ;
Φ= { } – mulţimea vidă
Multimea numerelor naturale:
N = {0, 1, 2, . . . , n, n + 1, . . . }
N* = {1, 2, . . . , n, n + 1, . . . }

b) sub forma scrierii între două acolade a proprietăţii sau proprietăţilor


comune a tuturor elementelor, adică astfel :
A = { x | P(x) } - A este mulţimea tuturor elementelor x care au
proprietatea P( x ) sau
B = { x | P1(x), P2(x) } - B este mulţimea tuturor elementelor x
care au proprietăţile P1( x ) şi P2( x ) .

c) sub forma unei diagrame în care sunt trecute toate elementele mulţimii:

A
.a .b - mulţimea A este reprezentată într-o diagramă
sub formă de cerc

.c .d

B. Operaţii cu mulţimi
a) REUNIUNEA MULŢIMILOR ( U ) - prin reuniunea a două sau mai multe
mulţime se obţine o nouă mulţime formată din elementele comune şi
necomune, luate o singura dată .

Ex . 1) Fie A = { 1,3,4,5,6 } şi B = { 2,3,5,6,8,9, }


Rezultă :
C = A U B = { 1,3,4,5,6 } U { 2,3,5,6,8,9, } = { 1,2,3,4, 5,6,8,9, }
Proprietăţile reuniunii :
1. Reflexivitatea :  A şi B mulţimi, avem A U B = B U A.
2. Asociativitatea :  A , B şi C mulţimi, avem (A U B) U C = A U (B U
3. Idempotenţa :  A o mulţime, avem : A U A = A
4.  A şi B mulţimi, avem A  A U B şi B  A U B.
.
b) INTERSECŢIA MULŢIMILOR (  )– prin intersecţia a două sau mai
multe
mulţime se obţine o nouă mulţime formată din elementele comune,
luate o singura dată .
58
Ex . 1) Fie A = { 1,3,4,5,6 } şi B = { 2,3,5,6,8,9, }
Rezultă :
C = A  B = { 1,3,4,5,6 }  { 2,3,5,6,8,9, } = { 3, 5,6 }
Observaţie : Dacă A  B = Φ atunci A şi B se numesc mulţimi
disjuncte .
Proprietăţile intersecţiei :
1. Comutativitatea :  A şi B mulţimi, avem A  B = B  A.
2. Asociativitatea :  A , B şi C mulţimi, avem
(A  B)  C = A  (B  C).
3. Idempotenţa :  A o mulţime, avem : A  A = A
4.  A, avem A  Φ = Φ  A = Φ
5.  A şi B mulţimi, avem A  B  A şi A  B  B
6. Distribuţia intersecţiei faţă de reuniune :  A , B şi C mulţimi, avem
( A U B)  C =( A  C ) U ( B  C )
7. Distribuţia reuniunii faţă de intersecţiee :  A , B şi C mulţimi, avem
( A  B) U C =( A U C )  ( B U C )
8. Absorbţia :  A şi B mulţimi, avem A  ( A U B ) = A şi respectiv
AU(A B)=A
c) DIFERENŢA MULŢIMILOR ( „ – „ ) - din C = A – B rezultă că C este
formată din elementele lui A rămase după ce eliminăm din A
elementele
care sunt comune şi lui B.
Ex . 1) Fie A = { 1,3,4,5,6 } şi B = { 2,3,5,6,8,9, }
Rezultă :
C = A - B = { 1,3,4,5,6 } - { 2,3,5,6,8,9, } = { 1, 4 } iar

D = B – A ={ 2,3,5,6,8,9, } - { 1,3,4,5,6 } ={ 2, 8, 9 }

d) DIFERENŢA SIMETRICĂ (Δ )– adică C = A Δ B, unde


AΔB=(A–B)U(B–A)
Ex . 1) Fie A = { 1,3,4,5,6 } şi B = { 2,3,5,6,8,9, }
Rezultă :
C = A - B = { 1,3,4,5,6 } - { 2,3,5,6,8,9, } = { 1, 4 } iar

D = B – A ={ 2,3,5,6,8,9, } - { 1,3,4,5,6 } = { 2, 8, 9 }
şi atunci
A Δ B = ( A – B ) U ( B – A ) = { 1, 4 } U { 2, 8, 9 } =
= { 1, 2, 4, 8, 9 }

e) PRODUSUL CARTEZIAN ( x ) – adică C = A x B , unde


A x B = {( x, y ) | x Є A şi y Є B }
Ex . 1) Fie A = { 1,3,4,5,6 } şi B = { 2,3,5,6,8,9, }
Rezultă :
A x B = {(1,2),(1,3),(1,5),(1,6),(1,8),(1,9),(3,2),(3,3),(3,5),(3,6),(3,8),(3,9),
(4,2),(4,3),(4,5),(4,6),(4,8),(4,9),(5,2),(5,3),(5,5),(5,6),(5,8),(5,9),
59
(6,2),(6,3),(6,5),(6,6),(6,8),(6,9) }

C. Cardinalul unei mulţimi


a) Relaţiile dintre mulţimi şi elementele acesteia :
Dacă A este o mulţime şi n este un element al acestei mulţimi, spunem
că n aparţine mulţimii A şi notăm aceasta prin : n Є A ( n aparţine lui A )
Ex. Fie A = { 1,3,4,5,6 }. Avem 1 Є A; 3 Є A; 4 Є A etc.
Dacă A este o mulţime şi m nu este un element al acestei mulţimi, spunem
că m nu aparţine mulţimii A şi notăm aceasta prin : m Є A ( m nu aparţine
lui A )
Ex. Fie A = { 1,3,4,5,6 }. Avem 2 Є A; 7 Є A; 9 Є A etc.

b) Relaţiile dintre mulţimi


1) INCLUZIUNE MULŢIMILOR (  sau  sau  sau  ) - spunem că o
mulţime A este inclusă într-o mulţime B, dacă toate elementele
lui A sunt şi elemente ale lui B
Proprietăţile incluziunii :
1. Reflexivitatea :  mulţimea A avem A = A .
2. Antisimetria : Dacă A  B şi B  A atunci A = B.
3. Tranzitivitatea : Dacă A  B şi B  C atunci A  C.
4.  mulţimea A rezultă Φ  B .
2) COMPLEMENTARA UNEI MULŢIMI ÎN RAPORT CU O ALTĂ
MULŢIME ( CAE)– prin complementara lui A în raport
cu E înţelegem o mulţime formată din elementele lui A care
nu aparţin şi lui E.
Proprietăţile complementarei :
1. CAA = Φ.
2. Principiul excluderii terţiului : A U CEA = A
3. Principiul necontradicţiei : A  CEA = Φ
4.  mulţimea A rezultă Φ  E .

3) EGALITAEA A DOUĂ SAU MAI MULTE MULŢIMI


Două mulţimi A şi B sunt egale dacă conţin aceleaşi elemente,
adică atât numărul elementelor cât şi proprietăţile acestora
sunt aceleaşi.
Ex. Fie A = { 1,3,5,7, 8, 9 } şi B = { 1,3, 5,7, 8,9 }
Rezultă : A = B sau B = A.
Observaţie : Două sau mai multe mulţimi care au acelaşi număr de
elemente, dar proprietăţile acestora diferă de la o mulţime
la alta atunci acestea se numesc ECHIVALENTE.
Ex. Fie A = { 1,3,5,7, 8, 9 } şi B = { a1, a2, a3, a4, a5, a6 }
Rezultă : A - are 6 elemente şi B are 6 elemente dar
elementele lui A sunt numere naturale iar elementele
lui B nu sunt numere naturale .Atunci A este
echivalentă cu B şi notăm aceasta astfel : A  B
60
c) Cardinalul unei mulţimi
Definiţie = numim cardinalul unei mulţimi M numărul elementelor acelei
mulţimi şi notăm aceasta astfel card M.
Ex. Fie A = { 1,3,5,7, 8, 9 } rezultă card A = 6
Fie A = { a1, a2, a3, a4, a5, a6, a7, a8 } rezultă card B = 8
Fie Φ mulţimea vidă. Rezultă card Φ = 0
Fie N = { 0, 1, 2, 3, … , n, …. } rezultă card N = 

Observaţie :
Dacă două mulţimi au acelaşi cardinal, atunci acestea sunt :
1) EGALE ( = ) dacă elementele acestora sunt identice, adică au
aceleaşi proprietăţi sau ;
2) ECHIVALENTE (  ) adică au acelaşi număr de elemente dar
proprietăţile elementelor celor două mulţimi sunt diferite .
3) Avem card ( A U B ) = card A + card B – card ( A  B ) şi
respectiv :
card ( A U B U C ) = card A + card B + card C – card ( A  B) –
- card ( B  C) – card (C  A) + card( A  B  C )

Exerciţii rezolvate

16.1. Se consideră mulţimile :


A1 = { 2 }, A2 = { 2, 4 }, A3 = { 2, 4, 6 }, …., An = { 2, 4, 6, …, 2n } , n Є N.
a) Determinaţi A7 ;
b) Demonstraţi că pentru niciuna dintre mulţimi suma
elementelor nu poate fi pătrat perfect ;
c) Aflaţi n Є N* pentru care suma elementelor este 2550.
Soluţie
a) Avem A7 = { 2, 4, 6, …, 2 x 7 } = { 2, 4, 6, 8, 10, 12, 14 }
b) Calculăm suma elementelor lui An şi obţinem : 2 + 4 + 6 + … + 2n = 2( 1 + 2 + 3 + … + n ) = n(n + 1 ) şi
deoarece n2 < n( n + 1 ) < ( n + 1 )2 rezultă concluzia .
c) Avem n( n + 1 ) = 2550 = 10 x 255 = 50 x 51 , deci n = 50

16.2. La teza de matematică, elevii clasei a V – a au avut de rezolvat 3 probleme.


După corectarea tezei, profesorul a constatat că :
 16 elevi au rezolvat corect problema 1 ;
 14 elevi au rezolvat corect problema 2 ;
 12 elevi au rezolvat corect problema 3 ;
 7 elevi au rezolvat corect problemele 1 şi 2 ;
 6 elevi au rezolvat corect problemele 1 şi 3 ;
 5 elevi au rezolvat corect problemele 2 şi 3 ;
 Fiecare elev dintre cie 26 ai clasei au rezolvat corect cel puţin o
problemă .
Aflaţi câţi elevi au rezolvat corect toate cele trei probleme .
61
Soluţie
Fie A = mulţimea elevilor care au rezolvat corect problema 1 ; B = mulţimea elevilor care au rezolvat
corect problema 2 ; C = mulţimea elevilor care au rezolvat corect problema 3 .
Avem : card A = 16; card B = 14 şi card C = 12; card( A  B) = 7 ; card( B  C) = 5; card( C  A) = 6
Mulţimea elevilor din clasă care au dat teza este A U B U C iar mulţimea elevilor care au rezolvat toate cele
trei probleme este A  B  C rezultă :
card ( A U B U C ) = 26
card ( A U B U C ) = card A + card B + card C – card ( A  B) – card ( B  C) – card (C  A) + card( A  B  C )
de unde obţinem :
card( A  B  C ) = card ( A U B U C ) – car A – card B – card C + card ( A  B) + card ( B  C) + card ( C  A) =
= 26 – 16 – 14 – 12 + 7 + 5 + 6 = 2

16.3. Se dau mulţimile : A = { ab Є N / ab ≤ 3( a + b ) } şi

B = { xy Є N / x1 + 1y – 4y = 3( x + y ) }
Să se determine mulţimile A  B, A U B , A – B .

Soluţie :
Avem succesiv : ab ≤ 3( a + b )  10 a + b ≤ 3a + 3b  7a ≤ 2b cu a, b ≤ 9.
Pentru a = 1, avem 7 ≤ 2b  b Є { 4, 5, 6, 7, 8, 9,}, deci rezultă o prima solutie

ab Є{ 14, 15, 16, 17, 18, 19,},

Pentru a = 2, avem 14 ≤ 2b  b Є { 7, 8, 9 }, deci rezultă o a doua solutie

ab Є{ 27, 28, 29}

Pentru a ≥ 3, avem 7a ≥ 2b pentru oricare b Є { 1, 2, 3, 4, 5, 6, 7, 8, 9 }, deci rezultă


că nu avem soluţii .
Aşadar obtinem : A = { 14, 15, 16, 17, 18, 19, 27, 28, 29 }

Analog din xy Є N / x1 + 1y – 4y = 3( x + y ) rezultă 10x + 1 + 10 + y – 4y = 3x + 3y 

 7x – 6y + 11 = 0 – adică o ecuaţie diofantică în x şi y din care rezultă :


6y – 7x = 11 (1)

Pentru x = 1 avem 6y – 7 = 11  6y = 18, de unde y = 3, adică obtinem o prima

solutie xy = 13 .

Pentru x = 2 avem 6y – 14 = 11  6y = 25, de unde rezulta ca nu avem solutii.

Pentru x = 3 avem 6y – 21 = 11  6y = 32, de unde rezulta ca nu avem solutii.

Pentru x = 4 avem 6y – 28 = 11  6y = 39, de unde rezulta ca nu avem solutii.

Pentru x = 5 avem 6y – 35 = 11  6y = 46, de unde rezulta ca nu avem solutii.

Pentru x = 6 avem 6y – 42 = 11  6y = 53, de unde rezulta ca nu avem solutii.

Pentru x ≥ 7 rezulta 7x ≥ 49 si respectiv 6y ≥ 60, deci y ≥ 10 care nu ne da solutii..

Asadar avem B = { 13 }

Atunci obtinem : A  B = { 14, 15, 16, 17, 18, 19, 27, 28, 29 }  { 13 } = Ø ;

A U B = { 14, 15, 16, 17, 18, 19, 27, 28, 29 } U { 13 } =

= { 13, 14, 15, 16, 17, 18, 19, 27, 28, 29 }

62
A – B = { 14, 15, 16, 17, 18, 19, 27, 28, 29 } - { 13 } =

= { 14, 15, 16, 17, 18, 19, 27, 28, 29 } = A

16.4. Fie mulţimea A = { 1, 2, 3, …, 2003 } . Se şterg, la întâmplare, două numere


ale mulţimii M şi se trec în locul lor diferenţa dintre numărul mai mare şi
numărul mai mic. Se repetă operaţiunea până când mulţimea M modificată
rămâne cu un singur element. Se cere paritatea acestui ultim element.

Soluţie :
Mulţimea M are 1007 numere impare şi 1006 numere pare. Rezultă
a) dacă se şterg iniţial două numere impare şi se pune în loc diferenţa lor atunci aceasta este un număr par, deci
rămân în mulţime tot un număr impar de numere impare ;
b) dacă se şterg iniţial două numere pare şi se pune în loc diferenţa lor atunci aceasta este un număr par, deci
rămân în mulţime tot un număr impar de numere impare ;
c) dacă se şterg iniţial un număr par şi un număr impar şi se pune în loc diferenţa lor atunci aceasta este un
număr impar, deci rămân în mulţime tot un număr impar de numere impare ;
Asadar ultimul element rămas în M modificată va fi un număr impar.

16.5. Problema nr. E,14281 – GM nr. 1 – / pg.42


Se consideră mulţimea A = { x3 + y3 | x, y Є N, x ≠ y }.
a) Verificaţi dacă 2828 ЄA şi 19721972 ЄA
b) Demonstraţi că A conţine o infinitate de elemente de forma n n cu n număr
natural nenul.
Soluţie
a) Avem succesiv ;
2828 = 28  2827 = ( 13 + 33 )  2827 = ( 13 + 33 )  ( 289 )3 = 13  ( 289 )3 + 33  ( 289 )3 =

= ( 289 )3 + ( 3  289 )3 = x3 + y3 Є A, unde x = 289 şi y = 3  289

Deoarece 1972 = 64 x 28 = 43 x 28 = 43( 13 + 33 ) = 43 + 123 analog obţinem :

17921792= 1792  17921791 = ( 43 + 123 )  ( 1792597 )3

Fie a = 1792597 . Atunci avem :

17921972 = ( 43 + 123 )  ( 1792597 )3 =( 43 + 123 )  a3 = 43  a3 + 123  a3 =

= ( 4  a )3 + ( 12  a )3 = x3 + y3 Є A, unde x = 4a, y = 12a şi a = 1792597

b) Observăm la pct.a) că în expresia nn dacă facem pe n = 28 obţinem primul caz, respectiv


2828 = x3 + y3 Є A , iar dacă facem pe n = 43  28 = 64  28 =1792 , obţinem de asemenea că
17921792 = x3 + y3 Є A .
Rezultă aşadar că putem verifica dacă relaţia se păstrează pentru expresii de genul
(4k)3  28 , adică produsul dintre puterile lui 4 şi 28.

Fie în expresia nn respectiv n = (4k)3  28 = (4k)3  ( 13 + 33 ) = (4k)3 + (3  4k)3 , deci

n = (4k)3 + (3  4k)3 , unde k Є N . (1)

Avem apoi :

63
nn = n  n n – 1 (2)

Pentru valoarea lui n din relaţia ( 1 ) , calculăm pe n – 1 , adică :

n – 1 = (4k)3 + (3  4k)3 – 1 = (3  4k) + (4k)3 – 1 = (3  4k) + (43k – 1 ) (3)

Dar 43k – 1 = ( 4 – 1 ) ( 43k – 1 + 43k – 2 + 43k – 3 + … + 42 + 4 + 1 ) =

= 3(43k – 1 + 43k – 2 + 43k – 3 + … + 42 + 4 + 1 ) = 3p , unde

p = 43k – 1 + 43k – 2 + 43k – 3 + … + 42 + 4 + 1

( a se vedea că în descompunerea lui 43k – 1 am aplicate cunoscuta formulă

qm – 1 = ( q – 1 )( qm – 1 + qm – 2 + qm – 3 + … + q2 + q + 1 )

Dacă n - 1 = 3p , atunci avem nn-1 = n3p şi folosind relaţiile ( 1 ) şi ( 2 ) avem :

nn = n  nn – 1 = n  n3p = [(4k)3 + (3  4k)3]  n3p = (4k)3  (np)3 + (3  4k)3  (np)3 =

= (4k  np)3 + (3  4k  np)3 = x3 + y3 Є A , unde

x = 4k  np ; y = 3  4k  np ; n = (4k)3 + (3  4k)3 ;

p = 43k – 1 + 43k – 2 + 43k – 3 + … + 42 + 4 + 1 şi k Є N

Cum k Є N rezultă că există o infinitate de soluţii de forma n = (4k)3 + (3  4k)3 astfel


încât să avem nn Є A .

17. Probleme pentru gimnaziu


1. Calculaţi sumele :
S = 85 + 985 + 9985 + 99985 + … + 99…9985
2011 ori
T = 17 + 197 + 1997 + 19997 + … + 199… 97
2011 ori

Soluţie
Avem succesiv :
S = 85 + 985 + 9985 + 99985 + … + 99…985 =
2011 ori
= ( 100 – 15 ) + ( 1000 – 15 ) + ( 10000 – 15 ) + … + ( 100…0 – 15 ) =
2013 ori
= 111…100 – 15 x 2012 = 11 … 111080920
2008 ori
Analog avem succesiv :
T = 17 + 197 + 1997 + 19997 + … + 199… 97 =
2011 ori

= 85 : 5 + 985 : 5 + 9985 : 5 + 99985 : 5 + …. + 99 … 985 : 5 =


2011 ori

= (85 + 985 + 9985 + 99985 + … + 99…985 ) : 5 = S : 5 =


64
2011 ori
= 22 … 2216184
2008 ori

2. Fie numărul 22012 = x1x2…xn şi 52012 = y1y2 … ym scrise în baza 10.


Câte cifre are numărul
A = x1x2 … xny1y2 … ym ?
Soluţie
Numărul A are evident n + m cifre .
Avem succesiv :
 din 22012 = x1x2 … xn avem : 10n – 1 < 22012 < 10n (1)

 Din 52012 = y1y2 … ym avem : 10m – 1 < 52012 < 10m ( 2 )


 Între ( 1 ) şi ( 2 ), prin înmulţire, membru cu membru, avem :
 10n + m – 2 < 102012 < 10n + m de unde rezultă n + m – 2 < 2012 < n + m
 Din ultima inegalitate rezultă n + m – 1 = 2012 , n + m = 2013 , deci numărul A are
2013 cifre.

3. Fie A = 22 … 255 … 5. Demonstraţi că A se divide cu 49 dacă şi


n ori n ori
numai dacă n este multiplu de 6 .

Soluţie
Avem A = 11 …1 x ( 2x10n + 5 )
de n ori
1) Vom arăta că dacă A se divide cu 49 atunci n se divide cu 6 :
Avem 11 … 1 = 111111x10n – 6 + 111111x10n – 12 + … + 111111x106 + B, unde
de n ori
BЄ { 1, 11, 111, 1111, 11111, 111111 }.
Cum 111111 = 7 x 15873 înseamnă că toţi termenii, cu excepţia lui B, se divid cu 7
Pentru ca 11 … 1 să se dividă cu 7 trebuie ca B să se dividă cu 7, adică
de n ori
B = 111111.
Aşadar 11 … 1 se divide cu 7 dacă n = M6, adică n = 6k.
de n ori
Pentru n = 6k , avem :
2x10n + 5 = 2x106k + 5 = 2 x( 1000000 )k + 5 = 2x( 7x142875 + 1 )k + 5 =

= M 7 + 2x1k + 5 = M7 + 7 = M7 şi atunci rezultă că A se divide cu 49 .


2) Vom arăta că dacă n = 6k, atunci A se divide cu 49 :
Avem
A = 11 …1 x( 2 x 10n + 5 ) şi atunci analog 2 x 10n + 5 se divide cu 7 , iar
de n ori
11 … 1 = 111111x106k-6 + 111111x106k – 12 + … + 111111 =
de n ori
= 111111( 106k – 6 + 106k – 12 + … + 1 ) =
= 7 x 15873 x ( 106k – 6 + 106k – 12 + … + 1 )
deci A se divide cu 7 x 7 = 49.

65
372372...372379
4. Se dă fracţia F  în care 372 şi 496 se repetă de acelaşi
496496...496505
număr de ori. Arătaţi că F este ireductibilă.

Soluţie
Fie n numărul de repetări ale celor două grupe de numere. Rezultă :

372n  379
F
496n  505
Dacă d este cel mai mare divizor comun al numărătorului şi numitorului rezultă :

d | 372n + 379 şi d | 496n + 505.

Atunci rezultă că d divide şi orice combinaţie lineară ale numărătorului şi numitorului,


adică are loc relaţia
d | a( 372n + 379 ) + y( 496n + 505 ).
Pentru a = 4 şi b = - 3 obţinem :

d | 4( 372n + 379 ) – 3( 496n + 505 ) adică

d | 1488n + 1516 – 1488n – 1515 = 1

Din d | 1 rezultă că fracţia este ireductibilă.

5. Determinaţi numerele naturale x, y, z ştiind că :

x y z
x2 + y2 + z2 = 179 şi  
x  3 y  7 z  11
Soluţie
x  3 y  7 z  11
Din a doua relaţie, obţinem relaţia :   care devine
x y z

3 7 11 3 7 11
1  1  1 sau    k de unde :
x y z x y z
x = 3k, y = 7k şi z = 11k.

9k2 + 49k2 + 121k2 = 179 sau

k2 = 1 de unde k = 1 şi atunci x = 3, y = 7 şi z = 11.

:
3
6. Pentru n număr natural, n ≥ 2, definim an  10n  n  2
Să se arate că an se poate scrie ca sumă a patru cuburi perfecte .

Soluţie
Se ştie că 100 = 1 + 8 + 27 + 64 = 13 + 23 + 33 + 43’
Apoi avem :
n3 – n + 2 = (n – 1 )n( n + 1 ) + 2 = 3k + 2 , deoarece produsul a trei
numere consecutive este multiplu de 3.
Astfel avem :

66
3
n2
an  10n = 103k + 2 = 103k x 102 = 100 x ( 10k )3 = (13 + 23 + 33 + 43’ ) x ( 10k )3 =

= ( 10k )3 + ( 2x10k )3 + ( 3x10k )3 + ( 4x10k )3

7. Fie a1, a2, a3, … , a2011 numere naturale impare.


Demonstraţi că
A  a12  a22  ...  a2011
2

este un număr iraţional .

Soluţie
Se ştie că dacă ak , k Є { 1, 2, 3, …, 2011 } este număr impar, atunci a k = M4 ±1
iar ak  M 4  1
2

Apoi avem : a1  a2  ...  a2011  M 4  1  1  ...  1  M 4  2011  M 4  3


2 2 2

Cum nu există pătrate de forma M4 + 3, rezultă că A este un număr iraţional.

(24 n  22 n 3  15) n 1
8. Arătaţi ca numărul este număr natural pentru
(3  4n  9) n
orice n natural .

Soluţie
Notăm cu F fracţia dată şi avem succesiv :

n
(24 n  22 n 3  15) n 2 n 3  24 n  22 n 3  15  2 n 3
F  (2 4n
 2  15)     (2  2
4n
 15)
(3  4n  9) n  3  4 n
 9 
n
 24 n  8  22n  16  1  2 n 3
F    (2  2
4n
 15)
 3  (4 n
 3) 
2 n 3
Cum paranteza rotundă F2  (2  2  15) este un număr natural, rămâne de arătat
4n

că paranteza dreaptă este un număr natural.


Avem astfel, succesiv :

n n n
 24 n  8  22 n  16  1   (22 n  4) 2  1  (2 2 n  4  1)(2 2 n  4  1) 
F1      3  (4n  3)     sau
 3  (4n  3)     3  (2 2 n  3) 
n n n
 (22 n  5)(22 n  3)   2 2 n  5   4n  5 
F1       
 3  (2  3)   3   3 
2n

Dar
4n  5  (3  1) n  5  M 3  1  5  M 3  6  M 3
4n  5
şi atunci este număr natural, deci F1 este număr natural şi
3

F = F1 x F2 este număr natural.

9. Arătaţi ca, pentru orice număr natural nenul n, există un cub perfect
cu exact n cifre de zero, în scrierea sa în baza zece.

67
Soluţie
Fie A numărul căutat. Avem situaţiile :
1) pentru n = 3k, rezultă : A = (10 )3k = (10k)3 = 100 … 0
de 3k ori

2) Pentru n = 3k + 1, rezultă : A = 103kx163 =( 10kx16 )3 care este cub perfect:.


Dar A = 103kx163 = 100 … 0 x 4096 = 409600 … 0
de 3k ori
deci A are 3k + 1 zerouri .

3) Pentru n = 3k + 2, rezultă : A = 103kx633 =( 10kx63 )3 care este cub perfect:.


Dar A = 103kx633 = 100 … 0 x 250047 = 25004700 … 0
de 3k ori
deci A are 3k + 2 zerouri .

10n  9
10. Arătaţi ca, pentru orice număr natural nenul n, fracţia este
15n  1
ireductibilă.

Soluţie
Fie d cel mai mare divizor comun al numărătorului şi numitorului.
Atunci d | ( 10n + 9 ) şi d | ( 15n + 1 ).
Dar d | 3(10n + 9 ) -2(15n + 1 ) = 25, adică d | 25, deci d Є {1, 5, 25 }.
Dacă d = 5, se observă că 5 nu divide 10n + 9 , deci atunci nici 25 nu divide 10n + 9.
Rezultă aşadar n =1 , deci fracţia este ireductibilă.:

11. Arătaţi ca, dacă a, b, c sunt laturile unui triunghi, atunci are loc
dubla inegalitate :
9 1 1 1 5
   
2(a  b  c ) a  b b  c c  a a  b  c

Soluţie
Inegalitatea din stânga o demonstrăm astfel :
Avem :.
1 1 1 12 12 12
    
ab bc ca a b bc ca
Şi folosind inegalitatea Cauchy – Buniakovski , avem :
12 12 12 (1  1  1) 2 9
   
a  b b  c c  a a  b  b  c  c  a 2(a  b  c )

Inegalitatea din dreapta o demonstrăm astfel :


1 1 1 5
   echivalentă cu
a b b c c  a a b c

a bc a b c a b c
  5
ab bc ca

Din ultima egalitate, obţinem succesiv :

c a b
1 1 1 5
ab bc ca

68
c a b
  2 (0)
ab bc ca

Cum a, b, c sunt laturile unui triunghi, rezultă :


a<b+c;b<c+a;c<a+b
de unde avem :

a + b +c < 2( b + c )
a + b +c < 2( c + a )
a + b +c < 2( a + b )
Din fiecare inegalitate , avem :

1 2 1 2 1 2
 ;  ;  sau
bc abc ca abc ab abc

a 2a
 (1)
bc abc

b 2b
 (2)
bc abc

c 2c
 (3)
bc abc
Adunând relaţiile ( 1 ), ( 2 ) şi ( 3 ) obţinem :

a b c 2(a  b  c)
    2 , adică inegalitatea ( 0 ).
bc c a a b a bc

12. Calculaţi partea întreagă a numărului :


1 1 1 1
a    ... 
2 1 3 2 4 3 100 99
Soluţie
Folosind notaţiile la modul general, avem :
1 k 1 1   1 1   1 1 
  k     k      
(k  1) k k (k  1)  k k 1   k k 1   k k 1 
 k k   1 1   k   1 1   1 1 
          1  k  1       2  
 k k 1   k k 1     k k 1   k k 1 
Astfel avem :
 1 1 1 1 1 1 1 1 
a  2       ...    , adică
 1 2 2 3 3 4 99 100 

 1 9
a  2  1    2   1,8
 10  10
Deci rezultă :  a    1,8  1

69
13. Să se determine numerele naturale a şi b astfel încât a 2 + 3a =b2.

Soluţie
Deoarece a2 < a2 + 3a < ( a + 2 )2 şi a2 + 3a trebuie să fie pătrat perfect rezultă :
a2 + 3a = ( a + 1 )2 de unde obţinem succesiv :
a2 + 3a = a2 + 2a + 1
3a – 2a = 1
a = 1 şi atunci b2 = 12 + 3 = 4 , deci b = 2

14. Determinaţi tripletele ( x, y, z ) de numere naturale pentru care


5
fracţia x  2 y  3z
este supraunitară.

Soluţie
Trebuie ca x + 2y + 3z < 5 şi atunci rezultă :
* pentru x = 0 , avem 2y + 3z < 5, de unde rezultă : y =0 si z = 1 sau y = 1 şi z = 0
sau y = 2 şi z = 0. Aşadar am obţinut soluţiile : ( 0, 0, 1 ), ( 0, 1, 0 ), ( 0, 2, 0 ).
* pentru x = 1, avem : 2y + 3z < 4, de unde rezultă : y = 0 şi z = 1 sau y = 1 şi z = 0
Aşadar am obţinut soluţiile : ( 1, 0, 1 ), ( 1, 1, 0 ) .
* pentru x = 2, avem : 2y + 3z < 3, de unde rezultă : y = 0 şi z = 0 sau y = 1 şi z = 0
Aşadar am obţinut soluţiile : ( 2, 0, 0 ), ( 2, 1, 0 ) .
* pentru x = 3, avem : 2y + 3z < 2, de unde rezultă : y = 0 şi z = 0
Aşadar am obţinut soluţia : ( 3, 0, 0 ).

15. Aflaţi ultimele două cifre ale câtului împărţirii numărului


7n+1 + 4x7n +17 la 11, ştiind că n este multiplu de 4.

Soluţie
Fie a = 7n+1 + 4x7n +17 şi c – câtul şi r - restul împărţirii lui a la 11. Rezultă relaţia :
a = 11c + r .
Dar
a = 7x7n + 4x7n + 11 + 6 = 11x ( 7n + 1 ) + 6, deci am obţinut
c = 7n + 1
Pentru n = 4k rezultă 7n = 74k iar 74k are ultimele două cifre „ 01 „
Asadar ultimele două cifre ale câtului sunt „ 02 „

16. Determinaţi cifrele a, b, c, d astfel abcd + bcda = 4576

Soluţie
Avem succesiv : abcd + bcda = 1001a + 1100b + 110c + 11d =
= 11( 91a + 100b + 10c + d ) iar relaţia din enunţ devine succesiv :
11( 91a + 100b + 10c + d ) = 11 x 416 sau
91a + 100b + 10c + d = 416
Cub a, b ≠ 0 rezultă :
 pentru a = 1 avem : 100b + 10c + d = 416 – 91 = 325 , deci b = 3, c = 2 şi d = 5 ;
aşadar o primă soluţie este a = 1, b = 3, c = 2 şi d = 5 ;
 pentru a = 2 avem : 100b + 10c + d = 416 – 182 = 234 , deci b = 2, c = 3 şi d = 4 ;
aşadar o a doua soluţie este a = 2, b = 2, c = 3 şi d = 4 ;
 pentru a = 3 avem : 100b + 10c + d = 416 – 273 = 143 , deci b = 1, c = 4 şi d = 3 ;
aşadar o a treia soluţie este a = 3, b = 1, c = 4 şi d = 3 ;
 pentru a > 4 nu avem soluţii ..
70
17. Considerăm numerele naturale nenule mai mici sau egale cu 2012
şi care nu se divid cu 3. Câte pătrate perfecte avem ?

Soluţie
Cum 452 = 2025 > 2012 şi 442 = 1936 < 2012 rezultă că pătratele perfecte în
mulţimea dată sunt :
11 = 1; 22 = 4; 32 = 9; 42 = 16; 52 = 25; …, 102 = 100; 112 = 121; … ; 302 = 900; 312 = 961;
322 = 1024; … ; 422 = 1764; 432 = 1849; 442 = 1936.
deci în total 44 de pătrate perfecte.
Dintre toate aceste eliminându-le pe cele care nu se divid cu 3, respectiv 14 numere
rezultă că rămân 44 – 14 = 30 numere.

18. Fie numerele naturale a şi b astfel încât :


a2  a  1 a2  a  2 a2  a  3 a 2  a  2012
   ...   2012
b2  b  1 b2  b  2 b2  b  3 b 2  b  2012
arătaţi că a = b .

Soluţie
1) Fie a < b. Atunci pentru k Є N, k Є { 1, 2, 3, …, 2012 }, rezultă :
a2 < b2; a2 + a + k < b2 + b + k, deci
a2  a  k
1 (1)
b2  b  k
Dând valori lui k de la 1 la 2012 şi însumând toate inegalităţile de tipul ( 1 )
obţinem :
a2  a  1 a2  a  2 a2  a  3 a 2  a  2012
   ...   1  1  1  ...  1
b2  b  1 b2  b  2 b2  b  3 b 2  b  2012
de 2012 ori
sau
a2  a  1 a2  a  2 a2  a  3 a 2  a  2012
   ...   2012 (2)
b2  b  1 b2  b  2 b2  b  3 b 2  b  2012
2) Fie a > b. Atunci pentru k Є N, k Є { 1, 2, 3, …, 2012 }, rezultă
a2 > b2; a2 + a + k > b2 + b + k, deci
a2  a  k (3)
1
b bk
2

Dând valori lui k de la 1 la 2012 şi însumând toate inegalităţile de tipul ( 3 )


obţinem :
a2  a  1 a2  a  2 a2  a  3 a 2  a  2012
   ...   1  1  1  ...  1
b2  b  1 b2  b  2 b2  b  3 b 2  b  2012
de 2012 ori
sau
a2  a  1 a2  a  2 a2  a  3 a 2  a  2012
   ...   2012 (4)
b2  b  1 b2  b  2 b2  b  3 b 2  b  2012
3) Rămâne ca a = b, caz în care pentru k Є N, k Є { 1, 2, 3, …, 2012 }, rezultă
a2 = b2; a2 + a + k = b2 + b + k, deci
a2  a  k (5)
1
b bk
2

Dând valori lui k de la 1 la 2012 şi însumând toate egalităţile de tipul ( 5 )


obţinem :

71
a2  a  1 a2  a  2 a2  a  3 a 2  a  2012
   ...   1  1  1  ...  1
b2  b  1 b2  b  2 b2  b  3 b 2  b  2012
de 2012 ori
sau
a2  a  1 a2  a  2 a2  a  3 a 2  a  2012
   ...   2012 (6)
b2  b  1 b2  b  2 b2  b  3 b 2  b  2012

19. Determinaţi numerele naturale diferite x, y, z pentru care


A = 2x x 3y x 5z are cel mai mic număr de divizori naturali.
Care dintre aceste numere este cel mai mic şi cel mai mare ?
Soluţie
Vom considera inclusiv exponentul nul .
Evident pentru x = 0, y = 1 şi z = 2 sau x = 0, y = 2 şi z = 1 sau x = 1, y = 0 şi z = 2
sau x = 1, y = 2 şi z = 0 sau x = 2, y = 0 şi z =1 sau x =2, y = 1 şi z = 0 numărul
va avea cel mai mic număr de divizori, respectiv, următorii :

x y z A Divizori
1 2
0 1 2 1x3 x5 3; 5; 3 x 5; 3 x 52
0 2 1 1x32x51 3, 5, 3 x 5, 32 x 5
2
1 0 2 2x1x5 2, 5, 2 x 5, 2 x 52
1 2 0 2x32x1 2, 3, 2 x 3, 2 x 32
2 0 1 22x1x5 2, 5, 2 x 5, 22 x 5
2
2 1 0 2 x3x1 2, 3, 2 x 3, 22 x 3
Cel mai mic număr va fi Am = 22 x 3 x 1 = 12, iar cel mai mare număr va fi
A = 1 x 31 x 52 = 75
ab ca a 4
20. Aflaţi numerele abc pentru care   .
5 3 4

Soluţie
Fie k valoarea comuna a rapoartelor. Rezultă :
ab ca a 4
   k , de unde obţinem :
5 3 4
10a + b = 5k (1)
10c + a = 3k (2)
a4 = 4k (3)
Din ( 3 ) rezultă a = nr.par, adică a Є { 2, 4, 6, 8 }.
Din ( 1 ) pentru a = 8 şi b = 9 rezultă 5k = 89, deci k ≤ 13.
Relaţia ( 3 ) este verificată doar pentru a = 2, pentru care obţinem 4k = 16, deci k = 4
( pentru k > 2 rezultă k > 13 care nu convine ).
Pentru a = 2 şi k = 4 obţinem :
20 + b = 20, deci b = 0 şi respectiv
10c + 2 = 12, deci c = 1
Aşadar soluţia este abc = 201.

21. Fie A mulţimea numerelor naturale care împărţite la 3 dau restul 1


şi împărţite la 5 dau restul 3.
a) Aflaţi cele mai mici două elemente ale lui A ;
b) Arătaţi că în mulţimea A nu există pătrate perfecte .
c) Dacă n aparţine leui A, aflaţi restul împărţirii lui n la 15.

72
Soluţie
a) Fie a numărul natural pentru care au loc relaţiile
a = 3p + 1 (1)
şi
a = 5q + 3 (2)
Avem evident
3p + 1 = 5q + 3
3p = 5q + 2 = 3q + 2( q + 1 )

3q  2(q  1) 2(q  1)
p q (3)
3 3
Din (2, 3) = 1, rezultă q + 1 = 3k , de unde q = 3k – 1 şi atunci

p = 3k – 1 + 2k = 5k – 1

Astfel avem
a = 3p + 1 = 3(5k – 1 ) + 1 = 15k – 2
sau
a = 5q + 3 = 5(3k – 1 ) + 3 = 15k – 2 ( se verifică )
Cele mai mici două valori se obţin pentru k = 1 când a = 13 şi respectiv
pentru k = 2 când a = 28 .

b) Pentru k = nr. par avem u(15k) = 0 şi atunci u( 15k – 2 ) = 8, iar pentru k = nr. impar
avem u(15k) = 5 şi atunci u(15k – 2 ) = 3.
Deci indiferent valorile lui k elementele mulţimii A sunt numere naturale care se
termină în 3 sau în 8 care nu pot fi terminaţiile unui pătrat perfect.

c) Avem succesiv n = 15k – 2 = 15k - 2 – 13 + 13 = 15k – 15 + 13 = 15(k – 1 ) + 13 ,


deci restul împărţirii lui n la 15 este 13 .

22. Considerăm şirul de fracţii


1 1 2 1 2 3 1 2 3 4
; ; ; ; ; ; ; ; ; ;.....
1 2 1 3 2 1 4 3 2 1
a) Scrieţi următoarele două fracţii ale şirului ;
9
b) Pe ce loc se află în acest şir fracţia ?
6
c) Ce fracţie ocupă locul 2008 în acest şir ?

Soluţie
a) Se observă că în şirul de fracţii,pentru un număr natural n se găsesc fracţiile de forma
1 2 3 4 n 1 n
; ; ; ;...; ;
n n 1 n  2 n  3 2 1
şi atunci următoarele două fracţii după grupul corespunzător lui
1 2 3 4
n = 4 ( adică ; ; ; ) sunt fracţiile corespunzătoare din grupul pentru n = 5 , adică
4 3 2 1
1 2
; ;....
5 4
b) Pentru un grup de fracţii corespunzătoare unui număr natural n , adică

73
1 2 3 4 n 1 n
; ; ; ;...; ; se observa că suma dintre numărător şi numitor
n n 1 n  2 n  3 2 1
rămâne constantă şi anume egală cu n + 1.
9
Atunci fracţia corespunde grupului lui n = 14 deoarece 9 + 6 = 15
6
13(13  1) 1314
Până la grupul n = 13 sunt 1 + 2 + 3 + … + 13 =   78 de termeni
2 2
1 2 3 4 5 6 7 8 9
Scriem grupul lui n = 14 şi avem ; ; ; ; ; ; ; ; ....
14 13 12 11 10 9 8 7 6
9 9
Până la fracţia în grup sunt 9 termeni deci fracţia ocupă locul 78 + 9 = 87 .
6 6

c) Determinăm valoarea lui n pentru corespunzător grupului care îl conţine pe al 2008 –lea
termen.
n(n  1)
Numărul total de termeni până la n - 1 este 1 + 2 + 3 + ... + (n -1) = şi trebuie să
2
n(n  1)
avem <2008 (1)
2
n( n  1)
Numărul total de termeni până la n este 1 + 2 + 3 + ... + n = şi trebuie să avem
2
n( n  1)
2008  (2)
2
Între ( 1 ) şi ( 2 ) rezultă :
n(n  1) n(n  1)
<2008 <  n( n – 1 ) < 4016 < n( n + 1 ) (3)
2 2

Din ( 3 ) rezultă 62 x 63 = 3906 < 4016 < 4032 = 63 x 64  3906 < 4016 < 4032
deci fracţia care ocupă locul 2008 se regăseşte printre termenii grupului pentru n = 63,
1 2 3 62 63
adică grupul ; ; ;...; ; ( vezi suma dintre numărător şi numitor este egală
63 62 61 2 1
cu 64 ) care în total are 63 de termeni.
62  63
Până la n = 62 sunt în şir un număr de 1 + 2 + 3 + ... + 62 = =1953 fracţii.
2
Cum 2008 – 1953 = 55, rezultă că din grupul lui n = 63 vom lua a 55-a fracţie, adică pe
55
locul 2008 se va afla fracţia . ( vezi 55 + 9 = 64 ).
9

23. a) Câte perechi de numere naturale ( m, n ) verifică egalitatea


2m + 5n = 2013 ?
b) Dacă m şi n sunt numere naturale astfel încât verifică egalitatea
2m + 5n = 2013, atunci m + n se divide cu 3.

Soluţie
a) Avem succesiv 2m = nr. par şi 2013 = nr.impar. Rezultă că 5n = nr. impar, deci n =
nr.impar, adică n = 2k + 1.
Pentru n= 2k + 1 avem succesiv :
2m + 5(2k + 1 ) = 2013
2m + 10k + 5 = 2013
2m = 2008 – 10k, deci m = 1004 – 5k
74
Cum m este nr.natural, rezultă
m≥0
1004 – 5k ≥ 0
5k≤ 1004
1004
k
5
k ≤ 200 , deci sunt 200 de perechi de numere naturale (m, n) pentru care are
loc relaţia 2m + 5n = 2013

b) Din n = 2k + 1 şi m = 1004 – 5k , rezultă

m + n = 1004 – 5k + 2k + 1 = 1005 – 3k = 3(335 – k ) , deci m + n se divide cu 3 .

24. Demonstraţi că nu există a, b numere naturale, astfel încât


a + b = 42 şi [a, b] = 252, unde [a, b] reprezintă cel mai mic multiplu
comun al numerelor a şi b.

Soluţie
Din [a, b] = 252 rezultă că cel puţin un număr dintre a şi b este număr par.
Dacă a este număr par, atunci din a + b = 42 rezultă că şi b este număr par.
Dacă a şi b sunt ambele numere pare, atunci cel mai mare divizor comun al lor
va fi un număr, adică de forma 2k , adică (a, b) = 2k.
Dacă (a, b) = 2k, atunci putem scrie relaţiile :
a = 2kp şi b = 2kq cu (p, q) = 1 (1)
Folosind ( 1 ) avem succesiv :
 din a + b = 42 , rezultă 2kp + 2kq = 42 sau 2k( p + q ) = 42 sau
k( p + q ) = 21 (2)
 mai folosim relaţia în divizorul comun, multiplul comun şi produsul a două
numere naturale, respectiv a x b = (a, b) x [a, b], de unde rezultă
2kp x 2kq = 2k x 252 sau 2kpq = 252 sau
kpq = 126 (3)
Din ( 1 ) şi ( 2 ) rezultă succesiv :

 din k( p + q ) = 21, rezultă k impar şi p + q de asemenea impar şi atunci


rezultă soluţiile ( k, (p + q ) ) Є { ( 1, 21), ( 3, 7 ), ( 7, 3 ), ( 21, 1 ) }
 pentru k = 1 din ( 2 ) avem pq = 126 = 2 x 32 x 7 şi cum p + q = 21 rezultă că nu
găsim soluţii numere naturale, care să verifice cele două relaţii ;
 pentru k = 3 din ( 2 ) avem pq = 42 = 2 x 3 x 7 şi cum p + q = 7 rezultă că nu
găsim soluţii numere naturale, care să verifice cele două relaţii ;
 pentru k = 7 din ( 2 ) avem pq = 18 = 2 x 32 şi cum p + q = 3 rezultă că nu
găsim soluţii numere naturale, care să verifice cele două relaţii ;
 pentru k = 21 din ( 2 ) avem pq = 6 = 2 x 3 şi cum p + q = 1 rezultă că nu
găsim soluţii numere naturale, care să verifice cele două relaţii ;

Epuizând toate variantele perechilor ( k, ( p + q ) ) rezultă că nu există numerele


naturale a şi b astfel încât a + b = 42 şi [a, b] = 252 .

75
25. Ştiind că 4a + 5b + 2c = 16 şi 2a + b + 2c = 6 calculaţi produsul
( a + 2b )( 3a + 3b + 2c ) .

Soluţie
Din cele două relaţii prin însumarea lor, membru cu membru, obţinem :
6a + 6b + 4c = 22 sau 3a + 3b + 2c = 11
Iar prun scăderea lor, membru cu membru, obţinem :
2a + 4b = 10 sau a + 2b = 5
Atunci rezultă ( a + b )( 3a + 3b + 2c ) = 55

26. Determinaţi valorile naturale ale numărului natural x pentru care este
îndeplinită relaţia : ( 3x + 1 ) | ( 7x + 9 )

Soluţie
Din ( 3x + 1 ) | ( 3x + 1 ) şi ( 3x + 1 ) | ( 7x + 9 ) rezultă

( 3x + 1 ) | 3( 7x + 9 ) – 7( 3x + 1 )  ( 3x + 1 ) | 21x + 27 – 21x – 7 

 ( 3x + 1 ) | 20 , deci ( 3x + 1 ) Є { 1, 2, 4, 5, 10, 20 } de unde singurele soluţii


pentru x sunt

x Є { 0, 1, 3 }

ROTARU OANA NICOLETA

Clasa a VI – a B

Şcoala generală nr. 11 „ George Tutoveanu „


Bârlad, iulie 2012

76

S-ar putea să vă placă și